SlideShare ist ein Scribd-Unternehmen logo
1 von 56
Downloaden Sie, um offline zu lesen
Estrategias
Metodológicas
para el área de
Matemática en EBA
MÓDULO
MINISTERIO DE EDUCACIÓN
Ministro de Educación
Jaime Saavedra Chanduví
Viceministro de Gestión Pedagógica
Flavio Felipe Figallo Rivadeneyra
Viceministro de Gestión Institucional
Juan Pablo Silva Macher
Directora General de Educación Básica Alternativa, Intercultural Bilingüe
y de Servicios Educativos en el Ámbito Rural - DIGEIBIRA
Elena Antonia Burga Cabrera
Dirección de Educación Básica Alternativa - DEBA	
Luis Alberto Hiraoka Mejía
Módulo de Orientaciones para el docente del Área de Matemática de EBA
©	 Ministerio de Educación
	 Av. De la Arqueología, cuadra 2, San Borja
	 Lima, Perú
	 Teléfono: 615-5800
	www.minedu.gob.pe
Primera Edición
Octubre de 2015
3Dirección de Educación Básica Alternativa
PRESENTACIÓN
El propósito de este módulo es contribuir a mejorar la práctica pedagógica del
docente de Educación Básica Alternativa mediante el fortalecimiento de sus
capacidades para construir y dinamizar procesos de enseñanza y aprendizaje
de la matemática, centrados en el estudiante y su contexto.
A través de la comprensión y aplicación de este módulo los docentes del Área de
Matemática serán capaces de:
•	 Identificar situaciones reales que enfrentan y resuelven los estudiantes
haciendo uso de conocimientos matemáticos.
•	 Diseñar experiencias de aprendizaje, en base a las situaciones identificadas,
que integren los saberes del estudiante con procesos constructivos y
significativos del aprendizaje de la matemática.
•	 Desarrollar las experiencias de aprendizaje en armonía con la cultura de los
estudiantes y el desarrollo de sus capacidades para el logro de competencias
matemáticas.
4 Estrategias Metodológicas para el Área de Comunicación Integral
ÍNDICE
Fundamentación del área de matemática y situaciones problemáticas
1. Fundamentación del área de matemática 	 6
2. Situaciones problemáticas 	 11
Cómo aprendemos matemática, sus procesos y componentesen EBA
1.	 ¿Cómo aprendemos matemática? 	 19
2.	 Procesos del área de matemática según el DCBN EBA 	 23
	 2.1. Resolución de problemas 	 23
	 2.2. Razonamiento y demostración 	 23
	 2.3. Comunicación matemática 	 23
3.	 Componentes del área de matemática según el DCBN EBA 	 24
	 3.1. Sistemas numéricos y funciones 	 24
	 3.2. Geometría y medida 	 25
	 3.3. Estadística y probabilidad 	 27
Orientaciones para aplicar estrategias en el área de matemática
1. 	 Secuencia didáctica de Brousseau 	 32
2. 	 La investigación 	 41
3. 	 Resolución de problemas 	 48
Unidad 1
Unidad 2
Unidad 3
Unidad 1
Fundamentación del área de
matemática y situaciones
problemáticas
6 Módulo de Orientaciones para el docente del Área de Matemática de EBA
Fundamentación del área de matemática1
Docente A:
A través del aprendizaje de la matemática,
los estudiantes de EBA desarrollan
capacidades para resolver una variedad
de operaciones y problemas tipo. De
este modo, están mejor preparados para
continuar estudios superiores.
Docente B
A través del aprendizaje de la matemática,
los estudiantes de EBA desarrollan
capacidades para comprender y resolver
las situaciones problemáticas que
enfrentan. De este modo, aprenden a
actuar y pensar matemáticamente ante
diversas situaciones.
1.	 En relación a las posturas de los docentes A y B: ¿qué aspectos compartes y cuáles no? ¿Por qué?
2.	 ¿Cómo diseñarías una Tabla para presentar tus respuestas?
7Dirección de Educación Básica Alternativa
La matemática no es ajena a los jóvenes y adultos, descubren su importancia a partir de la
necesidadde resolverlassituaciones problemáticasquesepresentan enla vidacotidiana:comprar,
vender, organizar sus cuentas, tomar decisiones en base a datos cuantitativos, diseñar y describir
formas, interpretar gráficos, representar rutas, estimar tiempos, analizar y reflexionar sobre el
costo de los servicios básicos, etc. Todo ello demanda el uso de conceptos, procedimientos y
estrategias matemáticas.
El desarrollo de competencias matemáticas en los jóvenes y adultos debe partir de situaciones
problemáticas reales que correspondan a su experiencia de vida, atiendan sus urgencias y las
demandas de la sociedad. Las situaciones deben ser analizadas y entendidas en su complejidad
a través de los conocimientos que ofrece la matemática, dado que el estudiante se enfrenta a
ellas en su día a día y necesita solucionarlas.
Actuar y pensar matemáticamente es fundamental para el estudiante, de este modo amplía su
horizonte y capacidades para plantear y resolver problemas en los que interviene la matemática,
utiliza y elabora estrategias, razona, argumenta, representa y comunica.
	 Dos facetas de la matemática
La matemática está en permanente construcción, es fruto de un proceso histórico. Concebirla
como ciencia acabada, exacta y rigurosamente deductiva tiene consecuencias negativas en el
plano de la enseñanza y aprendizaje, pues distorsionan la orientación y las actividades que
se proponen y desarrollan.
Los aspectos deductivos de esta ciencia son una faceta de ella, evidente en la cadena de pasos
lógicos y rigurosidad que lleva a una verdad irrefutable. La otra faceta está asociada a su
proceso de elaboración, incluye aspectos como la intuición, conjeturas, exploración, creatividad,
motivaciones y las emociones.
En el siguiente gráfico se muestra las distintas estrategias que emplean algunas personas para
calcular mentalmente el cambio de S/150 a euros:
Utilizar la matemática e involucrarse con el papel que tienen en el mundo es fundamental
paraquelosjóvenesyadultosenriquezcanelconocimientoquetienendelarealidad,tomen
decisiones informadas y desarrollen la capacidad de hacer juicios bien fundamentados.
8 Módulo de Orientaciones para el docente del Área de Matemática de EBA
	 Las matemáticas son un producto cultural
Toda persona desarrolla los procesos formativos de su personalidad en el ámbito de determinada
cultura. No hay cultura sin personas y no habrá consciencia ni pensamiento sin cultura.
La actividad matemática y las habilidades, actitudes y conocimientos asociados son un
componente importante de cada cultura y medio social, facilitan la relación y comunicación
entre personas y con el entorno, contribuyen a las actividades científicas y tecnológicas y a la
mejor comprensión de uno mismo.
Al expresar determinadas pautas de racionalidad e involucrar un lenguaje se desarrollan
capacidades de relación, representación y cuantificación. Por
ejemplo, al elaborar estadísticas empleamos procedimientos
similares, comunicamos los resultados usando términos
consensuados e interpretamos información en base a
criterios comunes.
Contribuye a expresar y potenciar múltiples actividades,
entre ellas las científicas y tecnológicas.
En el mundo se han identificado seis actividades
fundamentales que constituyen fuente para el desarrollo
de la matemática: contar, localizar, medir, diseñar, jugar y
explicar.
(DIBUJO ADAPTADO DE “LAS REMESAS“, de Jorge Llieff. No incluye los cálculos)
1 euro = S/3,43 y 10 euros S/34,3
20 euros = S/68,6 y 40 euros S/137,2
S/150 – S/137,2 = S/12,8 casi 4 euros
Entonces 40+4 = casi 44 euros
CASA DE CAMBIO
S/1 = 0,29 euros
S/100 = 29 euros
S/50 = 14,5 euros
S/150=29+14,5=43,5 euros
S/100 es 29 euros
S/150 es como 29+15
Casi 44 euros
S/1=0,29 euros
S/1,5 es casi 0,44
Sí, es casi 44 euros
9Dirección de Educación Básica Alternativa
	 Un enfoque intercultural de la educación matemática
Los estudiantes de EBA son parte de una sociedad multicultural y plurilingüe. La manera peculiar
cómo los grupos sociales y culturales –no solo pueblos originarios– construyen (o reconstruyen)
los conocimientos, desarrollan sus habilidades y establecen sus actitudes ha recibido especial
atención de la disciplina llamada “Etnomatemática”.
El currículo debe promover un proceso de interacción cultural entre el docente y el estudiante;
con el objetivo de que este último reciba una educación matemática enriquecida, pertinente para
los múltiples escenarios, reconstruyendo crítica y comprensivamente las conceptualizaciones,
procedimientos y valores de la cultura matemática.
El tratamiento curricular del área de matemática
debe tener un enfoque intercultural. Los procesos
de enseñanza y aprendizaje de la matemática están
inmersos en la basta diversidad cultural inherente al
país y el mundo globalizado. Es responsabilidad del
docente ayudar a procesar, crítica y creativamente, la
herencia cultural matemática.
Por ejemplo, los jóvenes y adultos utilizan la
matemática al desempeñar procesos productivos de
los que depende su subsistencia. En éstas situaciones
se evidencia la forma de entender y relacionarse con
los conceptos y procedimientos matemáticos: medir,
seguir secuencias, contar, estimar precios de venta,
calcular utilidades y pérdidas, etc.
	 Educación matemática y equidad
En ocasiones la matemática ha sido utilizada para discriminar, desalentando a los estudiantes
con dificultades de aprendizaje e influyendo en su retiro o abandono de las aulas.
Lademocratizacióndelaculturaexigelaincorporacióndetoda nuestrapoblaciónal conocimiento,
los valores y las actitudes inherentes a la educación básica. Es decir, un núcleo de conceptos,
procedimientos y actitudes matemáticas que debe formar parte del bagaje cultural que domina
cada ciudadano.
En los distintos niveles de concreción curricular se debe tener especial cuidado en identificar lo
anterior, distinguiéndolos de aquellos orientados a la formación ocupacional o especialización
10 Módulo de Orientaciones para el docente del Área de Matemática de EBA
con alta exigencia. Por ejemplo, todos debemos plantear y resolver problemas utilizando
determinados conceptos, procesos, algoritmos y estrategias; sin embargo, cada uno parte de
situaciones reales de su contexto (sembrar, medir, comprar, vender, etc.) y progresivamente
las amplía.
Peculiaridades en la enseñanza y el aprendizaje de los estudiantes de
EBA
Los estudiantes de EBA desarrollan ciertas formas de hacer y aprender la matemática. Los
docentes debemos identificarlas, investigar y potenciarlas. Por ejemplo, sus procedimientos de
cálculo son distintos a los que provienen del contexto académico, ello debido a que aprendieron
a calcular ante la necesidad de realizar transacciones de compra y venta.
Tienen capacidades, habilidades y destrezas que han desarrollado y continúan aplicando
en diferentes grados. Algunas de sus fortalezas son el cálculo mental, las estimaciones, las
comparaciones cuantitativas, los procedimientos de localización, el uso de la calculadora, entre
otras. Ello explica su desempeño exitoso en actividades de comercio minorista e incluso
mayorista, la facilidad para movilizarse en geografías complejas y la diversidad de labores que
realizan; pese al nulo o bajo nivel educativo.
	 ¿Qué debe hacer el docente?
El docente debe facilitar oportunidades de aprendizaje para que el estudiante sea capaz de
valorar y utilizar el aporte de la matemática en la comprensión de su entorno físico y cultural,
identificando y resolviendo problemas relacionados con su contexto. Además, reconocer que
los estudiantes han construido saberes matemáticos y estrategias en su interacción con el medio
y las personas, por lo que su experiencia constituye el punto de partida para el desarrollo de la
competencia matemática.
El Área de Matemática promueve experiencias significativas para que los estudiantes
construyan sus aprendizajes, en forma individual y en cooperación con otros, en un
encuentro enriquecedor del saber matemático desarrollado en su experiencia de vida, con
las capacidades, conocimientos y actitudes propias de la matemática.
DCBN EBA
11Dirección de Educación Básica Alternativa
Es importante que los estudiantes de EBA descubran en la matemática un instrumento o herramienta
intelectual que les ayude a solucionar diversas situaciones que se les presenta. En ese sentido el
rol del docente es generar experiencias de aprendizaje, partiendo de situaciones de la vida
cotidiana.
	 ¿Qué es una situación problemática?
Una situación problemática describe una actividad
o escenario donde se desenvuelve el estudiante
haciendo uso de los conocimientos matemáticos.
No es cualquier situación, debe permitir explicar
la funcionalidad de dichos conocimientos, de
este modo podrán ser utilizadas para dinamizar
aprendizajes.
En la situación se evidencian necesidades, intereses,
desafíos o dificultades. El estudiante explora,
moviliza y desarrolla saberes; es decir, evidencia sus
competencias y capacidades de forma integrada.
Las situaciones problemáticas provienen del contexto real, social, científico y matemático.
A.	 Situaciones del contexto real del estudiante
Las situaciones de contexto real se refieren a la experiencia personal, familiar, laboral,
social, comunitaria y pública de la vida de los estudiantes.
Comprenden desafíos relacionados a la persona y sus percepciones, familia y grupo
de pares: comprar y pagar, utilizar medidas al preparar alimentos, transportarse, viajar,
finanzas personales, cuidado de la salud, etc.
Situaciones problemáticas2
12 Módulo de Orientaciones para el docente del Área de Matemática de EBA
Aprendí a movilizarme en la
ciudad, no es igual que aquí.
Ahora calculo y optimizo
mi tiempo en función a las
rutas, comparo los costos
de los diferentes medios
de transporte y calculo mi
presupuesto
Situación: movilizarse para realizar las actividades cotidianas
Las personas utilizamos la matemática para calcular el presupuesto que requerimos para
movilizarnos, seleccionar la ruta más adecuada, optimizar los tiempos, etc. Realizamos
cálculos simples y planteamos problemas a partir de situaciones problemáticas propias
del entorno inmediato y la rutina cotidiana.
Para realizar éstas acciones y otras similares necesitamos tener nociones previas y
ejecutar procesos que requieren pensar y actuar matemáticamente ante situaciones
que involucran números, operaciones, cantidades, equivalencias, cambios, formas,
movimientos, localización, gestionar datos, etc.
Observa
13Dirección de Educación Básica Alternativa
Reflexiona
1.	 ¿Cuánto dinero gastas mensualmente en movilizarte a tu trabajo? ¿Qué distancia recorres?
¿Cuánto tiempo demoras?
2.	 ¿Cuál es la mejor oferta de yogurt? ¿Por qué?
El docente orienta a los estudiantes hacia el planteamiento y resolución de problemas
referidos a sus contextos de vida y, progresivamente, introduce problemas que no siendo
exclusivos de esos contextos incluyan escenarios familiares y requieran modelos matemáticos
para su  solución. Durante este proceso el estudiante:
•	 Matematiza las situaciones: reflexiona, reconoce y extrae lasmatemáticas contenidas
en la situación.
•	 Realiza procesos de conexión que involucran ideas y procedimientos matemáticos.
•	 Razona, analiza y comunica operaciones matemáticas.
•	 Utiliza el razonamiento matemático.
•	 Utiliza y construye estrategias.
3. 	 ¿En qué situaciones de tu día a día utilizas la matemática?
4. 	 ¿Cómo describirías dos situaciones de contexto real?
14 Módulo de Orientaciones para el docente del Área de Matemática de EBA
Observa
B. 	 Situaciones del contexto social y científico
Las situaciones de contexto social y científico están relacionados con la naturaleza, la vida,
las ciencias de la salud, las ciencias sociales, los conocimientos científicos y el uso de la
tecnología.
Si bien es cierto las personas están involucradas en todas las situaciones, en la categoría
de contexto social y científico el foco está en la perspectiva de retos relacionados con
las ciencias naturales, sociales, la ciencia y la tecnología: transporte público, sistemas de
votación, gobierno, políticas públicas, demografía, publicidad, estadísticas nacionales y
económicas.
Miércoles 4 de Abril del 2012 GESTIÓN
Resuelve
•	 ¿Cómo comunicarías oralmente esta
información gráfica?
•	 ¿Qué opinas de la demanda y oferta del
transporte urbano en Lima?
Fuente: MACROCONSULT
Oferta y Demanda del Transporte Urbano en Lima (2012)
15Dirección de Educación Básica Alternativa
Reflexiona
Observa
Las situaciones descritas requieren utilizar y desarrollar modelos matemáticos
que implican identificar información; utilizar, describir, explicar y evaluar
el comportamiento de un sistema de datos mediante el uso de variables,
conceptos, operaciones y expresiones matemáticas eidentificarlascondiciones,
restricciones y relaciones planteadas.
La persona debe tener capacidad para transformar el problema a una forma
matemática e interpretar o valorar los resultados o modelo obtenido en relación
al problema original. En este proceso es posible que utilice directamente un
modelo matemático, lo modifique o combine. El desarrollo de modelos hace
posible que la persona encuentre significado a sus saberes matemáticos y los
construya de forma progresiva al contrastarlos con la realidad.
Ante el crecimiento de la población
mundial la ciencia y la tecnología tienen
un rol importante para asegurar su
subsistencia en condiciones de calidad.
Además, las personas debemos ser más
responsables.
¿Cómo contribuye la matemática a este
propósito?
Situación: optimización del agua potable
Optimizar el uso del agua potable en el hogar, trabajo u otro espacio implica estimar el
volumen de agua consumida en las actividades cotidianas.
Identifica dos ejemplos del contexto social y científico que evidencien la utilización de la
matemática.  Descríbelos.
16 Módulo de Orientaciones para el docente del Área de Matemática de EBA
C.	 Situaciones de contexto matemático
Las situaciones de contexto matemático son retos o desafíos del propio conocimiento
matemático.
Utilizar el conocimiento matemático para resolver situaciones problemáticas implica
enfrentar y resolver retos o desafíos del propio contexto matemático.
	Examina:
	 •	 El 50% más el 20% de 2000 es 200
	 •	 1/3 de 300 es 100
Situación: representación gráfica (contexto matemático)
Presentar cada resultado del Estudio realizado por la ONG “Lima Cómo Vamos” utilizando
distintos tipos de gráficos estadísticos.
Resolver la situación requiere superar desafíos del contexto matemático:
•	 Comprender nociones de porcentaje.
•	 Comprender procedimientos propios de la estadística.
•	 Elaborar diversos tipos de gráficos estadísticos.
•	 Analizar las diversas posibilidades de representación gráfica y elegir la más
adecuada para comunicar cada resultado.
El estudiante necesita dominar conocimientos del contexto matemático y conocer
información que le permita resolver la situación. Por ejemplo:
Conocimientos
-	 Área del rectángulo: concepto y cálculo.
-	 Multiplicación y división: números de dos cifras enteras y un decimal.
Información
-	 Medidas del triángulo
Situación: Hallar el área de un triángulo de 10 cm de base y 7 cm de altura.
Examina:
17Dirección de Educación Básica Alternativa
Reflexiona
Situación: Pintar el cerco externo e interno de una IE
Información:
•	 Medidas del cerco: 225 m de largo x 4 m de alto
•	 Venta de pintura en dos presentaciones: balde de 5 galones= S/. 105 y balde de 1
galón = S/. 25,50
•	 1 litro de pintura rinde 10 m2
•	 1 galón = 3,8 litros
Preguntas
¿Cuánto es lo mínimo que se puede gastar en la compra de pintura?
¿Qué conocimientos matemáticos utilizaste?
Situación: Elaborar una torta de sorpresas utilizando cartulina y estimar el precio de venta
para ganar el 25% de lo invertido.
Preguntas
•	 ¿Qué conocimientos del contexto matemático necesita dominar el estudiante?
•	 ¿Qué información le darías al estudiante para que pueda  resolver la situación?
De acuerdo a la información que darás al estudiante: ¿cuántos pliegos de cartulina debe
comprar?
Unidad 2
Cómo aprendemos
matemática, sus procesos y
componentes en EBA
19Dirección de Educación Básica Alternativa
Para los jóvenes y adultos de EBA, la matemática está fuertemente vinculada a las actividades
que realizan; han tenido que aprenderla ante la necesidad de no ser engañados al comercializar
sus productos con el mejor precio posible, participar en actividades económicas, etc. Aplicar
la matemática les permite hacer, entender y proyectar con éxito procesos relacionados a su
subsistencia y cotidianidad.
En ese sentido la resolución de problemas es un aspecto fundamental en el desarrollo de la
matemática y también se puede decir que ayuda al estudiante a desarrollar su pensamiento
abstracto y lógico.
	 Enfoque centrado en la resolución de problemas
El enfoque centrado en la resolución de problemas se promueven aprendizajes a través de,
sobre y para la resolución de problemas (Gaulin 2001)
¿Cómo aprendemos matemática?1
La resolución de Problemas debe
plantearse en situaciones significativas
de contexto diverso, pues ello moviliza
el pensamiento matemático.
Los estudiantes deben encontrar
significado a la resolución de problemas,
valorar el conocimiento matemático
que se aplica y establecer relaciones de
funcionalidad.
La matemática se enseña y aprende
resolviendo problemas. Los estudiantes
contruyen nuevos conceptos, descubren
relaciones, elaboran procedimientos y
establecen relaciones.
20 Módulo de Orientaciones para el docente del Área de Matemática de EBA
A través de la resolución de problemas y el entorno del estudiante construye significados,
organiza objetos matemáticos y genera nuevos aprendizajes en un sentido constructivo y creador
de la actividad humana.
Sobre la resolución de problemas, el estudiante explica la necesidad de reflexionar sobre
los procesos empleados al resolver problemas: planeación, estrategias heurísticas, recursos,
procedimientos, conocimientos y capacidades movilizadas.
Para resolver problemas el estudiante enfrenta constantemente nuevas situaciones y problemas.
La resolución de problemas es el proceso central de hacer matemática, es el medio principal
para ver la funcionalidad de la matemática. Permite al estudiante situarse en diversos contextos
para crear, recrear, investigar y resolver problemas; utilizando diversos caminos de resolución, el
análisis de estrategias y formas de representación, la sistematización y comunicación de nuevos
conocimientos, etc.
El estudiante de EBA evidenciará que es competente en matemática si tiene la facultad de actuar
conscientemente en la resolución de problemas o el cumplimiento de exigencias complejas
relacionadas a la matemática, usando flexible y creativamente sus conocimientos y habilidades,
información o herramientas, así como sus valores, emociones y actitudes. De este modo actúa y
piensa matemáticamente ante diversas situaciones problemáticas.
El enfoque basado en la resolución de problemas para el despliegue de capacidades supone     
desarrollar competencias relacionadas a matematizar situaciones problemáticas que implican
construir modelos, utilizar estrategias al formular y resolver problemas, razonar y argumentar la
validez y pertinencia de los resultados alcanzados y comunicar los hallazgos.
Enseñanza
Aprendizaje
Enfoque
centrado en la
resolución de
problemas
“A través de”
Resolución de
problemas
Actuar y pensar
matemáticamente “Sobre la”
“Para la”
21Dirección de Educación Básica Alternativa
Para una mejor comprensión, a continuación se desarrollan conceptos fundamentales asociados
a aprender competencias:
	Competencia
El aprendizaje de una competencia es de carácter longitudinal, se reitera una y otra vez a fin de
que pueda irse complejizando de manera progresiva y permita al estudiante alcanzar niveles
cada vez más altos de desempeño.
Toda competencia implica actuar conscientemente sobre una situación, un problema o un objetivo
para alcanzar un logro esperado; evidenciando dominio, uso flexible y creativo de capacidades
y ética. Requiere saber transferir las capacidades del contexto en que fueron aprendidos a otras
situaciones, seleccionándolas y combinándolas en función del logro esperado.
Alcanzar desempeños competentes es un proceso complejo; no solo porque al actuar en
situación, para alcanzar el logro esperado, se deben evidenciar desempeños idóneos de carácter
vinculante con otras competencias sino porque no es cualquier situación o logro; son aquellos
que contribuyen al desarrollo personal y colectivo, con sentido y compromiso ético.
	 Aprendizaje situado
Las competencias se aprenden en función de situaciones. El concepto de situación es el elemento
central del proceso de aprendizaje. Es en situación que los jóvenes y adultos construyen, modifican
o refutan conocimientos contextualizados y desarrolla competencias a la vez situadas.
No se trata de aprender contenidos disciplinares descontextualizados (área, triángulo, ecuaciones,
etc.) sino de definir situaciones en las que sea posible construir, modificar o refutar conocimientos,
por ende, desarrollar capacidades y evidenciar desempeños competentes.
Las competencias se desarrollan a partir de situaciones identificadas en la experiencia de vida
de los jóvenes y adultos. Estas son fuente de aprendizaje y generadoras de situaciones de
aprendizaje. En este sentido, todo aprendizaje debe ser situado (Lave, 2003).
	 Pensamiento matemático
La competencia es un saber actuar de manera reflexiva y eficiente, tanto en el campo
de las relaciones de las personas con la naturaleza, con los objetos, con las ideas; como
en el de las relaciones sociales. Este saber actuar no alude solamente a una capacidad
manual, técnica, operativa, sino además a un saber cómo, por qué y para qué hacerlo.
Fuente: DCBN EBA
De otro lado, pensar matemáticamente se define como el conjunto de actividades
mentales u operaciones intelectuales que llevan al estudiante a dotar de significado a
lo que les rodea, resolver problemas sobre conceptos matemáticos, tomar una decisión
o llegar a una conclusión, en la que están involucrados procesos como la abstracción,
justificación, visualización, estimación, entre otros.
Cantoral 2005; Molina 2006; Carretero y Ascencio 2008.
22 Módulo de Orientaciones para el docente del Área de Matemática de EBA
Los jóvenes y adultos enfrentan regularmente situaciones que deben resolver haciendo uso
de conocimientos matemáticos: comprar, vender, movilizarse de un lugar a otro, gestionar su
presupuesto y actividad laboral, pagar servicios básicos e impuestos, evaluar préstamos de dinero,
juzgar cuestiones políticas y asuntos públicos, etc. En este proceso desarrollan una variedad de
estrategias y despliegan habilidades relacionadas con el razonamiento (cuantitativo y espacial) y
la intuición, basan su accionar en fundamentos, conceptos, habilidades y estrategias matemáticas.
Es decir, utilizan su pensamiento matemático para resolver problemas y desempeñarse de modo
competente en la vida cotidiana.
El sentido y utilidad de la matemática para los jóvenes y adultos está en el manejo y aplicación
de conceptos, operaciones, cálculos y medidas de uso común para identificar y solucionar
problemas relacionados a su vida personal, laboral, familiar y participación ciudadana; asimismo,
manejar instrumentos y medios tecnológicos, desde calculadoras básicas –ser analfabeto no es
un impedimento para utilizarlas- hasta tecnologías de información y comunicación.
Por otro lado, el ejercicio de su ciudadanía les demanda comprender y reflexionar sobre los
fenómenos sociales y ambientales, asimismo, emitir juicios y propuestas constructivas basadas
en el manejo y comprensión de información cuantitativa.
La matemática no está ausente de la cultura de los jóvenes y adultos ni de los procesos
de asimilación a otras culturas, a la que se incorporan con el objetivo de mejorar su condición
económica. En ambos escenarios el aprendizaje de la matemática es funcional y se constituye en
una herramienta para la sobrevivencia y exige reforzar y desarrollar su pensamiento matemático.
Vemos que el desarrollo del pensamiento matemático en jóvenes y adultos surge ante la
necesidad de comprender y solucionar situaciones de vida concretas, estas son oportunidades
para movilizar y poner en funcionamiento sus conocimientos, estrategias y recursos. Viabilizar
sus competencias matemáticas implica superar contenidos y enfoques desvinculados de su
quehacer e interés, desarrollar metodologías y materiales educativos que posibiliten el uso de la
matemática como recurso básico para la vida, generar experiencias de aprendizaje que rescaten
sus saberes y les posibiliten plantear y resolver problemas en base a situaciones problemáticas
de su contexto personal, cultural, natural y social, asociadas a nuevas demandas del mundo del
trabajo y la ciudadanía activa; desarrollando una variedad de estrategias.
	 Competencia matemática
•	 La competencia matemática es entendida como la capacidad de analizar, razonar y
comunicar según se plantean y resuelven los problemas que surgen del desarrollo
personal y la plena integración en la sociedad de la comunicación.
•	 Engloba tres dimensiones adecuadamente
relacionadas: contenidos, procesos y situaciones o
contextos.
•	 Se aplica para resolver los problemas de la vida
adulta y afrontar exigencias de diferente nivel y tipo,
de este modo es un prerrequisito o base para seguir
aprendiendo a lo largo de la vida.
•	 Se demuestra en la ejecución autónoma de sucesivos
procesos cognitivos, entre los que se encuentran las
destrezas básicas de cálculo, se centra en el proceso
y el razonamiento más que en el conocimiento.
23Dirección de Educación Básica Alternativa
El área de matemática en el DCBN EBA integra los procesos fundamentales con los
componentes, los cuales son asumidos como grandes bloques de contenidos. Adiciona
a ello las actitudes que contribuyen a una sólida formación integral. Esta es una forma
de concreción de las intencionalidades educativas. En el DCBN EBA se asumen tres procesos:
	 2.1. Resolución de problemas
Los problemas deben ser formulados y elegidos con la intención de posibilitar que lo aprendido
se consolide y amplíe, asimismo, se construya nuevos conocimientos. Implica identificar
los problemas dentro de contextos reales y superar la tendencia a reducirlos a tratamientos
abstractos y descontextualizados.
Es importante ser consciente del proceso seguido en la resolución de problemas y evaluar el
avance. El proceso de resolución de problemas sirve de contexto para el desarrollo de otros
procesos fundamentales. Al resolverlos necesariamente se razona y comunica y se interconecta
ideas matemáticas y representan.
	 2.2. Razonamiento y demostración
Las actividades de aprendizaje de la matemática deben propiciar que los estudiantes desarrollen
y evalúen argumentos utilizando nociones, conceptos y procedimientos matemáticos. Es decir,
que aprendan a razonar, tanto de manera heurística como deductiva.
Razonar heurísticamente implica hacer uso de la intuición, las conjeturas, la inducción a partir
de regularidades o patrones, tanto en situaciones del mundo real como en objetos simbólicos;
asimismo, preguntarse si son patrones accidentales o si hay razones para que aparezcan.
En forma progresiva se introduce la argumentación deductiva, la simbolización, la abstracción, el
rigor y la precisión. Un error es reducir el razonamiento al adiestramiento en “problemas tipo” o
trabajar un curso de razonamiento matemático en paralelo al desarrollo del área de matemática.
El docente debe propiciar un clima favorable a la libre expresión de ideas, sentimientos y
expectativas, fomentar la discusión e insistir en la elaboración de argumentos. El estudiante debe
aprender a proporcionar los fundamentos o razones de sus decisiones y valorarlas críticamente;
asimismo, derivar las implicancias de una situación hipotética y ser flexible a la modificación de
su punto de vista enbase a argumentos.
	 2.3. Comunicación matemática
Permite expresar, compartir y aclarar ideas; las cuales llegan a ser objetos de reflexión,
perfeccionamiento, discusión, análisis y reajuste. Este proceso involucra emociones y actitudes,
ayuda a dar significado y permanencia a las ideas y difundirlas. Las emociones y actitudes deben
ser moduladas, ya que pueden facilitar u obstaculizar el aprendizaje de la matemática.
La matemática aporta un lenguaje preciso, el cual permite a las personas expresar e interpretar ideas
matemáticas con argumentos convincentes, la exploración sistemática de alternativas, flexibilidad
en el razonamiento y la organización, consolidación y comunicación del pensamiento matemático.
El docente debe posibilitar que los estudiantes incorporen a su habla personal distintas
formas de expresión matemática: numérica, gráfica, geométrica, algebraica y probabilística.
Además, capacitar a los estudiantes para analizar y evaluar las estrategias y el conocimiento
matemático implicado en las actividades de las personas con las que interactúa, comunicándose
con pertinencia y compartiendo un significado y sentido.
Procesos del área de matemática según el
DCBN EBA2
24 Módulo de Orientaciones para el docente del Área de Matemática de EBA
En el DCBN EBA se asumen tres componentes para el área de matemática. Éstos se conciben
como grandes bloques de contenidos:
•		 Sistemas numéricos y funciones
•		 Geometría y medida
•		 Estadística y probabilidad
Cada componente es un medio para describir, comprender e interpretar los fenómenos naturales
y sociales que han determinado el desarrollo de determinados procedimientos y conceptos
matemáticos propios de situaciones relacionadas al componente; por ejemplo:
•		 Situaciones de cantidad, se modelan desde los sistemas numéricos y funciones.
•		 Situaciones de forma, se representan y relacionan desde la geometría y medida.
•		 Situaciones probabilísticas, se estiman ý representan desde la estadística y probabilidad.
Analicemos cada componente:
	 3.1. Sistemas numéricos y funciones
Nuestra sociedad está teñida por los números, desde nuestras cuentas personales y
actividades cotidianas hasta las prioridades nacionales y mundiales. Aprendemos sobre
ellos pero no siempre lo aplicamos a nuestra cotidianidad. Veamos el siguienteejemplo:
Aprendemos cuando somos capaces de atribuir significado a lo que aprendemos:
“Proceso de Construcción de Significados”. Sin embargo, tambiénaprendemos sin darle
significado al contenido o acción, en ese caso, solo somos capaces de memorizar y repetir
sin entender, por tanto, no somos capaces de transferir ese aprendizajea otro escenario.
Componentes del área de matemática según el
DCBN EBA3
50% + 20% es 70% de
descuento. Son S/30 por
la blusa No señora, el 50% de 100 es
50 y el 20% de 50  es 10.
El descuento es 50+10= 60.
Debe pagar S/40
Blusa S/. 100
Rebaja: 50% + 20%
25Dirección de Educación Básica Alternativa
Lo significativo varía de una a otra persona, los docentes más que generalizar aprendizajes
debemos buscar aquellos que sean lo más significativos posibles para quienes aprenden.
Por ejemplo, en relación a los sistemas numéricos, para nuestros estudiantes las siguientes
situaciones son significativas:
•		 Sueldo (mensual, quincenal, diario)
•		 Deudas y préstamos, pagos y cuotas.
•		 Precios de productos de la canasta familiar.
•		 Ofertas en anuncios publicitarios.
•		 Temperatura, radiación solar.
•		 Consumo de celulares: minutos y soles.
•		 Costo de medicinas, etc.
El componente sistemas numéricos y funciones, desarrollado en el DCBN EBA,
incluye el estudio de los números, sus distintas formas de representarlos, las
operaciones, las relaciones entre ellos y con conjuntos de números, los sistemas
numéricos, el álgebra y las funciones.
La orientación del abordaje es amplia, va más allá del manejo elemental de operaciones
básicas y la destreza operatoria con expresiones algebraicas. Específicamente, en cada
uno de los ciclos de EBA, el abordaje de este componente se centra en:
Ciclo inicial e intermedio Ciclo avanzado
•	 Desarrollo del sentido numérico, de
modo que haya comprensión de
los números.
•	 Relaciones entre los números.
•	 Significado de las operaciones.
•	 Cálculo fluido.
•	 Estimaciones razonables.
•	 Regularidades y funciones, de modo
sistemático.
•	 Identificación, representación y utilización
de las estructuras matemáticas utilizando
el simbolismo apropiado.
•	 Elaboración de modelos elementales para
representar o comprender relaciones
cuantitativas de situaciones o fenómenos
reales.
	 3.2. Geometría y medida
En nuestro mundo encontramos gran diversidad de objetos y situaciones que se relacionan
con geometría y las mediciones. Esta tendencia es creciente en el futuro, por lo que es
importante abordar a través del aprendizaje de la matemática habilidades necesarias para
desenvolverse con éxito en este y otros escenarios.
Los jóvenes y adultos experimentan diversas situaciones en las que se enfrentan a
problemas espaciales, los resuelven con mayor o menor éxito, por lo general, de modo
empírico. Ello les ha permitido construir una serie de referencias, por ejemplo: movilizarse
para ubicar una dirección identificada en un mapa de calles les exige localizar puntos
de referencia y reconocer distancias, construir sus casas exige la aplicación de nociones
geométricas: áreas, altura, peso, volumen, etc.
26 Módulo de Orientaciones para el docente del Área de Matemática de EBA
Pese a tener conocimientos empíricos de geometría logran construir colaborativamente y
con acierto sus viviendas. ¿Es básico y elemental su conocimiento? ¿Aplican las teorías que
nos cuesta tanto entender? ¿Influye el aprendizaje colaborativo?
Lo que aprendemos tiene relación con cómo percibimos nuestro entorno, un hecho o
situación determinada. También con cómo lo observamos, no solo con los ojos sino con
todos nuestros sentidos y emociones involucradas. Veamos cómo aprenden los jóvenes y
adultos en espacios formales y no formales la geometría:
En mi comunidad aprendí  a medir el terreno
para sembrar, calcular cuántas semillas y agua
necesito por hectárea, pesar la cosecha… ¡Ahora
me resulta más fácil aprender lo de aquí!
Optimicé los
cortes. De
cada bloque
de madera,
salieron 10
piezas.
Se necesitará 2m2
más de tejas.
1 galón de pintura da para dos
habitaciones.
La proporción
de la mezcla es
de 2 por 3.
27Dirección de Educación Básica Alternativa
En el medio en el que interactúan, los jóvenes y adultos, enfrentan una serie de situaciones
en las que están en contacto con diversos objetos en los que pueden reconocer formas y
cuerpos geométricos que van desde empaques (cajas de diversas formas en las que viene
artículos que compran o venden) hasta construcciones fabulosas (edificios en los que
trabajan como obreros de construcción) pasando por una serie de artículos tecnológicos
cada vez más sofisticados y modernos (celulares, calculadoras, televisores, etc.).
Aprender geometría relacionada a la serie de situaciones que enfrentan es fundamental
paralosjóvenesyadultos.Lespermitecomprenderyaplicarlosconocimientosmatemáticos
relacionados, interactuar con los objetos optimizando espacios, costos, desarrollar el
sentido de ubicación en el espacio, comprender las propiedades de las formas y cómo se
interrelacionan, etc. Sin embargo, el aprendizaje no solo es cuestión de conocimiento tiene
que ver con cómo perciben el entorno y activan sus capacidades, sentidos y emociones,
además, con la capacidad de modificar la estructura actual transfiriendo lo que se aprendió
a otros escenarios.
El componente geometría y medida, desarrollado en el DCBN EBA aborda el estudio de
las características y propiedades de las figuras y cuerpos geométricos, la localización
y descripción de relaciones espaciales mediante coordenadas y otros sistemas de
representación,lasimetría ylastransformaciones(traslación,reflexión,rotación, ampliación,
reducción) para analizar situaciones matemáticas y del entorno, la comprensión de los
atributos susceptibles de medición de los objetos, y los sistemas de unidades, procesos e
instrumentos de medición.
	 3.3. Estadística y probabilidad
Los estudiantes jóvenes y adultos enfrentan un mundo saturado de información y datos,
los cuales son fuente para la toma de decisiones. Necesitan desarrollar su pensamiento
estadístico para comprenderlos y mejorar sus posibilidades de éxito al interactuar en la
sociedad.
Así por ejemplo, para los estudiantes de EBA
de la zona rural es importante comprender
las condiciones meteorológicas, los
pronósticos del clima los ciclos de los
fenómenos naturales, etc., para planificar
su sembrío y crianza de animales,
comercializar sus productos, etc. La utilidad
de la estadística la experimentaran en
la medida que puedan relacionarla a las
situaciones que enfrentan y actuar usando
los conocimientos y procesos respectivos.
A lo anterior se suma la necesidad de los
jóvenes y adultos de ejercer a plenitud
su   ciudadanía. Ello implica, por ejemplo,
tener la capacidad de interpretar y evaluar
críticamente la información estadística
que presentan los medios informativos
en noticias, tablas y gráficos; asimismo,
argumentar, discutir y comunicar opiniones
y emitir juicio crítico.
28 Módulo de Orientaciones para el docente del Área de Matemática de EBA
Vivimos en una sociedad caracterizada por el crecimiento acelerado de la información y
desarrollo tecnológico que posibilita el tratamiento de grandes cantidades de información.
En ese sentido, el componente de estadística y probabilidad presentado en el DCBN EBA
involucra la organización, análisis y gestión de datos mediante herramientas eficaces.
Por otro lado, se aborda el tratamiento matemático de situaciones inciertas, el análisis
de datos y gráficos asociados a ellas, la evaluación de riesgos y beneficios, posibilitando
tomar decisiones con fundamento. Además, permite comprender juegos de azar, seguros,
simulación de situaciones y la confiabilidad de los resultados.
El estudiante de EBA debe aprender a:
•	 Recopilar, procesar, interpretar y
valorar los datos.
•	 Analizar las situaciones involucradas.
•	 Desarrollar modelos expresando un
lenguaje estadístico.
•	 Emplear variadas representaciones
para organizar datos.
Unidad 3
Orientaciones para aplicar
estrategias en el área de
matemática
30 Módulo de Orientaciones para el docente del Área de Matemática de EBA
	 Cómo desarrollar competencias matemáticas
En las unidades anteriores se ha sustentado la importancia de la matemática en la vida cotidiana,
en el sistema social productivo, el ambiente, la ciencia, la tecnología, etc. Además, lo significativo
que resulta presentarla y aprenderla como próxima a la realidad y en toda  su funcionalidad.
Hemos analizado la importancia de orientar el planteamiento y la resolución de problemas a
partir de situaciones reales de diversos contextos, despertando actitudes favorables hacia y
con la matemática. A lo largo de éste proceso es fundamental el desarrollo del pensamiento
matemático, mediante la realización de tareas y actividades de progresiva complejidad que
impliquen retos y dificultades cognitivas.
Sin retos no hay aprendizajes, corremos el riesgo de quedarnos solo en la selección de situaciones
y memorización o repetición de rutinas. Se debe incentivar en los estudiantes el razonamiento,
la argumentación, la investigación e indagación, la identificación y generación de estrategias, la
representación y comunicación de resultados; es decir, retarlos constantemente para que actúen
y piensen matemáticamente en diversas situaciones.
Desarrollar competencias matemáticas es un proceso complejo y dinámico, requiere la interacción
de varios factores e involucra procesos cognitivos. El docente debe garantizar este proceso
recurriendo a tareas y actividades matemáticas que generan una interacción dinámica entre
situaciones relacionadas a la vida y la práctica social del estudiante, el desarrollo de procesos
cognitivos y la construcción de los conocimientos matemáticos. Veamos:
Hemos visto que las competencias incluyen conocimientos, habilidades, actitudes y valores; este
conjunto de elementos son los recursos con los que contamos para resolver problemas, solo
cuando los movilizamos y utilizamos en ámbitos específicos evidenciamos nuestras competencias
y el nivel alcanzado. Son, por tanto, un sistema complejo de comprensión y actuación en que se
evidencia un saber y un querer: saber pensar, saber decir y saber hacer; y un querer vinculado
con las emociones, necesidades e intereses de nuestra vida. En el caso de los estudiantes de EBA,
su vida gira en torno a su familia, trabajo y comunidad; si lo que aprenden no lo vinculan a dichos
aspectos simplemente lo olvidarán y, por consiguiente, no aprenderán.
Situaciones
Presentadas en diversos contextos:
personal, social y científico o matemático.
Relacionados a la vida y práctica social de
los estudiantes
Construcción de los
conocimientos
matemáticos
Desarrollo de procesos
cognitivos
31Dirección de Educación Básica Alternativa
Para que los estudiantes de EBA logren competencias matemáticas se propone la construcción y
ejecución de experiencias de aprendizaje globalizadoras y contextualizadas. Dicha construcción
requiere la identificación de situaciones problemáticas en una variedad de contextos relacionados
a la vida de los estudiantes.
Las situaciones de la vida cotidiana son ideales para ser tratadas como situaciones problemáticas,
sin embargo, debemos tener presente que con frecuencia no suministran directamente datos
precisos, por lo que las condiciones e información que evidencian deben ser modificadas para que
su tratamiento y solución no sea laborioso y complicado. Para ello, es necesario que el docente
desarrolle un proceso de indagación que le facilite adquirir la información adecuada y necesaria.
Los estudiantes participan en la construcción de experiencias de aprendizaje
organizados en grupos de trabajo colaborativo.
Los jóvenes y adultos desarrollan sus capacidades mediante experiencias de aprendizaje
articuladas en torno a situaciones de interés y/o relevancia para sus vidas. Expresan, comparten
y analizan sus saberes previos y experiencias de vida, desarrollan habilidades y aprenden unos
de otros. Asumen consciencia de sus debilidades, fortalezas y estilos de aprender. A partir de
ello experimentan cómo superar las dificultades que enfrentan empleando sus potencialidades.
Las experiencias de aprendizaje favorecen la interacción, el apoyo mutuo, la confianza en uno
mismo, el respeto por el otro; en general, habilidades intra e interpersonales. Se potencian los
aportes y expectativas de los estudiantes con niveles distintos de aprendizaje, unos a otros
apalancan sus capacidades y evidencian sus competencias.
Es indispensable que los estudiantes participen en la planificación y ejecución de las Experiencias
de Aprendizaje organizados en Grupos de Inter y Auto aprendizaje (GIA), esta estrategia es
clave para renovar la práctica educativa entre estudiantes y docentes y garantizar un adecuado
encuentro entre la oferta y la demanda educativa y social.
En este proceso el docente es un dinamizador cultural de los procesos educativos, desde y para
la comunidad. Los estudiantes, con sus docentes, conforman comunidades de aprendizaje.
Comprender el sentido de las estrategias de aprendizaje
La selección e implementación de estrategias de aprendizaje tiene sentido en la medida
que responde a la comprensión de cómo aprende el estudiante de EBA, el dominio de los
aprendizajes involucrados y el logro de competencias.
A continuación presentamos orientaciones ejemplificadas para aplicar estrategias en el área
de matemática. Consideramos que el docente de Educación Básica Alternativa las aplicará con
acierto en las sesiones de aprendizaje, asimismo, las recreará y generará otras.
Entendemos las estrategias de aprendizaje como un proceso que integra principios,
pautas, y criterios con los procedimientos y actividades mediante las cuales los
docentes seleccionan, organizan y realizan las experiencias de aprendizaje en una relación
empática con los estudiantes como gestores de sus procesos de aprendizajes.
Las estrategias deben estar orientadas a favorecer que los estudiantes:
•	 Desarrollen competencias matemáticas.
•	 Encuentren sentido y satisfacción en lo que aprenden.
•	 Aumenten las posibilidades de éxito en las evaluaciones.
•	 Atribuyan resultados beneficiosos a sus esfuerzos.
32 Módulo de Orientaciones para el docente del Área de Matemática de EBA
El docente, además de identificar la situación problemática y plantear la experiencia de
aprendizaje, debe tener claridad sobre cómo va a enseñar y cuál es la intención que persigue al
desarrollar la experiencia, es decir, organizar la situación didáctica.
Presentamos como propuesta la Secuencia Didáctica de Brousseau1, a través de un ejemplo para
ser aplicado en el área de matemática con los estudiantes de Primer Grado del Ciclo Inicial de
Educación Básica Alternativa.
	 Analicemos información previa:
Situaciones didácticas de Brousseau
Una situación es didáctica cuando el docente, tiene la intención de enseñar, un
saber matemático dado explícitamente y debe darse en un medio. Sus fases son las
siguientes:
Secuencia didáctica de Brousseau1
1 	 El gráfico y la descripción de las fases han sido adaptadas de “Rutas del Aprendizaje – Versión 2015 – Área Curricular
Matemática – 1° y 2° Gdos de Educación Secundaria.
Acción
Formulación
Validación
Evaluación
Institucio-
nalización
a.
b.
c.
e.
d.
33Dirección de Educación Básica Alternativa
A continuación desarrollamos una experiencia de aprendizaje utilizando la Secuencia Didáctica
de Brousseau. Veamos cada una de las fases ejemplificada:
Experiencia de Aprendizaje para el Ciclo Inicial de EBA
Los estudiantes resolverán la situación problemática: movilizarse para realizar las actividades
cotidianas, a través de la “Secuencia Didáctica de Brousseau”.
Ciclo: Inicial de EBA
Grado: 1ro.
Área: matemática
Componente del Área: Sistemas numéricos y funciones
Competencia, aprendizajes a lograr e indicadores:
COMPETENCIA APRENDIZAJES A LOGRAR INDICADOR
Resuelve problemas rela-
cionados con su entorno
a través de estrategias que
involucran operaciones de
adición y sustracción con
números naturales, demos-
trando confianza en sus
propias capacidades y per-
severancia en la búsqueda
desoluciones.
Expresa e interpreta informa-
ción	 numérica concerniente a
su persona, familia, barrio o co-
munidad, tanto en sus propios
códigos como en lenguaje con-
vencional.
Ubica los números naturales en
la recta numérica.
Efectúa	operaciones sencillas
de adición y sustracción de
números naturales menores que
100.
Expresa de forma oral y escrita
el uso de los números, hasta
100, en contextos de la vida
cotidiana.
Elabora representaciones
de cantidades de hasta 100
objetos de forma gráfica y
simbólica.
Identifica cantidades de hasta
100 objetos expresándolos en
un modelo de solución aditiva.
Explica sus procedimientos o
resultados de forma breve.
Situación problemática: movilizarse para realizar las actividades
cotidianas
La mayoría de los estudiantes de EBA se movilizan utilizando variados medios de transporte.
Identifican, seleccionan y recorren diferentes rutas para ir a trabajar, visitar un familiar o cumplir
con un trámite o actividad pendiente.
Para realizar este proceso ejecutan acciones que requieren utilizar la matemática: identificación
de números, estimación de presupuesto, localización, gestión de datos, optimización del tiempo,
selección de rutas, etc. A lo largo de este proceso desarrollan su pensamiento matemático y por
ende competencias matemáticas.
	 Saberes previos
Antes de presentar una situación los estudiantes deben recuperar sus saberes previos. Las
preguntasy comentariosdebencentrarseenla expresión,porparte del estudiante, de información
cuantitativa:
34 Módulo de Orientaciones para el docente del Área de Matemática de EBA
¿A qué hora empiezan a movilizarse? ¿Qué rutas siguen? ¿Cuánto tiempo demoran en cada ruta?
¿Cuánto gastan?
	 Experiencia de Aprendizaje:
El docente dinamiza un proceso de diálogo para que los estudiantes exploren sus saberes previos:
El docente presenta una situación, en texto escrito e ilustrada, referida a la situación problemática
a abordar, con información cuantitativa resaltada y datos modificados para su tratamiento y
solución. Lee el texto y lo coloca en un lugar visible del aula, resalta la información cuantitativa y
la repite para asegurarse que el estudiante la entendió. Veamos:
¿Hacia qué lugares se movilizan? ¿Qué rutas siguen?
¿Cuánto tiempo se demoran? ¡Cuánto gastan?
Ana vive en San Juan de Lurigancho. Hoy irá a Villa El Salvador a visitar a su hija. Se movilizará
en moto, combi y tren eléctrico.
Faltan 5 minutos para las 6 de la mañana. Ana camina durante 3 minutos de su casa al
paradero de la moto.
Luego de 2 minutos Ana llega en moto al paradero de la combi. De allí, recorre 20 minutos
en combi hasta el paradero del tren.
Luego de 40 minutos en tren, Ana llega a Villa El Salvador.
Decido la ruta para llegar a mi destino
En mi pueblo uso acémila. Aquí
utilizo moto, combi y tren.
35Dirección de Educación Básica Alternativa
a.	 Fase de Acción
Involucra aspectos cognitivos y cuestiones de índole práctica, ambos dirigidos a la solución
de problemas que es preciso resolver en condiciones específicas.
Acciones del docente Acciones del estudiante
Expone la situación y las consignas, y se
asegura de que han sido bien comprendidas.
Inicia a partir de los conocimientos previos
del estudiante, para ello puede diseñar
actividades.
Leen el problema, analizan los factores
que la definen como tal, se identifican
con los datos, el propósito, la factibilidad
de su resolución y solución.
Interviene como mediador. Se abstiene de
brindar información que condicione la acción
de los estudiantes.
Se imaginan la situación apelando a sus
saberes previos.
Aclara consignas, promueve la aparición de
muchas ideas y señala contradicciones en los
procedimientos.
Movilizan aspectos cognitivos y
cuestiones de índole práctica, dirigidas a
la solución del problema.
Ejemplo:
El docente lee con apropiada entonación la situación problemática a los estudiantes,
resaltando la información numérica. Luego, los invita a comentar la situación, graficarla,
hacer hipótesis, pensar estrategias para solucionar el problema y tentar respuestas.
Algunas preguntas podrían ser:
	¿Cómo imaginas la situación?
	¿Cómo podemos representar el problema?
	¿Será útil elaborar un reloj de manecillas? ¿Por qué?
Ana camina 3 minutos de su
casa al paradero de la moto.
La moto llega al paradero
de la combi en 2 minutos.
5 minutos para las 6
36 Módulo de Orientaciones para el docente del Área de Matemática de EBA
Una vez elaborado el reloj de manecillas, el docente invita a los estudiantes a girar las
agujas del reloj para responder a las siguientes preguntas:
•		 ¿Qué hora es cuando Ana llega al paradero de la combi?
•		 ¿Cuánto demoró Ana del paradero de la combi al paradero del tren?
•		 ¿Qué hora es cuando Ana llega al paradero del tren?
•		 ¿Cuánto demoró Ana en llegar del paradero del tren a Villa El Salvador?
•		 ¿Qué hora es cuando Ana llega a Villa El Salvador?
•		 ¿Cuánto demoró Ana en total?
b.	 Fase de formulación
Se busca la adquisición de destrezas para la utilización de los lenguajes más apropiados, y
se mejora progresivamente la claridad, el orden y la precisión de los mensajes.
Acciones del docente Acciones del estudiante
Organizar a los estudiantes de modo
que puedan dividirse tareas, diseñar y
materializar la solución, seleccionar los
materiales, las herramientas, etc.
Obtiene el plan ordenado, procedimientos,
estrategias, recursos y el producto que
resuelve los problemas.
Indicar las pautas para que los estudiantes
utilicen los medios de representación
apropiados.
Explica los conocimientos en un lenguaje
que los demás puedan entender. Utiliza
representaciones convencionales para
comunicar.
Sondear el estado de los saberes previos y
los aspectos afectivos y actitudinales.
Pone énfasis en el manejo de lenguajes
muy variados, ya sea de tipo verbal, escrito,
gráfico, plástico, informático o matemático.
Detectar procedimientos inadecuados,
prejuicios, obstáculos y dificultades, para
trabajarlas con los estudiantes, según
convenga a su estrategia.
Ejemplo:
El docente observa los procedimientos que siguen sus estudiantes y los orienta.
Los estudiantes planifican y resuelven el problema, representan y comunican sus
resultados:
	 •	 ¿Cuánto crees que demoró Ana en llegar al
paradero de la combi?
	 •	 ¿Cómo sería la representación del problema en
una recta numérica?
	 •	 ¿Cómo representarías la suma?
	 •	 	¿Qué otras sumas debes realizar?
	 •	 	¿Cuál es el resultado final?
	 •	 	¿Has seguido algún orden para resolver el problema?
Ana llegó al paradero de
la combi en 5 minutos:
3 + 2 = 5 3 +
2
5
37Dirección de Educación Básica Alternativa
c.	 Fase de validación
Esunafasedebalanceyrepresentaciónderesultados,yde confrontacióndeprocedimientos
Acciones del docente Acciones del estudiante
El docente estimula y coordina las pruebas,
los ensayos, las exposiciones, los debates y
las justificaciones.
Los estudiantes verifican sus productos,
representaciones y resultados como parte
de las situaciones mismas sin tener que
recurrir al dictamen del docente.Absuelve las dudas y contradicciones
que aparezcan, señala procedimientos
diferentes, lenguajes inapropiados, y
busca que el consenso valide los saberes
utilizados.
En ese momento crece el valor de las
intervenciones del docente, que debe
recurrir a las explicaciones teóricas y
metodológicas necesarias, de acuerdo con
las dificultades surgidas.
Las producciones de las situaciones son
sometidas a ensayos y pruebas por sus
pares en un proceso metacognitivo que se
completa en la fase siguiente.
Esta es una buena oportunidad para tomar
datos evaluativos y para introducir nuevas
variantes de problematización. Confrontan sus procedimientos.
Coordina y resume las conclusiones que
son clave para la sistematización de la
próxima fase.
Ejemplo:
El docente interviene explicando el significado de la decena y el procedimiento de la
suma llevando, de tal manera que los estudiantes puedan aplicarlo a la solución del
problema.
A lo largo de su intervención, el docente utilizará números distintos a aquellos que
están involucrados en el problema. De esta manera, el estudiante podrá hacer la
transferencia de lo reforzado a la situación específica que debe resolver.
Veamos algunos ejemplos del refuerzo
D U
2 0
38 Módulo de Orientaciones para el docente del Área de Matemática de EBA
d.	 Fase de Institucionalización
Enestafasesegeneralizayseabstraelosconocimientosenbaseaprocedimientos realizados
y resultados obtenidos.
Acciones del docente Acciones del estudiante
Cumple un rol como mediador de
códigos de comunicación.
Elestudiantedescontextualizaydespersonaliza
el saber para ganar el estatus cultural y social
del objeto tecnológico autónomo, capaz de
hacerlo funcionar como herramienta eficaz en
otras situaciones.
Explica, sintetiza, resume y rescata los
conocimientos puestos en juego para
resolver la situación planteada.
Avanza en los niveles de abstracción
correspondientes, formalizando conceptos y
procedimientos matemáticos, contribuyendo
a re significar el aprendizaje en el contexto
global, explicando y redondeando el lenguaje
matemático apropiado.
Destaca la funcionalidad.
Rescata el valor de las nociones y los
métodos utilizados. Señala su alcance,
su generalidad y su importancia.
El estudiante traduce la situación, interpreta,
realiza representaciones simbólicas, discute
sus supuestos en su equipo, se comunica,
socializa sus resultados, encuentra el error en
el compañero, refuta y generaliza superando
los errores y el modelo intuitivo instalado.
Formaliza conceptos y procedimientos
matemáticos, contribuyendo a
resignificar el aprendizaje en el contexto
global del estudiante.
3 decenas unidades
La suma llevando
Para sumar se colocan
las unidades debajo
de las unidades y las
Decenas debajo de las
Decenas y se suma.
D U
4 8
2 3
7 1
1
1
39Dirección de Educación Básica Alternativa
Ejemplo:
Los estudiantes comparten sus representaciones, analizan los resultados de otros
compañeros. En este proceso explican y discuten sus supuestos, estrategias y
resultados. Veamos algunos ejemplos:
e.	 Fase de Evaluación
Se plantea una situación nueva articulada a los temas tratados.
Se realiza la autoevaluación del estudiante y la coevaluación entre pares, entendidas como
espacios de aprendizaje: aprendizaje y evaluación como proceso recursivo.
Acciones del docente Acciones del estudiante
El docente evalúa el desempeño del
estudiante a través del seguimiento de sus
actuaciones y productos obtenidos, desde
la aparición de los primeros borradores
hasta el producto final.
El estudiante realiza la autoevaluación y la
coevaluación entre pares como instancias
de aprendizaje: aprendizaje y evaluación
como proceso recursivo.
Puede solicitar trabajos adicionales con el
propósito de obtener más datos evaluativos
y permitir la transferencia y la nivelación.
Anticipa una nueva secuencia articulada
con los temas y/o contenidos tratados.
Ejemplo:
	 a.	El docente, en base a la situación anterior, brinda información adicional a los
estudiantes. Dicha información genera un mayor nivel de dificultad para la
resolución del problema.
	 	 Un día, Ana llegó con dos minutos de retraso al paradero de la moto. Ello ocasionó
que su viaje en combi demorara 8 minutos más de lo habitual y su viaje en tren
Formaliza conceptos y explica:
Estudiante: inicio en cero la recta numérica de los números naturales porque ello me
permite graficar adecuadamente el espacio que corresponde al número.
Encuentra los	errores, refuta y generaliza:
Estudiante: al graficar la suma de dos números naturales debes partir de cero”.
0 3 61 4 7 92 5 8 10
0
4
+2
3 61 42 5
40 Módulo de Orientaciones para el docente del Área de Matemática de EBA
10 minutos más de lo acostumbrado. ¿Cuánto más tardó Ana en llegar a Villa El
Salvador?
	 b.	 Los estudiantes utilizan el reloj de manecillas, que han elaborado, para comunicar:
	 	 •	 ¿Cuántos minutos demoran en recorrer una de sus rutas habituales?
	 	 •	 ¿Quién demora más? ¿Quién demora menos? ¿Cuál es la diferencia de tiempo
entre ambos?
Las fases mencionadas pueden ser utilizadas en el desarrollo de las diferentes competencias
matemáticas de cualquiera de los tres ciclos de Educación Básica Alternativa. A lo largo de
ellas observamos que el estudiante traduce la situación, interpreta, realiza representaciones
simbólicas, discute sus supuestos, comunica sus hallazgos y conclusiones, socializa sus
resultados, identifica errores propios y de sus compañeros, refuta y generaliza superando
los errores y el modelo intuitivo instalado para dar lugar a un nuevo modelo mental vía
el conflicto cognitivo. Este proceso sucede al surgir discrepancia entre la imagen mental
formada anteriormente y la solicitada, los estudiantes utilizan sus habilidades y construyen
conocimiento.
El docente asesorará al estudiante para que amplíe y/o identifique nuevas situaciones a
partir del contexto real del estudiante, asimismo, plantee y resuelva problemas.
Suben 2,
espera...
Lleva, lleva,
hace 6 minutos
pasó el otro.
Déjame 3
cuadras después
del parque.
Envía un mensaje,
avisa que llegamos
en 5 minutos.
41Dirección de Educación Básica Alternativa
Tal como hemos visto anteriormente, el docente, además de identificar la situación problemática
y plantear la experiencia de aprendizaje, debe tener claridad sobre cómo va a enseñar y cuál es
la intención que persigue al desarrollar la experiencia de aprendizaje.
Presentamos como propuesta la realización de la investigación en matemática, a través de un
ejemplo, planificado para los estudiantes de Tercer Grado del Ciclo Intermedio de EBA.
	 Analicemos información previa:
El ciclo de la investigación se inicia motivando al estudiante a hacer preguntas sobre sí mismo,
su entorno familiar, local u otro más amplio. Luego, elaboran un plan, recolectan datos por su
propia cuenta o hacen uso de datos ya existentes en distintas fuentes.
En grupo, los estudiantes, analizan los datos recolectados, construyen tablas, gráficos, buscan
patrones, hacen inferencias, predicciones para sacar conclusiones; interpretan, comunican y
generan nuevas preguntas.
	 Fases de la Investigación
La investigación2
Planteamiento
del problema
Desarrollo
del plan
Recolección
y manejo de
datos
Fase de
conclusiones
Análisis de
datos
a.
b.
c.
e.
d.
42 Módulo de Orientaciones para el docente del Área de Matemática de EBA
A continuación desarrollamos una experiencia de aprendizaje utilizandola investigación. Veamos
la secuencia ejemplificada para cada fase:
Experiencia de Aprendizaje para el Ciclo Intermedio de EBA
Losestudiantesresolveránlasituaciónproblemática:identificar ycomunicarinformación
estadística referida a la importancia de conocer el clima a través de la “Investigación”.
Ciclo: Intermedio de EBA
Grado: 3ro
Área: Matemática,
Área: CAS
Componente del Área: Estadística y probabilidad
Competencia, aprendizajes a lograr e indicadores:
COMPETENCIA
APRENDIZAJES A
LOGRAR
INDICADOR
Recolecta y organiza
datos, construye e in-
terpreta gráficos esta-
dísticos referentes a si-
tuaciones y fenómenos
de su entorno (natural,
económico, social) va-
lorando la importancia
del lenguaje gráfico en
la vida cotidiana.
Elabora gráficos de barras
con datos referidos a
situaciones cotidianas y
comunica el proceso	
que utiliza.
Interpreta diagramas,
esquemas, tablas, gráficos
de barras y pictogramas.
Emplea procedimientos de recolección
de datos: preguntas orales y escritas,
encuestas, registro de hechos.
Plantea relaciones entre los datos
(cualitativos y cuantitativos) en
situaciones de contexto personal,
expresándolos en tablas simples de
conteo, barras simples o pictogramas
(con escala dada)
Responde a preguntas sobre
información de tablas, pictogramas,
gráficos de barras simples, con datos
cuantitativos y cualitativos
Expresa sus conclusiones respecto a la
información obtenida.
Situación problemática: emplear y comunicar información estadística
referida al clima
Los estudiantes de EBA necesitan comprender y utilizar la información, de carácter estadístico,
que se presenta en distintos medios de difusión: periódicos, revistas, noticieros, encartes, etc. Éstas
sonoportunidades para movilizar yponer en funcionamiento sus conocimientos, estrategias y re-
cursos; por ende, desarrollar su pensamiento matemático.
La información que se presenta en los medios es muy variada, por lo que es importante que
el docente seleccione aquella que es relevante para el estudiante joven y adulto; por ejemplo,
aquella que está asociada a sus demandas de carácter laboral, ciudadano, ambiental, social,
económico, cuidado de la salud, etc.
43Dirección de Educación Básica Alternativa
En este caso elegimos información referida al clima, dado que por lo general el estudiante de
EBA está atento a conocer cómo evoluciona; tanto para vestirse apropiadamente como para
cuidar su salud y la de su familia o anticipar las condiciones climáticas favorables a su trabajo o
quehacer económico: venta de alimentos, cosecha, siembra, reproducción de animales, etc.
La comprensión y utilización de información estadística y cuantitativa referida al clima, reta al
estudiante en relación a utilizar apropiadamente la matemática: identificación y elaboración de
gráficos estadísticos, gestión de datos, comunicación matemática, etc. A lo largo de este proceso
desarrolla competencias matemáticas.
	 Saberes previos
El docente inicia la experiencia de aprendizaje, a través de la investigación, dinamizando un
proceso de diálogo orientado a que los estudiantes exploren sus saberes previos:
Las preguntas y comentarios deben centrarse en la expresión, por parte del estudiante, de
información cuantitativa, asimismo, comunicarla a terceros:
•	 ¿Cómo está el clima hoy? ¿Está más o menos caluroso que ayer? ¿Es más frio que el mes
pasado?
•	 ¿Es importante para ustedes conocer con anticipación cómo estará el clima? ¿Por qué?
•	 ¿A qué personas les puede interesar conocer cómo evolucionará el clima? ¿En qué les
beneficia ello?
	 Secuencia de la Investigación
a.	 Planteamiento del problema
Veamos con ejemplo cada una de las fases de la investigación.
El docente presenta una situación o problema a los estudiantes, ellas y ellos se organizan
en grupos para expresar su comprensión
b.	 Desarrollo del plan
El objetivo de esta fase es que los estudiantes conozcan el tema de estudio que van a
abordar, asimismo que planteen alternativas y opten por una. En este caso asumiremos
que los estudiantes han consensuado en que una forma de verificar si es importante para
los jóvenes y adultos conocer con anticipación las condiciones climáticas y tendencias de
Consideramos que las personas jóvenes y adultas, debido a las ocupaciones
laborales y familiares que tienen necesitan conocer con anticipación el estado del
clima y cómo evoluciona, no solo de su localidad sino en otros lugares. Por ejemplo,
para adquirir ropa de acuerdo a la estación y venderla en un lugar específico, para
comercializar alimentos apropiados a la estación, sembrar, cosechar, criar animales,
cuidar su salud y la de su familia o simplemente vestirse apropiadamente.
Asumimos que es importante para los jóvenes y adultos conocer con anticipación
las condiciones climáticas y las tendencias de su evolución ya que ello contribuye en
su quehacer económico, laboral y familiar ¿Cómo verificamos esta hipótesis?
44 Módulo de Orientaciones para el docente del Área de Matemática de EBA
su evolución es preguntándoles directamente. Para ello deciden entrevistarlos a un grupo
de ellos en sus domicilios, un día domingo.
Los estudiantes imaginan las entrevistas y analizan lo que puede suceder. Deciden que
es conveniente realizar la entrevista a través de un cuestionario que elaborarán con
anticipación y aplicarán a una muestra de jóvenes y adultos de 15 a más años.
En esta fase es importante que los estudiantes decidan e identifiquen la muestra y posibles
variables, también es parte de esta fase el diseño de un instrumento para el recojo de
información, en este caso un cuestionario que aplicaran en el momento de realizar la
entrevista a los jóvenes y adultos de 15 a más años.
	 Los estudiantes:
	 •	 Forman equipos de 4 a 5 estudiantes.
	 •	 Seleccionan el trabajo a investigar y se documentan sobre el tema de estudio.
	 •	 Diseñan una encuesta sencilla (4 a 6 preguntas) para recoger la información que
necesitan. Dos datos útiles a considerar son la edad y el sexo.
	 •	 Cada equipo recoge los datos a través de una encuesta.
	 •	 En cada pregunta los estudiantes deben reconocer la variable que se está analizando
y su tipo.
	 •	 Contrastan las tabas elaboradas, que deben ser iguales para todos, y corrigen los
errores.
Instrucciones: estimado vecino, esta encuesta nos ayudará a conocer sobre la importancia
que tiene para usted conocer con anticipación información sobre el clima.
Señalar con ( X )
1.	Edad:
De 15 a 24   (	)	 De 25 a 34  (	)
De 35 a 44   (	)	 De 45 a 54  (	)
De 55 a 64   (	)	 De 65 a  74 (	)	 De 75 a más (	)
2.	Sexo:
Mujer (	)	Varón (	)
3.	 ¿Le interesa conocer información sobre el clima?
Sí (	)	No (	)
4.	 ¿Por qué le interesa conocer información sobre el clima?
a.	 Para utilizarla en mi trabajo.	 ( )
b.	 Para cuidar mi salud y la de mi familia. ( )
c.	 Para vestirme apropiadamente. ( )
d. Otras (indicar cuál) …………………………………….
5. A través de qué medios te informas sobre el clima
      Diario (	)	Radio (	 )	 TV (	 )
Otro (indicar cuál) ………………………………………………………
45Dirección de Educación Básica Alternativa
c.	 Recolección y manejo de datos
Los estudiantes se organizan antes de realizar la encuesta y toman decisiones respecto a
acciones que deben realizar antes, durante y después de la encuesta; por ejemplo:
	 •	 ¿Cómo nos vamos a organizar para realizar la encuesta?
	 •	 ¿Cómo nos vamos a organizar para procesar la data?
	 •	 ¿Quiénes integrarán los equipos de encuestadores? ¿tendrán alguna identificación y
distintivo?
	 •	 ¿Cuál será el ámbito de la encuesta? ¿a cargo de qué ámbito estará cada equipo?
	 •	 ¿Cuál es exactamente la población a encuestar? En este caso son personas de 15 a más
años ¿habrá una edad límite?
	 •	 ¿Qué día realizaremos la encuesta? ¿De qué hora a qué hora se realizará?
Durante este proceso los estudiantes deben reconocer con claridad la población, la
muestra y las variables.
d.	 Análisis de datos
El docente debe monitorear y orientar esta fase asegurando el cumplimiento adecuado de
las acciones a realizarse, por ejemplo:
	 •	 La distribución equitativa de las encuestas en cada equipo.
	 •	 El correcto llenado de las tablas en base a las encuestas asignadas, por parte de cada
integrante de los equipos.
	 •	 La unificación en una sola tabla de la información procesada por cada integrante del
equipo. Esta acción puede estar a cargo del coordinador del equipo o un representante
que designen.
	 •	 La elaboración de las conclusiones en cada uno de los equipos.
	 •	 La presentación y socialización de las conclusiones por parte de cada equipo. En este
punto es necesario que los estudiantes analicen modelos de cómo se presenta la
información.
e.	 Fases de conclusiones
En esta fase los estudiantes:
	 •	 Desarrollan habilidades orientadas al desarrollo de competencias matemáticas. En este
caso, habilidades de analizar datos, extraer conclusiones, interpretar un dato en su
contexto, plantear afirmaciones, etc.
	 •	 Argumentan su opinión en función a los datos obtenidos a lo largo del proceso
vivenciado.
	 Actividades de extensión
Luego de realizar la investigación los estudiantes analizan otras situaciones problemática, en
texto escrito, cuadros y gráficos estadísticos. Se recomienda que en un inicio estén referidos al
clima y luego se amplíen hacia otras situaciones.
46 Módulo de Orientaciones para el docente del Área de Matemática de EBA
Es importante recordar que la información cuantitativa que se presente debe estar resaltada y,
cuando corresponda, los datos modificados para garantizar un tratamiento y solución adecuada
al nivel de complejidad que retará a los estudiantes.
El docente presenta la situación, lee el texto y lo coloca en un lugar visible del aula. Resalta la
información cuantitativa y la repite para asegurarse que el estudiante la entendió. Veamos un
ejemplo:
Docente:
Luego de que los estudiantes dialogan en base a las preguntas presentadas u otras similares, el
docente presenta información gráfica sobre el clima. Veamos:
Docente:
En un diario de circulación nacional se ha presentado la siguiente información referida al clima:
TEMPERATURA y RADIACIÓN SOLAR
LIMA, CUSCO, ICA, HUANCAYO, TACNA, AREQUIPA, MOQUEGUA y PIURA
Luego el docente dinamiza la realización de actividades en base a la información presentada.
Veamos:
En una escala de 1 a 10, considerando que 1 es frio y 10 es caluroso ¿En qué número
ubicarías el clima de hoy?
Al utilizar la escala de 1 a 10 ¿Todos tenemos la misma percepción respecto  a la medición
del clima?
¿Hay alguna convención o acuerdo establecido para medir el clima?
¿Dónde encontramos información sobre el clima?
¿Cómo comunicarías gráficamente el clima de diferentes regiones de nuestro país?
47Dirección de Educación Básica Alternativa
Actividades
1.	 Analizar el gráfico y compartir apreciaciones
Los estudiantes analizan el grafico y comparten sus interpretaciones. El docente debe dejar que
este proceso fluya y surjan correcciones entre los estudiantes, cuidando el diálogo asertivo y
el compartir afirmaciones con sustento. Finalmente, de no lograrse claridad, el docente cierra
esta parte reforzando la interpretación correcta del cuadro de Temperatura y Radicación Solar.
Veamos:
“Para cada ciudad, los valores numéricos que aparecen debajo del cuadro de la izquierda indican,
respectivamente, el valor mínimo y máximo de la temperatura. El cuadro de la derecha indica el
nivel de radiación solar”.
2.	 Elaborar cuadros para presentar parte de la información del cuadro
3.	 Presentar la información del cuadro utilizando un Gráfico de Barras
Temperaturas y radiación solar
Ciudad
Temperatura Radiación
solar+ Baja + Alta Diferencia
Lima 20° 28° 8° Muy alto
Cusco 9° 18° 9° Alto
4
8
12
16
20
24
28
temperatura
Variación de
temperaturas
Ciudades
Lima Cusco
48 Módulo de Orientaciones para el docente del Área de Matemática de EBA
Dadalaimportanciaypertinenciadelaresolucióndeproblemasenelaprendizajedelamatemática
es fundamental que los docentes preparemos a los estudiantes para enfrentar sistemáticamente
la solución de problemas y ser conscientes del proceso seguido.
Presentamos como propuesta la Resolución de Problemas, a través de un ejemplo para ser
aplicado en el área de matemática con los estudiantes de Primer Grado del Ciclo Avanzado de
Educación Básica Alternativa.
	 Analicemos información previa
Veamos cada uno de los 4 pasos
1.	 FAMILIARIZACIÓN: comprensión del problema
En esta fase el estudiante debe lograr comprender el problema. Se recomienda:
	 •	 Asegurar la lectura atenta del problema por parte del estudiante. Dar espacio para que
lo lea con tranquilidad.
	 •	 Que el estudiante exprese el problema con sus propias palabras, éste discurso oral
no necesariamente guardará el rigor de la formalidad exigida pero si evidenciará el
entendimiento de los elementos involucrados en el problema y lo que se pretende resolver.
	 •	 Que el estudiante explique a otro compañero de qué trata el problema, utilizando sus
propias palabras.
	 •	 Respetar el ritmo de aprendizaje del estudiante, sin presiones, ni apresuramientos, que
juegue con la situación, que pierda el miedo inicial.
Veamos algunas preguntas que el docente puede hacer a los estudiantes para facilitar la  
comprensión del problema:
	 •	 ¿De qué trata el problema?
	 •	 ¿Has visto alguna situación parecida?
	 •	 ¿Qué es lo que piden? ¿Cuál es la incógnita?
	 •	 ¿Cuáles son las condiciones? ¿La condición es suficiente para determinar la incógnita?
¿Es contradictoria? ¿Es redundante?
2.	 BÚSQUEDA DE ESTRATEGIAS: diseño o adaptación de una estrategia
Durante esta fase el estudiante explora la situación. En ese momento necesita poner en
valor una serie de estrategias heurísticas que le puedan ser útiles, además, saber elegir la
más adecuada, dependiendo de la estructura del problema.
Resolución de problemas3
George Polya (1887 – 1985). Matemático húngaro, fue uno de los primeros
investigadores que se dedicó a trabajar sistemáticamente la resolución de problemas.
En el año 1945 publicó el libro: Cómo plantear y resolver problemas (How to Solve It), en el
libro presenta los llamados 4 pasos de Polya para resolver problemas.
49Dirección de Educación Básica Alternativa
Esta fase es una de las más importantes en el proceso de solución de problemas pues
depende mucho de la base de conocimientos así como de la calidad del pensamiento
matemático. Algunas preguntas que el docente puede hacer en esta fase son:
	 •	 ¿Te has encontrado con un problema semejante? ¿O has visto el mismo problema
planteado de forma ligeramente diferente?
	 •	 ¿Conoces un problema relacionado con este?
	 •	 ¿Conoces alguna propiedad que te pueda ser útil?
	 •	 ¿Has resuelto antes un problema similar? ¿Se puede aplicar el método que empleaste
para solucionarlo al problema actual? ¿Puedes usar su resultado?
	 •	 ¿Puedes enunciar el problema de otra manera? ¿Puedes plantearlo de forma distinta?
En ese caso: ¿cambia la terminología?, ¿hay nuevas definiciones?
	 Algunas sugerencias pueden ser:
	 •	 Hazte un esquema, una figura o un diagrama para representar el problema.
	 •	 Supón el problema resuelto: ¿cuál sería el resultado? ¿cómo crees que lo solucionaron?
	 •	 Si no puedes resolver el problema propuesto, trata de resolver antes uno similar.
	 •	 Date ejemplos de la situación. Experimenta. Particulariza, recuerda que empezar por lo
fácil hace fácil lo difícil.
	 •	 Imagínate un problema análogo pero más sencillo.
	 •	 Resuelve una parte del problema.
	 •	 Considera sólo una parte de la condición, descarta la otra parte.
	 •	 Empieza al revés, usa el razonamiento regresivo.
	 •	 ¿Has empleado todos los datos? ¿Has empleado toda la condición?
3.	 EJECUCIÓN DEL PLAN: Ejecución de la estrategia
Una vez comprendido el problema, lo que se pretende lograr y decidido el camino a seguir
para su solución, se procede a ejecutar la estrategia de solución. Durante este proceso
entran a tallar los mecanismos de regulación mental y la habilidad para salir de bloqueos.
Es recomendable que el estudiante:
	 •	 Ejecute su plan de solución.
	 •	 Compruebe cada uno de los pasos: el problema ha sido resuelto ¿estás seguro? ¿Cómo
lo compruebas?
	 •	 Actúe con flexibilidad, es decir, cambie de estrategia si las cosas se complican
demasiado.
	 •	 Aprenda a ser perseverante y variado, es decir, por una parte no se debe abandonar
un aspecto examinado antes de que nos haya sugerido algo útil, por otro es necesario
examinar tantos aspectos como sea posible, intenta ver siempre algo nuevo.
50 Módulo de Orientaciones para el docente del Área de Matemática de EBA
4.	 VISIÓN RETROSPECTIVA
El estudiante debe aprender a mejorar sus habilidades para enfrentarse con problemas.
Los psicólogos e investigadores señalan a esta fase como la principal para que la persona
adquiera el conocimiento de sus procesos mentales así como sus preferencias y emociones
a lo largo del proceso de solución.
La solución de un problema involucra emociones diversas, estos sentimientos pueden
impulsar o bloquear a la persona.
Durante la fase de familiarización con el problema el estudiante suele experimentar una
tensión natural ante la búsqueda de un plan de resolución, tensión que puede desembocar
en interés o ansiedad. Cuando se produce la inspiración se tienen sentimientos positivos
que cobran más o menos intensidad según las expectativas que se tengan sobre el éxito
de dicho plan.
El estudiante debe examinar a fondo el proceso seguido y preguntarse:
•	 ¿En qué momento me quedé bloqueado?
•	 ¿Cómo logré salir del bloqueo?
•	 ¿Cómo llegué a la solución?
•	 ¿Puedo verificar cada paso seguido?
•	 ¿Por qué este camino me llevó a la solución?
•	 ¿Qué pista me ayudó a decidir la estrategia a usar?: un dato, algún problema similar, algún
modelo.
Comprensión del problema
Diseño o adaptación de una
estrategia
Ejecución de una estrategia
Visión retrospectiva
SÍ
NO
51Dirección de Educación Básica Alternativa
Es importante que el estudiante aprenda a:
•	 Distinguir entre el problema en sí y la verificación de los procesos generales de su solución.
•	 Reflexionar sobre sus emociones y estrategias de pensamiento.
•	 Generar experiencia para el futuro: cada vez que resuelve un problema está desarrollando
habilidades de solución y de trabajo con la matemática.
Durante el proceso de resolución de problemas los docentes debemos observar a los estudiantes
para:
•	 Darnos cuenta de sus errores.
•	 Identificar cómo convertir los errores en oportunidades para aprender.
•	 Identificar las estrategias que emplean y cómo actúan los estudiantes al resolver problemas:
¿son impulsivos? ¿se bloquean?
A continuación desarrollamos una experiencia de aprendizaje utilizando la Resolución de
Problemas, según lo planteado por Polya. Para ello presentamos algunos problemas que se
plantean a los estudiantes, previo a ello el docente debe haber identificado situaciones
problemáticas relacionadas y explorado los saberes previos, tal como lo hemos hecho en los
ejemplos para el ciclo inicial e intermedio (puntos 3.1 y 3.2)
Experiencia de Aprendizaje para el Ciclo Avanzado de EBA
Los estudiantes resolverán la situación problemática: a través de la “Resolución de Problemas”.
Ciclo: Avanzado de EBA
Grado: 1ro
Área: matemática
Componente del Área: Geometría y medida
Competencia, aprendizajes a lograr e indicadores:
COMPETENCIA
APRENDIZAJES A
LOGRAR
INDICADOR
Elabora estrategias y técnicas
para medir o estimar el
valor de una magnitud
correspondiente a un objeto
o fenómeno de su entorno
inmediato, con unidades
de longitud, superficie,
volumen, masa, tiempo
o unidades angulares,
mostrando curiosidad,
interés y seguridad al realizar
su trabajo.
Interpreta, identifi-
ca y relaciona uni-
dades de longitud,	
masa, superficie,
tiempo y volumen
en el contexto de la
vida diaria.
Interpreta datos y relaciones no
explicitas respecto a la localización de
lugares o desplazamientos de objetos,
expresándolos en un croquis en el primer
cuadrante del plano cartesiano.
Emplea el plano cartesiano al resolver
problemas de localización.
Aplica las propiedades de las figuras
bidimensionales (círculo, circunferencia)
al plantear o resolver problemas.
52 Módulo de Orientaciones para el docente del Área de Matemática de EBA
	 Situación problemática:
Seguimos los 4 pasos de Polya:
1.	 FAMILIARIZACIÓN: comprensión del problema
Los estudiantes observan el gráfico y se hacen preguntas conducentes a la comprensión
plena del problema. Veamos algunos ejemplos de preguntas:
•		 ¿De qué trata el problema?
•		 ¿Has visto alguna situación parecida?
•		 ¿Qué es lo que piden?
•		 ¿Cuál es la incógnita?
•		 ¿Hay suficiente información?
•		 ¿Cuáles son los datos?
•		 ¿Todos los datos son necesarios para resolver el problema?
•		 ¿Qué datos son necesarios?
•		 ¿Qué datos son innecesarios?
Problema 1
¿Cuánto tiempo demorará, aproximadamente, una persona en recorrer las 10 primeras
cuadras de la Marcha Juvenil?
Recorrido de una Marcha Juvenil
53Dirección de Educación Básica Alternativa
Los estudiantes deben lograr comprender el problema e identificar la data relevante que
lo caracteriza.
2.	 BÚSQUEDA DE ESTRATEGIAS: diseño o adaptación de una estrategia
Los estudiantes exploran la situación. Analizan posibles estrategias y las ponen en valor,
este proceso les permite aprender a elegir la más adecuada. Algunas preguntas que
orienten este proceso podrían ser:
	 •	 ¿Te has encontrado con un problema semejante?
	 •	 ¿Conoces alguna propiedad que te pueda ser útil? ¿Quizá una forma de simplificar el
problema?
Los estudiantes deben lograr proponer o adaptar una estrategia, por ejemplo:
Suponemos que la Marcha Juvenil avanza a ritmo constante y las cuadras tienen la misma
longitud. Esto nos permitirá estimar el tiempo según el número de cuadras avanzadas.
3.	 EJECUCIÓN DEL PLAN: Ejecución de la estrategia
Los estudiantes resuelven:
Observan en el gráfico que de la cuadra 33 de la Avenida Brasil a la cuadra 6 la marcha se
tarda 2 horas 50 minutos. Es decir:
27 cuadras	 	 2 horas 50 minutos = 60’  +  60’  + 50’ = 170’
1 cuadra	 	 170’ / 27 = 6’ aproximadamente
10 cuadras	 	 60’  = 1 hora aproximadamente
4.	 VISIÓN RETROSPECTIVA
Los estudiantes se preguntan
¿Comprendí la solución?
¿Hay otras formas de resolverlo?
Los estudiantes pueden llegar a conclusiones como las siguientes:
Otras formas de resolver el problema es:
•		 Mentalmente
•		 Midiendo los espacios en el gráfico
Problema 2
¿Cómo representaría en un Diagrama Cartesiano la relación entre el espacio y el tiempo
basándose en los datos de la Marcha Juvenil?
54 Módulo de Orientaciones para el docente del Área de Matemática de EBA
Veamos, de modo abreviado, la resolución del problema siguiendo los 4 pasos de Polya:
FAMILIARIZACIÓN: comprensión del problema
¿Puedes plantear el problema con tus propias palabras?
BÚSQUEDA DE ESTRATEGIAS: diseño o adaptación de una estrategia
Los estudiantes, orientados por el docente, averiguan cómo es un Diagrama Cartesiano y
resuelven un problema similar.
EJECUCIÓN DEL PLAN: Ejecución de la estrategia
Los estudiantes, en base al ejemplo trabajado, hacen su propia representación, de acuerdo
a lo solicitado.
El Diagrama Cartesiano se utiliza para representar la relación de dos variables,
por ejemplo, la cantidad de matracas compradas y el precio a pagar.
Gráfico: Oferta de matracas
1
2
3
4
Precio
S/.
1 2 3 4
b
Matracas
(unidades)
a
Lleve 3 y
pague 2
10
1h
2h
3h
4h
20 30 40
Tiempo
(horas)
41
Gráfico: Recorrido de la Marcha Juvenil (Espacio y tiempo)
Espacio (cuadras
Av. Brasil)
55Dirección de Educación Básica Alternativa
VISIÓN RETROSPECTIVA
Los estudiantes se preguntan y responden en base a reflexiones argumentadas:
¿Qué me ayudó a realizar correctamente la representación gráfica?
¿Puedo utilizar lo aprendido para solucionar otro problema?
Veamos, de modo abreviado, la resolución del problema siguiendo los 4 pasos de Polya:
FAMILIARIZACIÓN: comprensión del problema
¿Puedes plantear el problema de una manera más sencilla o directa?
¿Cuáles son los datos?
¿Todos los datos son necesarios para resolver el problema?
BÚSQUEDA DE ESTRATEGIAS: diseño o adaptación de una estrategia
Los estudiantes, al analizar cómo resolver el problema se dan cuenta que ya tienen una
parte del total, el 28,6%, y que la diferencia es lo que les falta para completar el 100%, por
tanto lo calcularán en base a una regla de tres simple. Además, necesitan recordar cómo
se elabora un Gráfico Circular y cómo se utiliza el transportador. Veamos:
Para construir un Gráfico Circular debemos recordar
que el ángulo de 360° se obtiene de hacer girar
una semirecta hasta colocarla en su posición inicial.
Ejemplo: en el gráfico de la izquierda la semirecta  
OA  gira 360°
Problema 3
Lee la siguiente situación y representa, utilizando un diagrama circular, el porcentaje
de familias que no tiene agua potable y el porcentaje de familias que sí tiene.
Agua: Derecho de todos ¿Cuántos
accedemos?
En el mundo cerca de 100 millones de personas
no tienen acceso al agua potable. Cada año más
de3millonesymediomuerenporenfermedades
transmitidas por agua contaminada. La diarrea
es una de ellas; mata más niños menores de 5
años que el SIDA, la malaria y la viruela juntos.
En Perú: 2 376 534 viviendas (28,6% del total)
no tienen agua potable, significa que las familias
que las habitan no acceden a este elemento
fundamental para la dignidad humana.
360°
0°,360°180°
270°
90°
O A
56 Módulo de Orientaciones para el docente del Área de Matemática de EBA
EJECUCIÓN DEL PLAN: Ejecución de la estrategia
Los estudiantes, en base a la estrategia planificada y lo recordado resuelven el problema:
Observan que el resultado debe presentarse como un número natural ya que las personas
son individualidades.
Resultado: 8 309 559 = 71,4%
Representamos en un Gráfico Circular el resultado obtenido y la información dada:
2 376 534 = 28,6%
A C36°
B
Entonces, 28,6% corresponde en un gráfico
circular a 103°. La diferencia será 71,4% que
corresponde a 257° (360 – 103)
⇒
	 =	 102,96
	 Redondeando	 =	 103
28,6 x 360
100
Calculamos la región circular (ángulo) que
corresponde al porcentaje dado (28,6%)
El total → 360° =   100%
Una parte →      x =  28,6%
270°
90°
180° 360°
103°
28,6%
71,4%
257°
2 376 534 (28,6%)
de viviendas no
tienen agua potable
8 309 559 (71,4%)
de viviendas si
tienen agua potable.
Recordemos también cómo se utiliza el transportador.
¿Cuántas viviendas de Perú sí tienen acceso al agua potable?
Sabemos que 2 376 534 equivale al 28,6%. El total de viviendas equivale al 100%. Si
restamos ambos porcentajes obtendremos el porcentaje de viviendas que sí tienen
agua potable: 100 – 28,6 = 71,4
Para calcular la cantidad a la que equivale el 71,4% utilizamos la regla de tres
simple, veamos:
2 376 534 x 71,4
28,6
= 8 309 559,4⇒
2 376 534	 →	 28,6%     
χ			 →	 71,4%

Weitere ähnliche Inhalte

Was ist angesagt?

Experiencia de Aprendizaje 1 Matematica 2022 Ccesa007.pdf
Experiencia de Aprendizaje 1 Matematica 2022  Ccesa007.pdfExperiencia de Aprendizaje 1 Matematica 2022  Ccesa007.pdf
Experiencia de Aprendizaje 1 Matematica 2022 Ccesa007.pdfDemetrio Ccesa Rayme
 
Sesion de estadistica
Sesion de estadisticaSesion de estadistica
Sesion de estadisticavictor alegre
 
Proyecto matemáticas
Proyecto matemáticasProyecto matemáticas
Proyecto matemáticassayn18
 
Unidad didáctica tercero de secundaria
Unidad didáctica tercero de secundariaUnidad didáctica tercero de secundaria
Unidad didáctica tercero de secundariajaime villanueva
 
Sesion de construccion de triangulos
Sesion de construccion de triangulosSesion de construccion de triangulos
Sesion de construccion de triangulosCarlos Collanqui
 
Sesion de aprendizaje triangulos notables
Sesion de aprendizaje  triangulos notablesSesion de aprendizaje  triangulos notables
Sesion de aprendizaje triangulos notablesDanteCalderon1
 
Sesión interés simple
Sesión interés simpleSesión interés simple
Sesión interés simpleVilma Bravo
 
Rutas de aprendizaje matematica
Rutas de aprendizaje matematicaRutas de aprendizaje matematica
Rutas de aprendizaje matematicaHIYOCHAJO
 
Diseño de sesión de aprendizaje nº 4
Diseño de sesión de aprendizaje nº 4Diseño de sesión de aprendizaje nº 4
Diseño de sesión de aprendizaje nº 4jaimes11
 
3° sesion de aprendizaje geometria - 2° secundaria
3° sesion de aprendizaje   geometria - 2° secundaria3° sesion de aprendizaje   geometria - 2° secundaria
3° sesion de aprendizaje geometria - 2° secundariaElder Junior Roman Leon
 

Was ist angesagt? (20)

MATEMÁTICA
MATEMÁTICAMATEMÁTICA
MATEMÁTICA
 
Experiencia de Aprendizaje 1 Matematica 2022 Ccesa007.pdf
Experiencia de Aprendizaje 1 Matematica 2022  Ccesa007.pdfExperiencia de Aprendizaje 1 Matematica 2022  Ccesa007.pdf
Experiencia de Aprendizaje 1 Matematica 2022 Ccesa007.pdf
 
Experiencia de-aprendizaje
Experiencia de-aprendizajeExperiencia de-aprendizaje
Experiencia de-aprendizaje
 
Sesion de estadistica
Sesion de estadisticaSesion de estadistica
Sesion de estadistica
 
Proyecto matemáticas
Proyecto matemáticasProyecto matemáticas
Proyecto matemáticas
 
Unidad didáctica tercero de secundaria
Unidad didáctica tercero de secundariaUnidad didáctica tercero de secundaria
Unidad didáctica tercero de secundaria
 
Números Racionales(Sesión de Aprendizaje)
Números Racionales(Sesión de Aprendizaje)Números Racionales(Sesión de Aprendizaje)
Números Racionales(Sesión de Aprendizaje)
 
Sesion 01 d estadistica
Sesion 01 d estadisticaSesion 01 d estadistica
Sesion 01 d estadistica
 
Sesion de construccion de triangulos
Sesion de construccion de triangulosSesion de construccion de triangulos
Sesion de construccion de triangulos
 
Sesion052 do
Sesion052 doSesion052 do
Sesion052 do
 
Sesion tablas y graficos estadísticos
Sesion tablas y graficos estadísticosSesion tablas y graficos estadísticos
Sesion tablas y graficos estadísticos
 
Sesión de aprendizaje
Sesión de aprendizajeSesión de aprendizaje
Sesión de aprendizaje
 
Sesion de aprendizaje triangulos notables
Sesion de aprendizaje  triangulos notablesSesion de aprendizaje  triangulos notables
Sesion de aprendizaje triangulos notables
 
Sesión interés simple
Sesión interés simpleSesión interés simple
Sesión interés simple
 
Sesion de aprendizaje 6° estadística
Sesion de aprendizaje 6° estadísticaSesion de aprendizaje 6° estadística
Sesion de aprendizaje 6° estadística
 
Rutas de aprendizaje matematica
Rutas de aprendizaje matematicaRutas de aprendizaje matematica
Rutas de aprendizaje matematica
 
Diseño de sesión de aprendizaje nº 4
Diseño de sesión de aprendizaje nº 4Diseño de sesión de aprendizaje nº 4
Diseño de sesión de aprendizaje nº 4
 
SESION PRIMERO DE SECUNDARIA MATE
SESION  PRIMERO DE  SECUNDARIA MATESESION  PRIMERO DE  SECUNDARIA MATE
SESION PRIMERO DE SECUNDARIA MATE
 
3° sesion de aprendizaje geometria - 2° secundaria
3° sesion de aprendizaje   geometria - 2° secundaria3° sesion de aprendizaje   geometria - 2° secundaria
3° sesion de aprendizaje geometria - 2° secundaria
 
Silabo vi y viii semestre didáctica de la matemática
Silabo vi y viii semestre didáctica de la matemáticaSilabo vi y viii semestre didáctica de la matemática
Silabo vi y viii semestre didáctica de la matemática
 

Andere mochten auch

Capacitación 15 10-2016. eba(1)
Capacitación 15 10-2016. eba(1)Capacitación 15 10-2016. eba(1)
Capacitación 15 10-2016. eba(1)Edgar Sanchez
 
Capacitacion docente 2015 3
Capacitacion docente 2015 3Capacitacion docente 2015 3
Capacitacion docente 2015 3Edgar Sanchez
 
Experiencia explosion de colores
Experiencia explosion de coloresExperiencia explosion de colores
Experiencia explosion de coloresJaviera Guzmán
 
Cuadro de situacion de aprendizaje
Cuadro de situacion de aprendizajeCuadro de situacion de aprendizaje
Cuadro de situacion de aprendizajeEdgar Sanchez
 
DiseñO Curricular BáSico Nacional Eba Reajustado
DiseñO Curricular BáSico Nacional Eba ReajustadoDiseñO Curricular BáSico Nacional Eba Reajustado
DiseñO Curricular BáSico Nacional Eba Reajustadotellinos
 
Simulacro Maestroeduca. Nombramiento y reubicación docente 2015
Simulacro Maestroeduca. Nombramiento y reubicación docente 2015Simulacro Maestroeduca. Nombramiento y reubicación docente 2015
Simulacro Maestroeduca. Nombramiento y reubicación docente 2015Francisco VÁSQUEZ CARRILLO
 
Visibilidad y diseminación del concocimiento
Visibilidad y diseminación del concocimientoVisibilidad y diseminación del concocimiento
Visibilidad y diseminación del concocimientoFlor Maria Mera Ortiz
 
Estrategias para el fortalecimiento de la matematica
Estrategias para el fortalecimiento de la matematicaEstrategias para el fortalecimiento de la matematica
Estrategias para el fortalecimiento de la matematicadogremarlucy
 
1. Simulacro de examen subprueba conocimientos pedagógicos 2015
1. Simulacro de examen subprueba conocimientos pedagógicos 20151. Simulacro de examen subprueba conocimientos pedagógicos 2015
1. Simulacro de examen subprueba conocimientos pedagógicos 2015Julio Najarro Laura
 
Teorías y enfoques para la práctica pedagógica
Teorías y enfoques para la práctica pedagógicaTeorías y enfoques para la práctica pedagógica
Teorías y enfoques para la práctica pedagógicaYaritza Ferreira
 
Simulacro de Evaluación para la II Escala Magisterial
Simulacro de Evaluación para la II Escala MagisterialSimulacro de Evaluación para la II Escala Magisterial
Simulacro de Evaluación para la II Escala MagisterialFrancisco VÁSQUEZ CARRILLO
 
Examen simulacro docente de nombramiento 2015-augeperu
Examen simulacro docente de nombramiento  2015-augeperuExamen simulacro docente de nombramiento  2015-augeperu
Examen simulacro docente de nombramiento 2015-augeperuColegio
 

Andere mochten auch (20)

Sesiòn de aprendizaje eba
Sesiòn de aprendizaje  ebaSesiòn de aprendizaje  eba
Sesiòn de aprendizaje eba
 
Módulo cas (1)
Módulo cas (1)Módulo cas (1)
Módulo cas (1)
 
Capacitación 15 10-2016. eba(1)
Capacitación 15 10-2016. eba(1)Capacitación 15 10-2016. eba(1)
Capacitación 15 10-2016. eba(1)
 
Capacitacion docente 2015 3
Capacitacion docente 2015 3Capacitacion docente 2015 3
Capacitacion docente 2015 3
 
Experiencia explosion de colores
Experiencia explosion de coloresExperiencia explosion de colores
Experiencia explosion de colores
 
Cuadro de situacion de aprendizaje
Cuadro de situacion de aprendizajeCuadro de situacion de aprendizaje
Cuadro de situacion de aprendizaje
 
¿Cómo rendir con éxito un examen docente?
¿Cómo rendir con éxito un examen docente?¿Cómo rendir con éxito un examen docente?
¿Cómo rendir con éxito un examen docente?
 
Recursos pedagogicos
Recursos pedagogicosRecursos pedagogicos
Recursos pedagogicos
 
Diapositivas de eba
Diapositivas de  ebaDiapositivas de  eba
Diapositivas de eba
 
DiseñO Curricular BáSico Nacional Eba Reajustado
DiseñO Curricular BáSico Nacional Eba ReajustadoDiseñO Curricular BáSico Nacional Eba Reajustado
DiseñO Curricular BáSico Nacional Eba Reajustado
 
Simulacro Maestroeduca. Nombramiento y reubicación docente 2015
Simulacro Maestroeduca. Nombramiento y reubicación docente 2015Simulacro Maestroeduca. Nombramiento y reubicación docente 2015
Simulacro Maestroeduca. Nombramiento y reubicación docente 2015
 
Visibilidad y diseminación del concocimiento
Visibilidad y diseminación del concocimientoVisibilidad y diseminación del concocimiento
Visibilidad y diseminación del concocimiento
 
Estrategias para el fortalecimiento de la matematica
Estrategias para el fortalecimiento de la matematicaEstrategias para el fortalecimiento de la matematica
Estrategias para el fortalecimiento de la matematica
 
12. modulo-progr-curricular-en-eba
12. modulo-progr-curricular-en-eba12. modulo-progr-curricular-en-eba
12. modulo-progr-curricular-en-eba
 
1. Simulacro de examen subprueba conocimientos pedagógicos 2015
1. Simulacro de examen subprueba conocimientos pedagógicos 20151. Simulacro de examen subprueba conocimientos pedagógicos 2015
1. Simulacro de examen subprueba conocimientos pedagógicos 2015
 
Teorías y enfoques para la práctica pedagógica
Teorías y enfoques para la práctica pedagógicaTeorías y enfoques para la práctica pedagógica
Teorías y enfoques para la práctica pedagógica
 
Simulacro de Evaluación para la II Escala Magisterial
Simulacro de Evaluación para la II Escala MagisterialSimulacro de Evaluación para la II Escala Magisterial
Simulacro de Evaluación para la II Escala Magisterial
 
CEBA "Elvira Garcia y Garcia
CEBA "Elvira Garcia y GarciaCEBA "Elvira Garcia y Garcia
CEBA "Elvira Garcia y Garcia
 
Educacion Basica Alternativa (2)
Educacion Basica Alternativa (2)Educacion Basica Alternativa (2)
Educacion Basica Alternativa (2)
 
Examen simulacro docente de nombramiento 2015-augeperu
Examen simulacro docente de nombramiento  2015-augeperuExamen simulacro docente de nombramiento  2015-augeperu
Examen simulacro docente de nombramiento 2015-augeperu
 

Ähnlich wie Mod matecompleto

Bases curriculares 7° a 2° medio matematica
Bases curriculares 7° a 2° medio matematicaBases curriculares 7° a 2° medio matematica
Bases curriculares 7° a 2° medio matematicaromibain
 
Bases curriculares 7 basico a 2 medio matematica
Bases curriculares 7 basico a 2 medio   matematicaBases curriculares 7 basico a 2 medio   matematica
Bases curriculares 7 basico a 2 medio matematicaInes Caceres
 
Malla de matematicas hecha para cada uno de los grados del colegio
Malla de matematicas hecha para cada uno de los grados del colegioMalla de matematicas hecha para cada uno de los grados del colegio
Malla de matematicas hecha para cada uno de los grados del colegioDanielaRiveraArenas
 
Programación curricular anual de matematica 4° adaptación
Programación curricular anual de matematica 4° adaptaciónProgramación curricular anual de matematica 4° adaptación
Programación curricular anual de matematica 4° adaptaciónJhon Hinostroza Palomino
 
Evaluación censal pisa 2013 3 publicacion mate
Evaluación censal pisa 2013 3 publicacion mateEvaluación censal pisa 2013 3 publicacion mate
Evaluación censal pisa 2013 3 publicacion matecervello2014
 
Programación curricular anual de matematica 2011
Programación curricular anual de matematica 2011Programación curricular anual de matematica 2011
Programación curricular anual de matematica 2011Francisco Contreras
 
Didactica de las Matematicas en Educacion Infantil UNIR Ccesa007.pdf
Didactica de las Matematicas en Educacion Infantil UNIR  Ccesa007.pdfDidactica de las Matematicas en Educacion Infantil UNIR  Ccesa007.pdf
Didactica de las Matematicas en Educacion Infantil UNIR Ccesa007.pdfDemetrio Ccesa Rayme
 
Propuestas del aprendizaje fundamental matemática
Propuestas del aprendizaje fundamental matemáticaPropuestas del aprendizaje fundamental matemática
Propuestas del aprendizaje fundamental matemáticaCarlos Yampufé
 
Prontuario matemáticas contemporáneas
Prontuario matemáticas contemporáneasProntuario matemáticas contemporáneas
Prontuario matemáticas contemporáneasJuan Serrano
 
Prontuario matemáticas contemporáneas
Prontuario matemáticas contemporáneasProntuario matemáticas contemporáneas
Prontuario matemáticas contemporáneasJuan Serrano
 
Didactica de las matematicas en Educacio.pdf
Didactica de las matematicas en Educacio.pdfDidactica de las matematicas en Educacio.pdf
Didactica de las matematicas en Educacio.pdfBritoSay3
 
Bc matem+ítica y estad+¡stica
Bc   matem+ítica y estad+¡sticaBc   matem+ítica y estad+¡stica
Bc matem+ítica y estad+¡sticaivanrcamacho
 
Bc matem+ítica y estad+¡stica
Bc   matem+ítica y estad+¡sticaBc   matem+ítica y estad+¡stica
Bc matem+ítica y estad+¡sticaivanrcamacho
 
Proyecto Final Ochoa Rojas Celso Diplomado En Linea.
Proyecto Final   Ochoa Rojas Celso Diplomado En Linea.Proyecto Final   Ochoa Rojas Celso Diplomado En Linea.
Proyecto Final Ochoa Rojas Celso Diplomado En Linea.Celso Ochoa
 
Bases matematica 2012
Bases matematica  2012Bases matematica  2012
Bases matematica 2012patulk3
 

Ähnlich wie Mod matecompleto (20)

Bases curriculares 7° a 2° medio matematica
Bases curriculares 7° a 2° medio matematicaBases curriculares 7° a 2° medio matematica
Bases curriculares 7° a 2° medio matematica
 
Bases curriculares 7 basico a 2 medio matematica
Bases curriculares 7 basico a 2 medio   matematicaBases curriculares 7 basico a 2 medio   matematica
Bases curriculares 7 basico a 2 medio matematica
 
Mod mate3
Mod mate3Mod mate3
Mod mate3
 
Malla de matematicas hecha para cada uno de los grados del colegio
Malla de matematicas hecha para cada uno de los grados del colegioMalla de matematicas hecha para cada uno de los grados del colegio
Malla de matematicas hecha para cada uno de los grados del colegio
 
Programación curricular anual de matematica 4° adaptación
Programación curricular anual de matematica 4° adaptaciónProgramación curricular anual de matematica 4° adaptación
Programación curricular anual de matematica 4° adaptación
 
Bases 1 2
Bases 1 2Bases 1 2
Bases 1 2
 
Proyecto
ProyectoProyecto
Proyecto
 
Evaluación censal pisa 2013 3 publicacion mate
Evaluación censal pisa 2013 3 publicacion mateEvaluación censal pisa 2013 3 publicacion mate
Evaluación censal pisa 2013 3 publicacion mate
 
Programación curricular anual de matematica 2011
Programación curricular anual de matematica 2011Programación curricular anual de matematica 2011
Programación curricular anual de matematica 2011
 
Didactica de las Matematicas en Educacion Infantil UNIR Ccesa007.pdf
Didactica de las Matematicas en Educacion Infantil UNIR  Ccesa007.pdfDidactica de las Matematicas en Educacion Infantil UNIR  Ccesa007.pdf
Didactica de las Matematicas en Educacion Infantil UNIR Ccesa007.pdf
 
PRUEBAS TIPO PISA MATEMATICA RESUELTAS
PRUEBAS TIPO PISA MATEMATICA RESUELTASPRUEBAS TIPO PISA MATEMATICA RESUELTAS
PRUEBAS TIPO PISA MATEMATICA RESUELTAS
 
Propuestas del aprendizaje fundamental matemática
Propuestas del aprendizaje fundamental matemáticaPropuestas del aprendizaje fundamental matemática
Propuestas del aprendizaje fundamental matemática
 
Prontuario matemáticas contemporáneas
Prontuario matemáticas contemporáneasProntuario matemáticas contemporáneas
Prontuario matemáticas contemporáneas
 
Prontuario matemáticas contemporáneas
Prontuario matemáticas contemporáneasProntuario matemáticas contemporáneas
Prontuario matemáticas contemporáneas
 
Didactica de las matematicas en Educacio.pdf
Didactica de las matematicas en Educacio.pdfDidactica de las matematicas en Educacio.pdf
Didactica de las matematicas en Educacio.pdf
 
Matemáticas
MatemáticasMatemáticas
Matemáticas
 
Bc matem+ítica y estad+¡stica
Bc   matem+ítica y estad+¡sticaBc   matem+ítica y estad+¡stica
Bc matem+ítica y estad+¡stica
 
Bc matem+ítica y estad+¡stica
Bc   matem+ítica y estad+¡sticaBc   matem+ítica y estad+¡stica
Bc matem+ítica y estad+¡stica
 
Proyecto Final Ochoa Rojas Celso Diplomado En Linea.
Proyecto Final   Ochoa Rojas Celso Diplomado En Linea.Proyecto Final   Ochoa Rojas Celso Diplomado En Linea.
Proyecto Final Ochoa Rojas Celso Diplomado En Linea.
 
Bases matematica 2012
Bases matematica  2012Bases matematica  2012
Bases matematica 2012
 

Mehr von Edgar Sanchez

Balotario funcion reguladora y superv. del estado (1)
Balotario funcion reguladora y superv. del estado (1)Balotario funcion reguladora y superv. del estado (1)
Balotario funcion reguladora y superv. del estado (1)Edgar Sanchez
 
Proyecto educativo de educación física dos
Proyecto educativo de educación física dosProyecto educativo de educación física dos
Proyecto educativo de educación física dosEdgar Sanchez
 
Casos de estrategias uno
Casos de estrategias unoCasos de estrategias uno
Casos de estrategias unoEdgar Sanchez
 
Casos de estrategias uno (1)
Casos de estrategias uno (1)Casos de estrategias uno (1)
Casos de estrategias uno (1)Edgar Sanchez
 
Capacitacion para cetpro (1)
Capacitacion para cetpro (1)Capacitacion para cetpro (1)
Capacitacion para cetpro (1)Edgar Sanchez
 
Examen escrito de ciencia
Examen escrito de cienciaExamen escrito de ciencia
Examen escrito de cienciaEdgar Sanchez
 
Interes compuesto manolito1
Interes compuesto manolito1Interes compuesto manolito1
Interes compuesto manolito1Edgar Sanchez
 
Las seis llaves maestras de las matemáticas financieras
Las seis llaves maestras de las matemáticas financierasLas seis llaves maestras de las matemáticas financieras
Las seis llaves maestras de las matemáticas financierasEdgar Sanchez
 
Examen escrito de razonamiento matematico series
Examen escrito de razonamiento matematico seriesExamen escrito de razonamiento matematico series
Examen escrito de razonamiento matematico seriesEdgar Sanchez
 
Evolución histórica del dólar
Evolución histórica del dólarEvolución histórica del dólar
Evolución histórica del dólarEdgar Sanchez
 
Cta1 programacion anual
Cta1 programacion anualCta1 programacion anual
Cta1 programacion anualEdgar Sanchez
 
Carpetapedagogicapercyhuanuco 130412104125-phpapp02
Carpetapedagogicapercyhuanuco 130412104125-phpapp02Carpetapedagogicapercyhuanuco 130412104125-phpapp02
Carpetapedagogicapercyhuanuco 130412104125-phpapp02Edgar Sanchez
 
Capacitacion docente 2015 simulacros de examenes ocentes con 742
Capacitacion docente 2015 simulacros de examenes ocentes con 742Capacitacion docente 2015 simulacros de examenes ocentes con 742
Capacitacion docente 2015 simulacros de examenes ocentes con 742Edgar Sanchez
 
Capacitacion docente 2015 simulacros de examenes ocentes con 742
Capacitacion docente 2015 simulacros de examenes ocentes con 742Capacitacion docente 2015 simulacros de examenes ocentes con 742
Capacitacion docente 2015 simulacros de examenes ocentes con 742Edgar Sanchez
 
Capacitacion docente 2015 6
Capacitacion docente 2015 6Capacitacion docente 2015 6
Capacitacion docente 2015 6Edgar Sanchez
 

Mehr von Edgar Sanchez (20)

Balotario funcion reguladora y superv. del estado (1)
Balotario funcion reguladora y superv. del estado (1)Balotario funcion reguladora y superv. del estado (1)
Balotario funcion reguladora y superv. del estado (1)
 
Sesion ecuaciones
Sesion ecuacionesSesion ecuaciones
Sesion ecuaciones
 
Proyecto educativo de educación física dos
Proyecto educativo de educación física dosProyecto educativo de educación física dos
Proyecto educativo de educación física dos
 
Casos de estrategias uno
Casos de estrategias unoCasos de estrategias uno
Casos de estrategias uno
 
Casos de estrategias uno (1)
Casos de estrategias uno (1)Casos de estrategias uno (1)
Casos de estrategias uno (1)
 
Capacitacion para cetpro (1)
Capacitacion para cetpro (1)Capacitacion para cetpro (1)
Capacitacion para cetpro (1)
 
Examen escrito de ciencia
Examen escrito de cienciaExamen escrito de ciencia
Examen escrito de ciencia
 
Interes compuesto manolito1
Interes compuesto manolito1Interes compuesto manolito1
Interes compuesto manolito1
 
Las seis llaves maestras de las matemáticas financieras
Las seis llaves maestras de las matemáticas financierasLas seis llaves maestras de las matemáticas financieras
Las seis llaves maestras de las matemáticas financieras
 
Examen escrito de razonamiento matematico series
Examen escrito de razonamiento matematico seriesExamen escrito de razonamiento matematico series
Examen escrito de razonamiento matematico series
 
Cta1 unidad1
Cta1 unidad1Cta1 unidad1
Cta1 unidad1
 
Evolución histórica del dólar
Evolución histórica del dólarEvolución histórica del dólar
Evolución histórica del dólar
 
Cta1 u1 sesion4
Cta1 u1 sesion4Cta1 u1 sesion4
Cta1 u1 sesion4
 
Cta1 u1 sesion2
Cta1 u1 sesion2Cta1 u1 sesion2
Cta1 u1 sesion2
 
Cta1 u1 sesion1 (1)
Cta1 u1 sesion1 (1)Cta1 u1 sesion1 (1)
Cta1 u1 sesion1 (1)
 
Cta1 programacion anual
Cta1 programacion anualCta1 programacion anual
Cta1 programacion anual
 
Carpetapedagogicapercyhuanuco 130412104125-phpapp02
Carpetapedagogicapercyhuanuco 130412104125-phpapp02Carpetapedagogicapercyhuanuco 130412104125-phpapp02
Carpetapedagogicapercyhuanuco 130412104125-phpapp02
 
Capacitacion docente 2015 simulacros de examenes ocentes con 742
Capacitacion docente 2015 simulacros de examenes ocentes con 742Capacitacion docente 2015 simulacros de examenes ocentes con 742
Capacitacion docente 2015 simulacros de examenes ocentes con 742
 
Capacitacion docente 2015 simulacros de examenes ocentes con 742
Capacitacion docente 2015 simulacros de examenes ocentes con 742Capacitacion docente 2015 simulacros de examenes ocentes con 742
Capacitacion docente 2015 simulacros de examenes ocentes con 742
 
Capacitacion docente 2015 6
Capacitacion docente 2015 6Capacitacion docente 2015 6
Capacitacion docente 2015 6
 

Kürzlich hochgeladen

Estrategias de enseñanza - aprendizaje. Seminario de Tecnologia..pptx.pdf
Estrategias de enseñanza - aprendizaje. Seminario de Tecnologia..pptx.pdfEstrategias de enseñanza - aprendizaje. Seminario de Tecnologia..pptx.pdf
Estrategias de enseñanza - aprendizaje. Seminario de Tecnologia..pptx.pdfAlfredoRamirez953210
 
FICHA DE MONITOREO Y ACOMPAÑAMIENTO 2024 MINEDU
FICHA DE MONITOREO Y ACOMPAÑAMIENTO  2024 MINEDUFICHA DE MONITOREO Y ACOMPAÑAMIENTO  2024 MINEDU
FICHA DE MONITOREO Y ACOMPAÑAMIENTO 2024 MINEDUgustavorojas179704
 
Introducción:Los objetivos de Desarrollo Sostenible
Introducción:Los objetivos de Desarrollo SostenibleIntroducción:Los objetivos de Desarrollo Sostenible
Introducción:Los objetivos de Desarrollo SostenibleJonathanCovena1
 
5° SEM29 CRONOGRAMA PLANEACIÓN DOCENTE DARUKEL 23-24.pdf
5° SEM29 CRONOGRAMA PLANEACIÓN DOCENTE DARUKEL 23-24.pdf5° SEM29 CRONOGRAMA PLANEACIÓN DOCENTE DARUKEL 23-24.pdf
5° SEM29 CRONOGRAMA PLANEACIÓN DOCENTE DARUKEL 23-24.pdfOswaldoGonzalezCruz
 
CIENCIAS NATURALES 4 TO ambientes .docx
CIENCIAS NATURALES 4 TO  ambientes .docxCIENCIAS NATURALES 4 TO  ambientes .docx
CIENCIAS NATURALES 4 TO ambientes .docxAgustinaNuez21
 
PPT GESTIÓN ESCOLAR 2024 Comités y Compromisos.pptx
PPT GESTIÓN ESCOLAR 2024 Comités y Compromisos.pptxPPT GESTIÓN ESCOLAR 2024 Comités y Compromisos.pptx
PPT GESTIÓN ESCOLAR 2024 Comités y Compromisos.pptxOscarEduardoSanchezC
 
NARRACIONES SOBRE LA VIDA DEL GENERAL ELOY ALFARO
NARRACIONES SOBRE LA VIDA DEL GENERAL ELOY ALFARONARRACIONES SOBRE LA VIDA DEL GENERAL ELOY ALFARO
NARRACIONES SOBRE LA VIDA DEL GENERAL ELOY ALFAROJosé Luis Palma
 
Fisiologia.Articular. 3 Kapandji.6a.Ed.pdf
Fisiologia.Articular. 3 Kapandji.6a.Ed.pdfFisiologia.Articular. 3 Kapandji.6a.Ed.pdf
Fisiologia.Articular. 3 Kapandji.6a.Ed.pdfcoloncopias5
 
BIOLOGIA_banco de preguntas_editorial icfes examen de estado .pdf
BIOLOGIA_banco de preguntas_editorial icfes examen de estado .pdfBIOLOGIA_banco de preguntas_editorial icfes examen de estado .pdf
BIOLOGIA_banco de preguntas_editorial icfes examen de estado .pdfCESARMALAGA4
 
Presentación de Estrategias de Enseñanza-Aprendizaje Virtual.pptx
Presentación de Estrategias de Enseñanza-Aprendizaje Virtual.pptxPresentación de Estrategias de Enseñanza-Aprendizaje Virtual.pptx
Presentación de Estrategias de Enseñanza-Aprendizaje Virtual.pptxYeseniaRivera50
 
periodico mural y sus partes y caracteristicas
periodico mural y sus partes y caracteristicasperiodico mural y sus partes y caracteristicas
periodico mural y sus partes y caracteristicas123yudy
 
Instrucciones para la aplicacion de la PAA-2024b - (Mayo 2024)
Instrucciones para la aplicacion de la PAA-2024b - (Mayo 2024)Instrucciones para la aplicacion de la PAA-2024b - (Mayo 2024)
Instrucciones para la aplicacion de la PAA-2024b - (Mayo 2024)veganet
 
TRIPTICO-SISTEMA-MUSCULAR. PARA NIÑOS DE PRIMARIA
TRIPTICO-SISTEMA-MUSCULAR. PARA NIÑOS DE PRIMARIATRIPTICO-SISTEMA-MUSCULAR. PARA NIÑOS DE PRIMARIA
TRIPTICO-SISTEMA-MUSCULAR. PARA NIÑOS DE PRIMARIAAbelardoVelaAlbrecht1
 
La evolucion de la especie humana-primero de secundaria
La evolucion de la especie humana-primero de secundariaLa evolucion de la especie humana-primero de secundaria
La evolucion de la especie humana-primero de secundariamarco carlos cuyo
 
Tarea 5-Selección de herramientas digitales-Carol Eraso.pdf
Tarea 5-Selección de herramientas digitales-Carol Eraso.pdfTarea 5-Selección de herramientas digitales-Carol Eraso.pdf
Tarea 5-Selección de herramientas digitales-Carol Eraso.pdfCarol Andrea Eraso Guerrero
 
Uses of simple past and time expressions
Uses of simple past and time expressionsUses of simple past and time expressions
Uses of simple past and time expressionsConsueloSantana3
 
LA OVEJITA QUE VINO A CENAR CUENTO INFANTIL.pdf
LA OVEJITA QUE VINO A CENAR CUENTO INFANTIL.pdfLA OVEJITA QUE VINO A CENAR CUENTO INFANTIL.pdf
LA OVEJITA QUE VINO A CENAR CUENTO INFANTIL.pdfNataliaMalky1
 

Kürzlich hochgeladen (20)

Estrategias de enseñanza - aprendizaje. Seminario de Tecnologia..pptx.pdf
Estrategias de enseñanza - aprendizaje. Seminario de Tecnologia..pptx.pdfEstrategias de enseñanza - aprendizaje. Seminario de Tecnologia..pptx.pdf
Estrategias de enseñanza - aprendizaje. Seminario de Tecnologia..pptx.pdf
 
FICHA DE MONITOREO Y ACOMPAÑAMIENTO 2024 MINEDU
FICHA DE MONITOREO Y ACOMPAÑAMIENTO  2024 MINEDUFICHA DE MONITOREO Y ACOMPAÑAMIENTO  2024 MINEDU
FICHA DE MONITOREO Y ACOMPAÑAMIENTO 2024 MINEDU
 
Introducción:Los objetivos de Desarrollo Sostenible
Introducción:Los objetivos de Desarrollo SostenibleIntroducción:Los objetivos de Desarrollo Sostenible
Introducción:Los objetivos de Desarrollo Sostenible
 
Tema 7.- E-COMMERCE SISTEMAS DE INFORMACION.pdf
Tema 7.- E-COMMERCE SISTEMAS DE INFORMACION.pdfTema 7.- E-COMMERCE SISTEMAS DE INFORMACION.pdf
Tema 7.- E-COMMERCE SISTEMAS DE INFORMACION.pdf
 
5° SEM29 CRONOGRAMA PLANEACIÓN DOCENTE DARUKEL 23-24.pdf
5° SEM29 CRONOGRAMA PLANEACIÓN DOCENTE DARUKEL 23-24.pdf5° SEM29 CRONOGRAMA PLANEACIÓN DOCENTE DARUKEL 23-24.pdf
5° SEM29 CRONOGRAMA PLANEACIÓN DOCENTE DARUKEL 23-24.pdf
 
CIENCIAS NATURALES 4 TO ambientes .docx
CIENCIAS NATURALES 4 TO  ambientes .docxCIENCIAS NATURALES 4 TO  ambientes .docx
CIENCIAS NATURALES 4 TO ambientes .docx
 
PPT GESTIÓN ESCOLAR 2024 Comités y Compromisos.pptx
PPT GESTIÓN ESCOLAR 2024 Comités y Compromisos.pptxPPT GESTIÓN ESCOLAR 2024 Comités y Compromisos.pptx
PPT GESTIÓN ESCOLAR 2024 Comités y Compromisos.pptx
 
NARRACIONES SOBRE LA VIDA DEL GENERAL ELOY ALFARO
NARRACIONES SOBRE LA VIDA DEL GENERAL ELOY ALFARONARRACIONES SOBRE LA VIDA DEL GENERAL ELOY ALFARO
NARRACIONES SOBRE LA VIDA DEL GENERAL ELOY ALFARO
 
Fisiologia.Articular. 3 Kapandji.6a.Ed.pdf
Fisiologia.Articular. 3 Kapandji.6a.Ed.pdfFisiologia.Articular. 3 Kapandji.6a.Ed.pdf
Fisiologia.Articular. 3 Kapandji.6a.Ed.pdf
 
PPTX: La luz brilla en la oscuridad.pptx
PPTX: La luz brilla en la oscuridad.pptxPPTX: La luz brilla en la oscuridad.pptx
PPTX: La luz brilla en la oscuridad.pptx
 
BIOLOGIA_banco de preguntas_editorial icfes examen de estado .pdf
BIOLOGIA_banco de preguntas_editorial icfes examen de estado .pdfBIOLOGIA_banco de preguntas_editorial icfes examen de estado .pdf
BIOLOGIA_banco de preguntas_editorial icfes examen de estado .pdf
 
Presentación de Estrategias de Enseñanza-Aprendizaje Virtual.pptx
Presentación de Estrategias de Enseñanza-Aprendizaje Virtual.pptxPresentación de Estrategias de Enseñanza-Aprendizaje Virtual.pptx
Presentación de Estrategias de Enseñanza-Aprendizaje Virtual.pptx
 
periodico mural y sus partes y caracteristicas
periodico mural y sus partes y caracteristicasperiodico mural y sus partes y caracteristicas
periodico mural y sus partes y caracteristicas
 
Instrucciones para la aplicacion de la PAA-2024b - (Mayo 2024)
Instrucciones para la aplicacion de la PAA-2024b - (Mayo 2024)Instrucciones para la aplicacion de la PAA-2024b - (Mayo 2024)
Instrucciones para la aplicacion de la PAA-2024b - (Mayo 2024)
 
TRIPTICO-SISTEMA-MUSCULAR. PARA NIÑOS DE PRIMARIA
TRIPTICO-SISTEMA-MUSCULAR. PARA NIÑOS DE PRIMARIATRIPTICO-SISTEMA-MUSCULAR. PARA NIÑOS DE PRIMARIA
TRIPTICO-SISTEMA-MUSCULAR. PARA NIÑOS DE PRIMARIA
 
La evolucion de la especie humana-primero de secundaria
La evolucion de la especie humana-primero de secundariaLa evolucion de la especie humana-primero de secundaria
La evolucion de la especie humana-primero de secundaria
 
Tarea 5-Selección de herramientas digitales-Carol Eraso.pdf
Tarea 5-Selección de herramientas digitales-Carol Eraso.pdfTarea 5-Selección de herramientas digitales-Carol Eraso.pdf
Tarea 5-Selección de herramientas digitales-Carol Eraso.pdf
 
TL/CNL – 2.ª FASE .
TL/CNL – 2.ª FASE                       .TL/CNL – 2.ª FASE                       .
TL/CNL – 2.ª FASE .
 
Uses of simple past and time expressions
Uses of simple past and time expressionsUses of simple past and time expressions
Uses of simple past and time expressions
 
LA OVEJITA QUE VINO A CENAR CUENTO INFANTIL.pdf
LA OVEJITA QUE VINO A CENAR CUENTO INFANTIL.pdfLA OVEJITA QUE VINO A CENAR CUENTO INFANTIL.pdf
LA OVEJITA QUE VINO A CENAR CUENTO INFANTIL.pdf
 

Mod matecompleto

  • 1. Estrategias Metodológicas para el área de Matemática en EBA MÓDULO
  • 2. MINISTERIO DE EDUCACIÓN Ministro de Educación Jaime Saavedra Chanduví Viceministro de Gestión Pedagógica Flavio Felipe Figallo Rivadeneyra Viceministro de Gestión Institucional Juan Pablo Silva Macher Directora General de Educación Básica Alternativa, Intercultural Bilingüe y de Servicios Educativos en el Ámbito Rural - DIGEIBIRA Elena Antonia Burga Cabrera Dirección de Educación Básica Alternativa - DEBA Luis Alberto Hiraoka Mejía Módulo de Orientaciones para el docente del Área de Matemática de EBA © Ministerio de Educación Av. De la Arqueología, cuadra 2, San Borja Lima, Perú Teléfono: 615-5800 www.minedu.gob.pe Primera Edición Octubre de 2015
  • 3. 3Dirección de Educación Básica Alternativa PRESENTACIÓN El propósito de este módulo es contribuir a mejorar la práctica pedagógica del docente de Educación Básica Alternativa mediante el fortalecimiento de sus capacidades para construir y dinamizar procesos de enseñanza y aprendizaje de la matemática, centrados en el estudiante y su contexto. A través de la comprensión y aplicación de este módulo los docentes del Área de Matemática serán capaces de: • Identificar situaciones reales que enfrentan y resuelven los estudiantes haciendo uso de conocimientos matemáticos. • Diseñar experiencias de aprendizaje, en base a las situaciones identificadas, que integren los saberes del estudiante con procesos constructivos y significativos del aprendizaje de la matemática. • Desarrollar las experiencias de aprendizaje en armonía con la cultura de los estudiantes y el desarrollo de sus capacidades para el logro de competencias matemáticas.
  • 4. 4 Estrategias Metodológicas para el Área de Comunicación Integral ÍNDICE Fundamentación del área de matemática y situaciones problemáticas 1. Fundamentación del área de matemática 6 2. Situaciones problemáticas 11 Cómo aprendemos matemática, sus procesos y componentesen EBA 1. ¿Cómo aprendemos matemática? 19 2. Procesos del área de matemática según el DCBN EBA 23 2.1. Resolución de problemas 23 2.2. Razonamiento y demostración 23 2.3. Comunicación matemática 23 3. Componentes del área de matemática según el DCBN EBA 24 3.1. Sistemas numéricos y funciones 24 3.2. Geometría y medida 25 3.3. Estadística y probabilidad 27 Orientaciones para aplicar estrategias en el área de matemática 1. Secuencia didáctica de Brousseau 32 2. La investigación 41 3. Resolución de problemas 48 Unidad 1 Unidad 2 Unidad 3
  • 5. Unidad 1 Fundamentación del área de matemática y situaciones problemáticas
  • 6. 6 Módulo de Orientaciones para el docente del Área de Matemática de EBA Fundamentación del área de matemática1 Docente A: A través del aprendizaje de la matemática, los estudiantes de EBA desarrollan capacidades para resolver una variedad de operaciones y problemas tipo. De este modo, están mejor preparados para continuar estudios superiores. Docente B A través del aprendizaje de la matemática, los estudiantes de EBA desarrollan capacidades para comprender y resolver las situaciones problemáticas que enfrentan. De este modo, aprenden a actuar y pensar matemáticamente ante diversas situaciones. 1. En relación a las posturas de los docentes A y B: ¿qué aspectos compartes y cuáles no? ¿Por qué? 2. ¿Cómo diseñarías una Tabla para presentar tus respuestas?
  • 7. 7Dirección de Educación Básica Alternativa La matemática no es ajena a los jóvenes y adultos, descubren su importancia a partir de la necesidadde resolverlassituaciones problemáticasquesepresentan enla vidacotidiana:comprar, vender, organizar sus cuentas, tomar decisiones en base a datos cuantitativos, diseñar y describir formas, interpretar gráficos, representar rutas, estimar tiempos, analizar y reflexionar sobre el costo de los servicios básicos, etc. Todo ello demanda el uso de conceptos, procedimientos y estrategias matemáticas. El desarrollo de competencias matemáticas en los jóvenes y adultos debe partir de situaciones problemáticas reales que correspondan a su experiencia de vida, atiendan sus urgencias y las demandas de la sociedad. Las situaciones deben ser analizadas y entendidas en su complejidad a través de los conocimientos que ofrece la matemática, dado que el estudiante se enfrenta a ellas en su día a día y necesita solucionarlas. Actuar y pensar matemáticamente es fundamental para el estudiante, de este modo amplía su horizonte y capacidades para plantear y resolver problemas en los que interviene la matemática, utiliza y elabora estrategias, razona, argumenta, representa y comunica. Dos facetas de la matemática La matemática está en permanente construcción, es fruto de un proceso histórico. Concebirla como ciencia acabada, exacta y rigurosamente deductiva tiene consecuencias negativas en el plano de la enseñanza y aprendizaje, pues distorsionan la orientación y las actividades que se proponen y desarrollan. Los aspectos deductivos de esta ciencia son una faceta de ella, evidente en la cadena de pasos lógicos y rigurosidad que lleva a una verdad irrefutable. La otra faceta está asociada a su proceso de elaboración, incluye aspectos como la intuición, conjeturas, exploración, creatividad, motivaciones y las emociones. En el siguiente gráfico se muestra las distintas estrategias que emplean algunas personas para calcular mentalmente el cambio de S/150 a euros: Utilizar la matemática e involucrarse con el papel que tienen en el mundo es fundamental paraquelosjóvenesyadultosenriquezcanelconocimientoquetienendelarealidad,tomen decisiones informadas y desarrollen la capacidad de hacer juicios bien fundamentados.
  • 8. 8 Módulo de Orientaciones para el docente del Área de Matemática de EBA Las matemáticas son un producto cultural Toda persona desarrolla los procesos formativos de su personalidad en el ámbito de determinada cultura. No hay cultura sin personas y no habrá consciencia ni pensamiento sin cultura. La actividad matemática y las habilidades, actitudes y conocimientos asociados son un componente importante de cada cultura y medio social, facilitan la relación y comunicación entre personas y con el entorno, contribuyen a las actividades científicas y tecnológicas y a la mejor comprensión de uno mismo. Al expresar determinadas pautas de racionalidad e involucrar un lenguaje se desarrollan capacidades de relación, representación y cuantificación. Por ejemplo, al elaborar estadísticas empleamos procedimientos similares, comunicamos los resultados usando términos consensuados e interpretamos información en base a criterios comunes. Contribuye a expresar y potenciar múltiples actividades, entre ellas las científicas y tecnológicas. En el mundo se han identificado seis actividades fundamentales que constituyen fuente para el desarrollo de la matemática: contar, localizar, medir, diseñar, jugar y explicar. (DIBUJO ADAPTADO DE “LAS REMESAS“, de Jorge Llieff. No incluye los cálculos) 1 euro = S/3,43 y 10 euros S/34,3 20 euros = S/68,6 y 40 euros S/137,2 S/150 – S/137,2 = S/12,8 casi 4 euros Entonces 40+4 = casi 44 euros CASA DE CAMBIO S/1 = 0,29 euros S/100 = 29 euros S/50 = 14,5 euros S/150=29+14,5=43,5 euros S/100 es 29 euros S/150 es como 29+15 Casi 44 euros S/1=0,29 euros S/1,5 es casi 0,44 Sí, es casi 44 euros
  • 9. 9Dirección de Educación Básica Alternativa Un enfoque intercultural de la educación matemática Los estudiantes de EBA son parte de una sociedad multicultural y plurilingüe. La manera peculiar cómo los grupos sociales y culturales –no solo pueblos originarios– construyen (o reconstruyen) los conocimientos, desarrollan sus habilidades y establecen sus actitudes ha recibido especial atención de la disciplina llamada “Etnomatemática”. El currículo debe promover un proceso de interacción cultural entre el docente y el estudiante; con el objetivo de que este último reciba una educación matemática enriquecida, pertinente para los múltiples escenarios, reconstruyendo crítica y comprensivamente las conceptualizaciones, procedimientos y valores de la cultura matemática. El tratamiento curricular del área de matemática debe tener un enfoque intercultural. Los procesos de enseñanza y aprendizaje de la matemática están inmersos en la basta diversidad cultural inherente al país y el mundo globalizado. Es responsabilidad del docente ayudar a procesar, crítica y creativamente, la herencia cultural matemática. Por ejemplo, los jóvenes y adultos utilizan la matemática al desempeñar procesos productivos de los que depende su subsistencia. En éstas situaciones se evidencia la forma de entender y relacionarse con los conceptos y procedimientos matemáticos: medir, seguir secuencias, contar, estimar precios de venta, calcular utilidades y pérdidas, etc. Educación matemática y equidad En ocasiones la matemática ha sido utilizada para discriminar, desalentando a los estudiantes con dificultades de aprendizaje e influyendo en su retiro o abandono de las aulas. Lademocratizacióndelaculturaexigelaincorporacióndetoda nuestrapoblaciónal conocimiento, los valores y las actitudes inherentes a la educación básica. Es decir, un núcleo de conceptos, procedimientos y actitudes matemáticas que debe formar parte del bagaje cultural que domina cada ciudadano. En los distintos niveles de concreción curricular se debe tener especial cuidado en identificar lo anterior, distinguiéndolos de aquellos orientados a la formación ocupacional o especialización
  • 10. 10 Módulo de Orientaciones para el docente del Área de Matemática de EBA con alta exigencia. Por ejemplo, todos debemos plantear y resolver problemas utilizando determinados conceptos, procesos, algoritmos y estrategias; sin embargo, cada uno parte de situaciones reales de su contexto (sembrar, medir, comprar, vender, etc.) y progresivamente las amplía. Peculiaridades en la enseñanza y el aprendizaje de los estudiantes de EBA Los estudiantes de EBA desarrollan ciertas formas de hacer y aprender la matemática. Los docentes debemos identificarlas, investigar y potenciarlas. Por ejemplo, sus procedimientos de cálculo son distintos a los que provienen del contexto académico, ello debido a que aprendieron a calcular ante la necesidad de realizar transacciones de compra y venta. Tienen capacidades, habilidades y destrezas que han desarrollado y continúan aplicando en diferentes grados. Algunas de sus fortalezas son el cálculo mental, las estimaciones, las comparaciones cuantitativas, los procedimientos de localización, el uso de la calculadora, entre otras. Ello explica su desempeño exitoso en actividades de comercio minorista e incluso mayorista, la facilidad para movilizarse en geografías complejas y la diversidad de labores que realizan; pese al nulo o bajo nivel educativo. ¿Qué debe hacer el docente? El docente debe facilitar oportunidades de aprendizaje para que el estudiante sea capaz de valorar y utilizar el aporte de la matemática en la comprensión de su entorno físico y cultural, identificando y resolviendo problemas relacionados con su contexto. Además, reconocer que los estudiantes han construido saberes matemáticos y estrategias en su interacción con el medio y las personas, por lo que su experiencia constituye el punto de partida para el desarrollo de la competencia matemática. El Área de Matemática promueve experiencias significativas para que los estudiantes construyan sus aprendizajes, en forma individual y en cooperación con otros, en un encuentro enriquecedor del saber matemático desarrollado en su experiencia de vida, con las capacidades, conocimientos y actitudes propias de la matemática. DCBN EBA
  • 11. 11Dirección de Educación Básica Alternativa Es importante que los estudiantes de EBA descubran en la matemática un instrumento o herramienta intelectual que les ayude a solucionar diversas situaciones que se les presenta. En ese sentido el rol del docente es generar experiencias de aprendizaje, partiendo de situaciones de la vida cotidiana. ¿Qué es una situación problemática? Una situación problemática describe una actividad o escenario donde se desenvuelve el estudiante haciendo uso de los conocimientos matemáticos. No es cualquier situación, debe permitir explicar la funcionalidad de dichos conocimientos, de este modo podrán ser utilizadas para dinamizar aprendizajes. En la situación se evidencian necesidades, intereses, desafíos o dificultades. El estudiante explora, moviliza y desarrolla saberes; es decir, evidencia sus competencias y capacidades de forma integrada. Las situaciones problemáticas provienen del contexto real, social, científico y matemático. A. Situaciones del contexto real del estudiante Las situaciones de contexto real se refieren a la experiencia personal, familiar, laboral, social, comunitaria y pública de la vida de los estudiantes. Comprenden desafíos relacionados a la persona y sus percepciones, familia y grupo de pares: comprar y pagar, utilizar medidas al preparar alimentos, transportarse, viajar, finanzas personales, cuidado de la salud, etc. Situaciones problemáticas2
  • 12. 12 Módulo de Orientaciones para el docente del Área de Matemática de EBA Aprendí a movilizarme en la ciudad, no es igual que aquí. Ahora calculo y optimizo mi tiempo en función a las rutas, comparo los costos de los diferentes medios de transporte y calculo mi presupuesto Situación: movilizarse para realizar las actividades cotidianas Las personas utilizamos la matemática para calcular el presupuesto que requerimos para movilizarnos, seleccionar la ruta más adecuada, optimizar los tiempos, etc. Realizamos cálculos simples y planteamos problemas a partir de situaciones problemáticas propias del entorno inmediato y la rutina cotidiana. Para realizar éstas acciones y otras similares necesitamos tener nociones previas y ejecutar procesos que requieren pensar y actuar matemáticamente ante situaciones que involucran números, operaciones, cantidades, equivalencias, cambios, formas, movimientos, localización, gestionar datos, etc. Observa
  • 13. 13Dirección de Educación Básica Alternativa Reflexiona 1. ¿Cuánto dinero gastas mensualmente en movilizarte a tu trabajo? ¿Qué distancia recorres? ¿Cuánto tiempo demoras? 2. ¿Cuál es la mejor oferta de yogurt? ¿Por qué? El docente orienta a los estudiantes hacia el planteamiento y resolución de problemas referidos a sus contextos de vida y, progresivamente, introduce problemas que no siendo exclusivos de esos contextos incluyan escenarios familiares y requieran modelos matemáticos para su solución. Durante este proceso el estudiante: • Matematiza las situaciones: reflexiona, reconoce y extrae lasmatemáticas contenidas en la situación. • Realiza procesos de conexión que involucran ideas y procedimientos matemáticos. • Razona, analiza y comunica operaciones matemáticas. • Utiliza el razonamiento matemático. • Utiliza y construye estrategias. 3. ¿En qué situaciones de tu día a día utilizas la matemática? 4. ¿Cómo describirías dos situaciones de contexto real?
  • 14. 14 Módulo de Orientaciones para el docente del Área de Matemática de EBA Observa B. Situaciones del contexto social y científico Las situaciones de contexto social y científico están relacionados con la naturaleza, la vida, las ciencias de la salud, las ciencias sociales, los conocimientos científicos y el uso de la tecnología. Si bien es cierto las personas están involucradas en todas las situaciones, en la categoría de contexto social y científico el foco está en la perspectiva de retos relacionados con las ciencias naturales, sociales, la ciencia y la tecnología: transporte público, sistemas de votación, gobierno, políticas públicas, demografía, publicidad, estadísticas nacionales y económicas. Miércoles 4 de Abril del 2012 GESTIÓN Resuelve • ¿Cómo comunicarías oralmente esta información gráfica? • ¿Qué opinas de la demanda y oferta del transporte urbano en Lima? Fuente: MACROCONSULT Oferta y Demanda del Transporte Urbano en Lima (2012)
  • 15. 15Dirección de Educación Básica Alternativa Reflexiona Observa Las situaciones descritas requieren utilizar y desarrollar modelos matemáticos que implican identificar información; utilizar, describir, explicar y evaluar el comportamiento de un sistema de datos mediante el uso de variables, conceptos, operaciones y expresiones matemáticas eidentificarlascondiciones, restricciones y relaciones planteadas. La persona debe tener capacidad para transformar el problema a una forma matemática e interpretar o valorar los resultados o modelo obtenido en relación al problema original. En este proceso es posible que utilice directamente un modelo matemático, lo modifique o combine. El desarrollo de modelos hace posible que la persona encuentre significado a sus saberes matemáticos y los construya de forma progresiva al contrastarlos con la realidad. Ante el crecimiento de la población mundial la ciencia y la tecnología tienen un rol importante para asegurar su subsistencia en condiciones de calidad. Además, las personas debemos ser más responsables. ¿Cómo contribuye la matemática a este propósito? Situación: optimización del agua potable Optimizar el uso del agua potable en el hogar, trabajo u otro espacio implica estimar el volumen de agua consumida en las actividades cotidianas. Identifica dos ejemplos del contexto social y científico que evidencien la utilización de la matemática. Descríbelos.
  • 16. 16 Módulo de Orientaciones para el docente del Área de Matemática de EBA C. Situaciones de contexto matemático Las situaciones de contexto matemático son retos o desafíos del propio conocimiento matemático. Utilizar el conocimiento matemático para resolver situaciones problemáticas implica enfrentar y resolver retos o desafíos del propio contexto matemático. Examina: • El 50% más el 20% de 2000 es 200 • 1/3 de 300 es 100 Situación: representación gráfica (contexto matemático) Presentar cada resultado del Estudio realizado por la ONG “Lima Cómo Vamos” utilizando distintos tipos de gráficos estadísticos. Resolver la situación requiere superar desafíos del contexto matemático: • Comprender nociones de porcentaje. • Comprender procedimientos propios de la estadística. • Elaborar diversos tipos de gráficos estadísticos. • Analizar las diversas posibilidades de representación gráfica y elegir la más adecuada para comunicar cada resultado. El estudiante necesita dominar conocimientos del contexto matemático y conocer información que le permita resolver la situación. Por ejemplo: Conocimientos - Área del rectángulo: concepto y cálculo. - Multiplicación y división: números de dos cifras enteras y un decimal. Información - Medidas del triángulo Situación: Hallar el área de un triángulo de 10 cm de base y 7 cm de altura. Examina:
  • 17. 17Dirección de Educación Básica Alternativa Reflexiona Situación: Pintar el cerco externo e interno de una IE Información: • Medidas del cerco: 225 m de largo x 4 m de alto • Venta de pintura en dos presentaciones: balde de 5 galones= S/. 105 y balde de 1 galón = S/. 25,50 • 1 litro de pintura rinde 10 m2 • 1 galón = 3,8 litros Preguntas ¿Cuánto es lo mínimo que se puede gastar en la compra de pintura? ¿Qué conocimientos matemáticos utilizaste? Situación: Elaborar una torta de sorpresas utilizando cartulina y estimar el precio de venta para ganar el 25% de lo invertido. Preguntas • ¿Qué conocimientos del contexto matemático necesita dominar el estudiante? • ¿Qué información le darías al estudiante para que pueda resolver la situación? De acuerdo a la información que darás al estudiante: ¿cuántos pliegos de cartulina debe comprar?
  • 18. Unidad 2 Cómo aprendemos matemática, sus procesos y componentes en EBA
  • 19. 19Dirección de Educación Básica Alternativa Para los jóvenes y adultos de EBA, la matemática está fuertemente vinculada a las actividades que realizan; han tenido que aprenderla ante la necesidad de no ser engañados al comercializar sus productos con el mejor precio posible, participar en actividades económicas, etc. Aplicar la matemática les permite hacer, entender y proyectar con éxito procesos relacionados a su subsistencia y cotidianidad. En ese sentido la resolución de problemas es un aspecto fundamental en el desarrollo de la matemática y también se puede decir que ayuda al estudiante a desarrollar su pensamiento abstracto y lógico. Enfoque centrado en la resolución de problemas El enfoque centrado en la resolución de problemas se promueven aprendizajes a través de, sobre y para la resolución de problemas (Gaulin 2001) ¿Cómo aprendemos matemática?1 La resolución de Problemas debe plantearse en situaciones significativas de contexto diverso, pues ello moviliza el pensamiento matemático. Los estudiantes deben encontrar significado a la resolución de problemas, valorar el conocimiento matemático que se aplica y establecer relaciones de funcionalidad. La matemática se enseña y aprende resolviendo problemas. Los estudiantes contruyen nuevos conceptos, descubren relaciones, elaboran procedimientos y establecen relaciones.
  • 20. 20 Módulo de Orientaciones para el docente del Área de Matemática de EBA A través de la resolución de problemas y el entorno del estudiante construye significados, organiza objetos matemáticos y genera nuevos aprendizajes en un sentido constructivo y creador de la actividad humana. Sobre la resolución de problemas, el estudiante explica la necesidad de reflexionar sobre los procesos empleados al resolver problemas: planeación, estrategias heurísticas, recursos, procedimientos, conocimientos y capacidades movilizadas. Para resolver problemas el estudiante enfrenta constantemente nuevas situaciones y problemas. La resolución de problemas es el proceso central de hacer matemática, es el medio principal para ver la funcionalidad de la matemática. Permite al estudiante situarse en diversos contextos para crear, recrear, investigar y resolver problemas; utilizando diversos caminos de resolución, el análisis de estrategias y formas de representación, la sistematización y comunicación de nuevos conocimientos, etc. El estudiante de EBA evidenciará que es competente en matemática si tiene la facultad de actuar conscientemente en la resolución de problemas o el cumplimiento de exigencias complejas relacionadas a la matemática, usando flexible y creativamente sus conocimientos y habilidades, información o herramientas, así como sus valores, emociones y actitudes. De este modo actúa y piensa matemáticamente ante diversas situaciones problemáticas. El enfoque basado en la resolución de problemas para el despliegue de capacidades supone desarrollar competencias relacionadas a matematizar situaciones problemáticas que implican construir modelos, utilizar estrategias al formular y resolver problemas, razonar y argumentar la validez y pertinencia de los resultados alcanzados y comunicar los hallazgos. Enseñanza Aprendizaje Enfoque centrado en la resolución de problemas “A través de” Resolución de problemas Actuar y pensar matemáticamente “Sobre la” “Para la”
  • 21. 21Dirección de Educación Básica Alternativa Para una mejor comprensión, a continuación se desarrollan conceptos fundamentales asociados a aprender competencias: Competencia El aprendizaje de una competencia es de carácter longitudinal, se reitera una y otra vez a fin de que pueda irse complejizando de manera progresiva y permita al estudiante alcanzar niveles cada vez más altos de desempeño. Toda competencia implica actuar conscientemente sobre una situación, un problema o un objetivo para alcanzar un logro esperado; evidenciando dominio, uso flexible y creativo de capacidades y ética. Requiere saber transferir las capacidades del contexto en que fueron aprendidos a otras situaciones, seleccionándolas y combinándolas en función del logro esperado. Alcanzar desempeños competentes es un proceso complejo; no solo porque al actuar en situación, para alcanzar el logro esperado, se deben evidenciar desempeños idóneos de carácter vinculante con otras competencias sino porque no es cualquier situación o logro; son aquellos que contribuyen al desarrollo personal y colectivo, con sentido y compromiso ético. Aprendizaje situado Las competencias se aprenden en función de situaciones. El concepto de situación es el elemento central del proceso de aprendizaje. Es en situación que los jóvenes y adultos construyen, modifican o refutan conocimientos contextualizados y desarrolla competencias a la vez situadas. No se trata de aprender contenidos disciplinares descontextualizados (área, triángulo, ecuaciones, etc.) sino de definir situaciones en las que sea posible construir, modificar o refutar conocimientos, por ende, desarrollar capacidades y evidenciar desempeños competentes. Las competencias se desarrollan a partir de situaciones identificadas en la experiencia de vida de los jóvenes y adultos. Estas son fuente de aprendizaje y generadoras de situaciones de aprendizaje. En este sentido, todo aprendizaje debe ser situado (Lave, 2003). Pensamiento matemático La competencia es un saber actuar de manera reflexiva y eficiente, tanto en el campo de las relaciones de las personas con la naturaleza, con los objetos, con las ideas; como en el de las relaciones sociales. Este saber actuar no alude solamente a una capacidad manual, técnica, operativa, sino además a un saber cómo, por qué y para qué hacerlo. Fuente: DCBN EBA De otro lado, pensar matemáticamente se define como el conjunto de actividades mentales u operaciones intelectuales que llevan al estudiante a dotar de significado a lo que les rodea, resolver problemas sobre conceptos matemáticos, tomar una decisión o llegar a una conclusión, en la que están involucrados procesos como la abstracción, justificación, visualización, estimación, entre otros. Cantoral 2005; Molina 2006; Carretero y Ascencio 2008.
  • 22. 22 Módulo de Orientaciones para el docente del Área de Matemática de EBA Los jóvenes y adultos enfrentan regularmente situaciones que deben resolver haciendo uso de conocimientos matemáticos: comprar, vender, movilizarse de un lugar a otro, gestionar su presupuesto y actividad laboral, pagar servicios básicos e impuestos, evaluar préstamos de dinero, juzgar cuestiones políticas y asuntos públicos, etc. En este proceso desarrollan una variedad de estrategias y despliegan habilidades relacionadas con el razonamiento (cuantitativo y espacial) y la intuición, basan su accionar en fundamentos, conceptos, habilidades y estrategias matemáticas. Es decir, utilizan su pensamiento matemático para resolver problemas y desempeñarse de modo competente en la vida cotidiana. El sentido y utilidad de la matemática para los jóvenes y adultos está en el manejo y aplicación de conceptos, operaciones, cálculos y medidas de uso común para identificar y solucionar problemas relacionados a su vida personal, laboral, familiar y participación ciudadana; asimismo, manejar instrumentos y medios tecnológicos, desde calculadoras básicas –ser analfabeto no es un impedimento para utilizarlas- hasta tecnologías de información y comunicación. Por otro lado, el ejercicio de su ciudadanía les demanda comprender y reflexionar sobre los fenómenos sociales y ambientales, asimismo, emitir juicios y propuestas constructivas basadas en el manejo y comprensión de información cuantitativa. La matemática no está ausente de la cultura de los jóvenes y adultos ni de los procesos de asimilación a otras culturas, a la que se incorporan con el objetivo de mejorar su condición económica. En ambos escenarios el aprendizaje de la matemática es funcional y se constituye en una herramienta para la sobrevivencia y exige reforzar y desarrollar su pensamiento matemático. Vemos que el desarrollo del pensamiento matemático en jóvenes y adultos surge ante la necesidad de comprender y solucionar situaciones de vida concretas, estas son oportunidades para movilizar y poner en funcionamiento sus conocimientos, estrategias y recursos. Viabilizar sus competencias matemáticas implica superar contenidos y enfoques desvinculados de su quehacer e interés, desarrollar metodologías y materiales educativos que posibiliten el uso de la matemática como recurso básico para la vida, generar experiencias de aprendizaje que rescaten sus saberes y les posibiliten plantear y resolver problemas en base a situaciones problemáticas de su contexto personal, cultural, natural y social, asociadas a nuevas demandas del mundo del trabajo y la ciudadanía activa; desarrollando una variedad de estrategias. Competencia matemática • La competencia matemática es entendida como la capacidad de analizar, razonar y comunicar según se plantean y resuelven los problemas que surgen del desarrollo personal y la plena integración en la sociedad de la comunicación. • Engloba tres dimensiones adecuadamente relacionadas: contenidos, procesos y situaciones o contextos. • Se aplica para resolver los problemas de la vida adulta y afrontar exigencias de diferente nivel y tipo, de este modo es un prerrequisito o base para seguir aprendiendo a lo largo de la vida. • Se demuestra en la ejecución autónoma de sucesivos procesos cognitivos, entre los que se encuentran las destrezas básicas de cálculo, se centra en el proceso y el razonamiento más que en el conocimiento.
  • 23. 23Dirección de Educación Básica Alternativa El área de matemática en el DCBN EBA integra los procesos fundamentales con los componentes, los cuales son asumidos como grandes bloques de contenidos. Adiciona a ello las actitudes que contribuyen a una sólida formación integral. Esta es una forma de concreción de las intencionalidades educativas. En el DCBN EBA se asumen tres procesos: 2.1. Resolución de problemas Los problemas deben ser formulados y elegidos con la intención de posibilitar que lo aprendido se consolide y amplíe, asimismo, se construya nuevos conocimientos. Implica identificar los problemas dentro de contextos reales y superar la tendencia a reducirlos a tratamientos abstractos y descontextualizados. Es importante ser consciente del proceso seguido en la resolución de problemas y evaluar el avance. El proceso de resolución de problemas sirve de contexto para el desarrollo de otros procesos fundamentales. Al resolverlos necesariamente se razona y comunica y se interconecta ideas matemáticas y representan. 2.2. Razonamiento y demostración Las actividades de aprendizaje de la matemática deben propiciar que los estudiantes desarrollen y evalúen argumentos utilizando nociones, conceptos y procedimientos matemáticos. Es decir, que aprendan a razonar, tanto de manera heurística como deductiva. Razonar heurísticamente implica hacer uso de la intuición, las conjeturas, la inducción a partir de regularidades o patrones, tanto en situaciones del mundo real como en objetos simbólicos; asimismo, preguntarse si son patrones accidentales o si hay razones para que aparezcan. En forma progresiva se introduce la argumentación deductiva, la simbolización, la abstracción, el rigor y la precisión. Un error es reducir el razonamiento al adiestramiento en “problemas tipo” o trabajar un curso de razonamiento matemático en paralelo al desarrollo del área de matemática. El docente debe propiciar un clima favorable a la libre expresión de ideas, sentimientos y expectativas, fomentar la discusión e insistir en la elaboración de argumentos. El estudiante debe aprender a proporcionar los fundamentos o razones de sus decisiones y valorarlas críticamente; asimismo, derivar las implicancias de una situación hipotética y ser flexible a la modificación de su punto de vista enbase a argumentos. 2.3. Comunicación matemática Permite expresar, compartir y aclarar ideas; las cuales llegan a ser objetos de reflexión, perfeccionamiento, discusión, análisis y reajuste. Este proceso involucra emociones y actitudes, ayuda a dar significado y permanencia a las ideas y difundirlas. Las emociones y actitudes deben ser moduladas, ya que pueden facilitar u obstaculizar el aprendizaje de la matemática. La matemática aporta un lenguaje preciso, el cual permite a las personas expresar e interpretar ideas matemáticas con argumentos convincentes, la exploración sistemática de alternativas, flexibilidad en el razonamiento y la organización, consolidación y comunicación del pensamiento matemático. El docente debe posibilitar que los estudiantes incorporen a su habla personal distintas formas de expresión matemática: numérica, gráfica, geométrica, algebraica y probabilística. Además, capacitar a los estudiantes para analizar y evaluar las estrategias y el conocimiento matemático implicado en las actividades de las personas con las que interactúa, comunicándose con pertinencia y compartiendo un significado y sentido. Procesos del área de matemática según el DCBN EBA2
  • 24. 24 Módulo de Orientaciones para el docente del Área de Matemática de EBA En el DCBN EBA se asumen tres componentes para el área de matemática. Éstos se conciben como grandes bloques de contenidos: • Sistemas numéricos y funciones • Geometría y medida • Estadística y probabilidad Cada componente es un medio para describir, comprender e interpretar los fenómenos naturales y sociales que han determinado el desarrollo de determinados procedimientos y conceptos matemáticos propios de situaciones relacionadas al componente; por ejemplo: • Situaciones de cantidad, se modelan desde los sistemas numéricos y funciones. • Situaciones de forma, se representan y relacionan desde la geometría y medida. • Situaciones probabilísticas, se estiman ý representan desde la estadística y probabilidad. Analicemos cada componente: 3.1. Sistemas numéricos y funciones Nuestra sociedad está teñida por los números, desde nuestras cuentas personales y actividades cotidianas hasta las prioridades nacionales y mundiales. Aprendemos sobre ellos pero no siempre lo aplicamos a nuestra cotidianidad. Veamos el siguienteejemplo: Aprendemos cuando somos capaces de atribuir significado a lo que aprendemos: “Proceso de Construcción de Significados”. Sin embargo, tambiénaprendemos sin darle significado al contenido o acción, en ese caso, solo somos capaces de memorizar y repetir sin entender, por tanto, no somos capaces de transferir ese aprendizajea otro escenario. Componentes del área de matemática según el DCBN EBA3 50% + 20% es 70% de descuento. Son S/30 por la blusa No señora, el 50% de 100 es 50 y el 20% de 50 es 10. El descuento es 50+10= 60. Debe pagar S/40 Blusa S/. 100 Rebaja: 50% + 20%
  • 25. 25Dirección de Educación Básica Alternativa Lo significativo varía de una a otra persona, los docentes más que generalizar aprendizajes debemos buscar aquellos que sean lo más significativos posibles para quienes aprenden. Por ejemplo, en relación a los sistemas numéricos, para nuestros estudiantes las siguientes situaciones son significativas: • Sueldo (mensual, quincenal, diario) • Deudas y préstamos, pagos y cuotas. • Precios de productos de la canasta familiar. • Ofertas en anuncios publicitarios. • Temperatura, radiación solar. • Consumo de celulares: minutos y soles. • Costo de medicinas, etc. El componente sistemas numéricos y funciones, desarrollado en el DCBN EBA, incluye el estudio de los números, sus distintas formas de representarlos, las operaciones, las relaciones entre ellos y con conjuntos de números, los sistemas numéricos, el álgebra y las funciones. La orientación del abordaje es amplia, va más allá del manejo elemental de operaciones básicas y la destreza operatoria con expresiones algebraicas. Específicamente, en cada uno de los ciclos de EBA, el abordaje de este componente se centra en: Ciclo inicial e intermedio Ciclo avanzado • Desarrollo del sentido numérico, de modo que haya comprensión de los números. • Relaciones entre los números. • Significado de las operaciones. • Cálculo fluido. • Estimaciones razonables. • Regularidades y funciones, de modo sistemático. • Identificación, representación y utilización de las estructuras matemáticas utilizando el simbolismo apropiado. • Elaboración de modelos elementales para representar o comprender relaciones cuantitativas de situaciones o fenómenos reales. 3.2. Geometría y medida En nuestro mundo encontramos gran diversidad de objetos y situaciones que se relacionan con geometría y las mediciones. Esta tendencia es creciente en el futuro, por lo que es importante abordar a través del aprendizaje de la matemática habilidades necesarias para desenvolverse con éxito en este y otros escenarios. Los jóvenes y adultos experimentan diversas situaciones en las que se enfrentan a problemas espaciales, los resuelven con mayor o menor éxito, por lo general, de modo empírico. Ello les ha permitido construir una serie de referencias, por ejemplo: movilizarse para ubicar una dirección identificada en un mapa de calles les exige localizar puntos de referencia y reconocer distancias, construir sus casas exige la aplicación de nociones geométricas: áreas, altura, peso, volumen, etc.
  • 26. 26 Módulo de Orientaciones para el docente del Área de Matemática de EBA Pese a tener conocimientos empíricos de geometría logran construir colaborativamente y con acierto sus viviendas. ¿Es básico y elemental su conocimiento? ¿Aplican las teorías que nos cuesta tanto entender? ¿Influye el aprendizaje colaborativo? Lo que aprendemos tiene relación con cómo percibimos nuestro entorno, un hecho o situación determinada. También con cómo lo observamos, no solo con los ojos sino con todos nuestros sentidos y emociones involucradas. Veamos cómo aprenden los jóvenes y adultos en espacios formales y no formales la geometría: En mi comunidad aprendí a medir el terreno para sembrar, calcular cuántas semillas y agua necesito por hectárea, pesar la cosecha… ¡Ahora me resulta más fácil aprender lo de aquí! Optimicé los cortes. De cada bloque de madera, salieron 10 piezas. Se necesitará 2m2 más de tejas. 1 galón de pintura da para dos habitaciones. La proporción de la mezcla es de 2 por 3.
  • 27. 27Dirección de Educación Básica Alternativa En el medio en el que interactúan, los jóvenes y adultos, enfrentan una serie de situaciones en las que están en contacto con diversos objetos en los que pueden reconocer formas y cuerpos geométricos que van desde empaques (cajas de diversas formas en las que viene artículos que compran o venden) hasta construcciones fabulosas (edificios en los que trabajan como obreros de construcción) pasando por una serie de artículos tecnológicos cada vez más sofisticados y modernos (celulares, calculadoras, televisores, etc.). Aprender geometría relacionada a la serie de situaciones que enfrentan es fundamental paralosjóvenesyadultos.Lespermitecomprenderyaplicarlosconocimientosmatemáticos relacionados, interactuar con los objetos optimizando espacios, costos, desarrollar el sentido de ubicación en el espacio, comprender las propiedades de las formas y cómo se interrelacionan, etc. Sin embargo, el aprendizaje no solo es cuestión de conocimiento tiene que ver con cómo perciben el entorno y activan sus capacidades, sentidos y emociones, además, con la capacidad de modificar la estructura actual transfiriendo lo que se aprendió a otros escenarios. El componente geometría y medida, desarrollado en el DCBN EBA aborda el estudio de las características y propiedades de las figuras y cuerpos geométricos, la localización y descripción de relaciones espaciales mediante coordenadas y otros sistemas de representación,lasimetría ylastransformaciones(traslación,reflexión,rotación, ampliación, reducción) para analizar situaciones matemáticas y del entorno, la comprensión de los atributos susceptibles de medición de los objetos, y los sistemas de unidades, procesos e instrumentos de medición. 3.3. Estadística y probabilidad Los estudiantes jóvenes y adultos enfrentan un mundo saturado de información y datos, los cuales son fuente para la toma de decisiones. Necesitan desarrollar su pensamiento estadístico para comprenderlos y mejorar sus posibilidades de éxito al interactuar en la sociedad. Así por ejemplo, para los estudiantes de EBA de la zona rural es importante comprender las condiciones meteorológicas, los pronósticos del clima los ciclos de los fenómenos naturales, etc., para planificar su sembrío y crianza de animales, comercializar sus productos, etc. La utilidad de la estadística la experimentaran en la medida que puedan relacionarla a las situaciones que enfrentan y actuar usando los conocimientos y procesos respectivos. A lo anterior se suma la necesidad de los jóvenes y adultos de ejercer a plenitud su ciudadanía. Ello implica, por ejemplo, tener la capacidad de interpretar y evaluar críticamente la información estadística que presentan los medios informativos en noticias, tablas y gráficos; asimismo, argumentar, discutir y comunicar opiniones y emitir juicio crítico.
  • 28. 28 Módulo de Orientaciones para el docente del Área de Matemática de EBA Vivimos en una sociedad caracterizada por el crecimiento acelerado de la información y desarrollo tecnológico que posibilita el tratamiento de grandes cantidades de información. En ese sentido, el componente de estadística y probabilidad presentado en el DCBN EBA involucra la organización, análisis y gestión de datos mediante herramientas eficaces. Por otro lado, se aborda el tratamiento matemático de situaciones inciertas, el análisis de datos y gráficos asociados a ellas, la evaluación de riesgos y beneficios, posibilitando tomar decisiones con fundamento. Además, permite comprender juegos de azar, seguros, simulación de situaciones y la confiabilidad de los resultados. El estudiante de EBA debe aprender a: • Recopilar, procesar, interpretar y valorar los datos. • Analizar las situaciones involucradas. • Desarrollar modelos expresando un lenguaje estadístico. • Emplear variadas representaciones para organizar datos.
  • 29. Unidad 3 Orientaciones para aplicar estrategias en el área de matemática
  • 30. 30 Módulo de Orientaciones para el docente del Área de Matemática de EBA Cómo desarrollar competencias matemáticas En las unidades anteriores se ha sustentado la importancia de la matemática en la vida cotidiana, en el sistema social productivo, el ambiente, la ciencia, la tecnología, etc. Además, lo significativo que resulta presentarla y aprenderla como próxima a la realidad y en toda su funcionalidad. Hemos analizado la importancia de orientar el planteamiento y la resolución de problemas a partir de situaciones reales de diversos contextos, despertando actitudes favorables hacia y con la matemática. A lo largo de éste proceso es fundamental el desarrollo del pensamiento matemático, mediante la realización de tareas y actividades de progresiva complejidad que impliquen retos y dificultades cognitivas. Sin retos no hay aprendizajes, corremos el riesgo de quedarnos solo en la selección de situaciones y memorización o repetición de rutinas. Se debe incentivar en los estudiantes el razonamiento, la argumentación, la investigación e indagación, la identificación y generación de estrategias, la representación y comunicación de resultados; es decir, retarlos constantemente para que actúen y piensen matemáticamente en diversas situaciones. Desarrollar competencias matemáticas es un proceso complejo y dinámico, requiere la interacción de varios factores e involucra procesos cognitivos. El docente debe garantizar este proceso recurriendo a tareas y actividades matemáticas que generan una interacción dinámica entre situaciones relacionadas a la vida y la práctica social del estudiante, el desarrollo de procesos cognitivos y la construcción de los conocimientos matemáticos. Veamos: Hemos visto que las competencias incluyen conocimientos, habilidades, actitudes y valores; este conjunto de elementos son los recursos con los que contamos para resolver problemas, solo cuando los movilizamos y utilizamos en ámbitos específicos evidenciamos nuestras competencias y el nivel alcanzado. Son, por tanto, un sistema complejo de comprensión y actuación en que se evidencia un saber y un querer: saber pensar, saber decir y saber hacer; y un querer vinculado con las emociones, necesidades e intereses de nuestra vida. En el caso de los estudiantes de EBA, su vida gira en torno a su familia, trabajo y comunidad; si lo que aprenden no lo vinculan a dichos aspectos simplemente lo olvidarán y, por consiguiente, no aprenderán. Situaciones Presentadas en diversos contextos: personal, social y científico o matemático. Relacionados a la vida y práctica social de los estudiantes Construcción de los conocimientos matemáticos Desarrollo de procesos cognitivos
  • 31. 31Dirección de Educación Básica Alternativa Para que los estudiantes de EBA logren competencias matemáticas se propone la construcción y ejecución de experiencias de aprendizaje globalizadoras y contextualizadas. Dicha construcción requiere la identificación de situaciones problemáticas en una variedad de contextos relacionados a la vida de los estudiantes. Las situaciones de la vida cotidiana son ideales para ser tratadas como situaciones problemáticas, sin embargo, debemos tener presente que con frecuencia no suministran directamente datos precisos, por lo que las condiciones e información que evidencian deben ser modificadas para que su tratamiento y solución no sea laborioso y complicado. Para ello, es necesario que el docente desarrolle un proceso de indagación que le facilite adquirir la información adecuada y necesaria. Los estudiantes participan en la construcción de experiencias de aprendizaje organizados en grupos de trabajo colaborativo. Los jóvenes y adultos desarrollan sus capacidades mediante experiencias de aprendizaje articuladas en torno a situaciones de interés y/o relevancia para sus vidas. Expresan, comparten y analizan sus saberes previos y experiencias de vida, desarrollan habilidades y aprenden unos de otros. Asumen consciencia de sus debilidades, fortalezas y estilos de aprender. A partir de ello experimentan cómo superar las dificultades que enfrentan empleando sus potencialidades. Las experiencias de aprendizaje favorecen la interacción, el apoyo mutuo, la confianza en uno mismo, el respeto por el otro; en general, habilidades intra e interpersonales. Se potencian los aportes y expectativas de los estudiantes con niveles distintos de aprendizaje, unos a otros apalancan sus capacidades y evidencian sus competencias. Es indispensable que los estudiantes participen en la planificación y ejecución de las Experiencias de Aprendizaje organizados en Grupos de Inter y Auto aprendizaje (GIA), esta estrategia es clave para renovar la práctica educativa entre estudiantes y docentes y garantizar un adecuado encuentro entre la oferta y la demanda educativa y social. En este proceso el docente es un dinamizador cultural de los procesos educativos, desde y para la comunidad. Los estudiantes, con sus docentes, conforman comunidades de aprendizaje. Comprender el sentido de las estrategias de aprendizaje La selección e implementación de estrategias de aprendizaje tiene sentido en la medida que responde a la comprensión de cómo aprende el estudiante de EBA, el dominio de los aprendizajes involucrados y el logro de competencias. A continuación presentamos orientaciones ejemplificadas para aplicar estrategias en el área de matemática. Consideramos que el docente de Educación Básica Alternativa las aplicará con acierto en las sesiones de aprendizaje, asimismo, las recreará y generará otras. Entendemos las estrategias de aprendizaje como un proceso que integra principios, pautas, y criterios con los procedimientos y actividades mediante las cuales los docentes seleccionan, organizan y realizan las experiencias de aprendizaje en una relación empática con los estudiantes como gestores de sus procesos de aprendizajes. Las estrategias deben estar orientadas a favorecer que los estudiantes: • Desarrollen competencias matemáticas. • Encuentren sentido y satisfacción en lo que aprenden. • Aumenten las posibilidades de éxito en las evaluaciones. • Atribuyan resultados beneficiosos a sus esfuerzos.
  • 32. 32 Módulo de Orientaciones para el docente del Área de Matemática de EBA El docente, además de identificar la situación problemática y plantear la experiencia de aprendizaje, debe tener claridad sobre cómo va a enseñar y cuál es la intención que persigue al desarrollar la experiencia, es decir, organizar la situación didáctica. Presentamos como propuesta la Secuencia Didáctica de Brousseau1, a través de un ejemplo para ser aplicado en el área de matemática con los estudiantes de Primer Grado del Ciclo Inicial de Educación Básica Alternativa. Analicemos información previa: Situaciones didácticas de Brousseau Una situación es didáctica cuando el docente, tiene la intención de enseñar, un saber matemático dado explícitamente y debe darse en un medio. Sus fases son las siguientes: Secuencia didáctica de Brousseau1 1 El gráfico y la descripción de las fases han sido adaptadas de “Rutas del Aprendizaje – Versión 2015 – Área Curricular Matemática – 1° y 2° Gdos de Educación Secundaria. Acción Formulación Validación Evaluación Institucio- nalización a. b. c. e. d.
  • 33. 33Dirección de Educación Básica Alternativa A continuación desarrollamos una experiencia de aprendizaje utilizando la Secuencia Didáctica de Brousseau. Veamos cada una de las fases ejemplificada: Experiencia de Aprendizaje para el Ciclo Inicial de EBA Los estudiantes resolverán la situación problemática: movilizarse para realizar las actividades cotidianas, a través de la “Secuencia Didáctica de Brousseau”. Ciclo: Inicial de EBA Grado: 1ro. Área: matemática Componente del Área: Sistemas numéricos y funciones Competencia, aprendizajes a lograr e indicadores: COMPETENCIA APRENDIZAJES A LOGRAR INDICADOR Resuelve problemas rela- cionados con su entorno a través de estrategias que involucran operaciones de adición y sustracción con números naturales, demos- trando confianza en sus propias capacidades y per- severancia en la búsqueda desoluciones. Expresa e interpreta informa- ción numérica concerniente a su persona, familia, barrio o co- munidad, tanto en sus propios códigos como en lenguaje con- vencional. Ubica los números naturales en la recta numérica. Efectúa operaciones sencillas de adición y sustracción de números naturales menores que 100. Expresa de forma oral y escrita el uso de los números, hasta 100, en contextos de la vida cotidiana. Elabora representaciones de cantidades de hasta 100 objetos de forma gráfica y simbólica. Identifica cantidades de hasta 100 objetos expresándolos en un modelo de solución aditiva. Explica sus procedimientos o resultados de forma breve. Situación problemática: movilizarse para realizar las actividades cotidianas La mayoría de los estudiantes de EBA se movilizan utilizando variados medios de transporte. Identifican, seleccionan y recorren diferentes rutas para ir a trabajar, visitar un familiar o cumplir con un trámite o actividad pendiente. Para realizar este proceso ejecutan acciones que requieren utilizar la matemática: identificación de números, estimación de presupuesto, localización, gestión de datos, optimización del tiempo, selección de rutas, etc. A lo largo de este proceso desarrollan su pensamiento matemático y por ende competencias matemáticas. Saberes previos Antes de presentar una situación los estudiantes deben recuperar sus saberes previos. Las preguntasy comentariosdebencentrarseenla expresión,porparte del estudiante, de información cuantitativa:
  • 34. 34 Módulo de Orientaciones para el docente del Área de Matemática de EBA ¿A qué hora empiezan a movilizarse? ¿Qué rutas siguen? ¿Cuánto tiempo demoran en cada ruta? ¿Cuánto gastan? Experiencia de Aprendizaje: El docente dinamiza un proceso de diálogo para que los estudiantes exploren sus saberes previos: El docente presenta una situación, en texto escrito e ilustrada, referida a la situación problemática a abordar, con información cuantitativa resaltada y datos modificados para su tratamiento y solución. Lee el texto y lo coloca en un lugar visible del aula, resalta la información cuantitativa y la repite para asegurarse que el estudiante la entendió. Veamos: ¿Hacia qué lugares se movilizan? ¿Qué rutas siguen? ¿Cuánto tiempo se demoran? ¡Cuánto gastan? Ana vive en San Juan de Lurigancho. Hoy irá a Villa El Salvador a visitar a su hija. Se movilizará en moto, combi y tren eléctrico. Faltan 5 minutos para las 6 de la mañana. Ana camina durante 3 minutos de su casa al paradero de la moto. Luego de 2 minutos Ana llega en moto al paradero de la combi. De allí, recorre 20 minutos en combi hasta el paradero del tren. Luego de 40 minutos en tren, Ana llega a Villa El Salvador. Decido la ruta para llegar a mi destino En mi pueblo uso acémila. Aquí utilizo moto, combi y tren.
  • 35. 35Dirección de Educación Básica Alternativa a. Fase de Acción Involucra aspectos cognitivos y cuestiones de índole práctica, ambos dirigidos a la solución de problemas que es preciso resolver en condiciones específicas. Acciones del docente Acciones del estudiante Expone la situación y las consignas, y se asegura de que han sido bien comprendidas. Inicia a partir de los conocimientos previos del estudiante, para ello puede diseñar actividades. Leen el problema, analizan los factores que la definen como tal, se identifican con los datos, el propósito, la factibilidad de su resolución y solución. Interviene como mediador. Se abstiene de brindar información que condicione la acción de los estudiantes. Se imaginan la situación apelando a sus saberes previos. Aclara consignas, promueve la aparición de muchas ideas y señala contradicciones en los procedimientos. Movilizan aspectos cognitivos y cuestiones de índole práctica, dirigidas a la solución del problema. Ejemplo: El docente lee con apropiada entonación la situación problemática a los estudiantes, resaltando la información numérica. Luego, los invita a comentar la situación, graficarla, hacer hipótesis, pensar estrategias para solucionar el problema y tentar respuestas. Algunas preguntas podrían ser:  ¿Cómo imaginas la situación?  ¿Cómo podemos representar el problema?  ¿Será útil elaborar un reloj de manecillas? ¿Por qué? Ana camina 3 minutos de su casa al paradero de la moto. La moto llega al paradero de la combi en 2 minutos. 5 minutos para las 6
  • 36. 36 Módulo de Orientaciones para el docente del Área de Matemática de EBA Una vez elaborado el reloj de manecillas, el docente invita a los estudiantes a girar las agujas del reloj para responder a las siguientes preguntas: • ¿Qué hora es cuando Ana llega al paradero de la combi? • ¿Cuánto demoró Ana del paradero de la combi al paradero del tren? • ¿Qué hora es cuando Ana llega al paradero del tren? • ¿Cuánto demoró Ana en llegar del paradero del tren a Villa El Salvador? • ¿Qué hora es cuando Ana llega a Villa El Salvador? • ¿Cuánto demoró Ana en total? b. Fase de formulación Se busca la adquisición de destrezas para la utilización de los lenguajes más apropiados, y se mejora progresivamente la claridad, el orden y la precisión de los mensajes. Acciones del docente Acciones del estudiante Organizar a los estudiantes de modo que puedan dividirse tareas, diseñar y materializar la solución, seleccionar los materiales, las herramientas, etc. Obtiene el plan ordenado, procedimientos, estrategias, recursos y el producto que resuelve los problemas. Indicar las pautas para que los estudiantes utilicen los medios de representación apropiados. Explica los conocimientos en un lenguaje que los demás puedan entender. Utiliza representaciones convencionales para comunicar. Sondear el estado de los saberes previos y los aspectos afectivos y actitudinales. Pone énfasis en el manejo de lenguajes muy variados, ya sea de tipo verbal, escrito, gráfico, plástico, informático o matemático. Detectar procedimientos inadecuados, prejuicios, obstáculos y dificultades, para trabajarlas con los estudiantes, según convenga a su estrategia. Ejemplo: El docente observa los procedimientos que siguen sus estudiantes y los orienta. Los estudiantes planifican y resuelven el problema, representan y comunican sus resultados: • ¿Cuánto crees que demoró Ana en llegar al paradero de la combi? • ¿Cómo sería la representación del problema en una recta numérica? • ¿Cómo representarías la suma? • ¿Qué otras sumas debes realizar? • ¿Cuál es el resultado final? • ¿Has seguido algún orden para resolver el problema? Ana llegó al paradero de la combi en 5 minutos: 3 + 2 = 5 3 + 2 5
  • 37. 37Dirección de Educación Básica Alternativa c. Fase de validación Esunafasedebalanceyrepresentaciónderesultados,yde confrontacióndeprocedimientos Acciones del docente Acciones del estudiante El docente estimula y coordina las pruebas, los ensayos, las exposiciones, los debates y las justificaciones. Los estudiantes verifican sus productos, representaciones y resultados como parte de las situaciones mismas sin tener que recurrir al dictamen del docente.Absuelve las dudas y contradicciones que aparezcan, señala procedimientos diferentes, lenguajes inapropiados, y busca que el consenso valide los saberes utilizados. En ese momento crece el valor de las intervenciones del docente, que debe recurrir a las explicaciones teóricas y metodológicas necesarias, de acuerdo con las dificultades surgidas. Las producciones de las situaciones son sometidas a ensayos y pruebas por sus pares en un proceso metacognitivo que se completa en la fase siguiente. Esta es una buena oportunidad para tomar datos evaluativos y para introducir nuevas variantes de problematización. Confrontan sus procedimientos. Coordina y resume las conclusiones que son clave para la sistematización de la próxima fase. Ejemplo: El docente interviene explicando el significado de la decena y el procedimiento de la suma llevando, de tal manera que los estudiantes puedan aplicarlo a la solución del problema. A lo largo de su intervención, el docente utilizará números distintos a aquellos que están involucrados en el problema. De esta manera, el estudiante podrá hacer la transferencia de lo reforzado a la situación específica que debe resolver. Veamos algunos ejemplos del refuerzo D U 2 0
  • 38. 38 Módulo de Orientaciones para el docente del Área de Matemática de EBA d. Fase de Institucionalización Enestafasesegeneralizayseabstraelosconocimientosenbaseaprocedimientos realizados y resultados obtenidos. Acciones del docente Acciones del estudiante Cumple un rol como mediador de códigos de comunicación. Elestudiantedescontextualizaydespersonaliza el saber para ganar el estatus cultural y social del objeto tecnológico autónomo, capaz de hacerlo funcionar como herramienta eficaz en otras situaciones. Explica, sintetiza, resume y rescata los conocimientos puestos en juego para resolver la situación planteada. Avanza en los niveles de abstracción correspondientes, formalizando conceptos y procedimientos matemáticos, contribuyendo a re significar el aprendizaje en el contexto global, explicando y redondeando el lenguaje matemático apropiado. Destaca la funcionalidad. Rescata el valor de las nociones y los métodos utilizados. Señala su alcance, su generalidad y su importancia. El estudiante traduce la situación, interpreta, realiza representaciones simbólicas, discute sus supuestos en su equipo, se comunica, socializa sus resultados, encuentra el error en el compañero, refuta y generaliza superando los errores y el modelo intuitivo instalado. Formaliza conceptos y procedimientos matemáticos, contribuyendo a resignificar el aprendizaje en el contexto global del estudiante. 3 decenas unidades La suma llevando Para sumar se colocan las unidades debajo de las unidades y las Decenas debajo de las Decenas y se suma. D U 4 8 2 3 7 1 1 1
  • 39. 39Dirección de Educación Básica Alternativa Ejemplo: Los estudiantes comparten sus representaciones, analizan los resultados de otros compañeros. En este proceso explican y discuten sus supuestos, estrategias y resultados. Veamos algunos ejemplos: e. Fase de Evaluación Se plantea una situación nueva articulada a los temas tratados. Se realiza la autoevaluación del estudiante y la coevaluación entre pares, entendidas como espacios de aprendizaje: aprendizaje y evaluación como proceso recursivo. Acciones del docente Acciones del estudiante El docente evalúa el desempeño del estudiante a través del seguimiento de sus actuaciones y productos obtenidos, desde la aparición de los primeros borradores hasta el producto final. El estudiante realiza la autoevaluación y la coevaluación entre pares como instancias de aprendizaje: aprendizaje y evaluación como proceso recursivo. Puede solicitar trabajos adicionales con el propósito de obtener más datos evaluativos y permitir la transferencia y la nivelación. Anticipa una nueva secuencia articulada con los temas y/o contenidos tratados. Ejemplo: a. El docente, en base a la situación anterior, brinda información adicional a los estudiantes. Dicha información genera un mayor nivel de dificultad para la resolución del problema. Un día, Ana llegó con dos minutos de retraso al paradero de la moto. Ello ocasionó que su viaje en combi demorara 8 minutos más de lo habitual y su viaje en tren Formaliza conceptos y explica: Estudiante: inicio en cero la recta numérica de los números naturales porque ello me permite graficar adecuadamente el espacio que corresponde al número. Encuentra los errores, refuta y generaliza: Estudiante: al graficar la suma de dos números naturales debes partir de cero”. 0 3 61 4 7 92 5 8 10 0 4 +2 3 61 42 5
  • 40. 40 Módulo de Orientaciones para el docente del Área de Matemática de EBA 10 minutos más de lo acostumbrado. ¿Cuánto más tardó Ana en llegar a Villa El Salvador? b. Los estudiantes utilizan el reloj de manecillas, que han elaborado, para comunicar: • ¿Cuántos minutos demoran en recorrer una de sus rutas habituales? • ¿Quién demora más? ¿Quién demora menos? ¿Cuál es la diferencia de tiempo entre ambos? Las fases mencionadas pueden ser utilizadas en el desarrollo de las diferentes competencias matemáticas de cualquiera de los tres ciclos de Educación Básica Alternativa. A lo largo de ellas observamos que el estudiante traduce la situación, interpreta, realiza representaciones simbólicas, discute sus supuestos, comunica sus hallazgos y conclusiones, socializa sus resultados, identifica errores propios y de sus compañeros, refuta y generaliza superando los errores y el modelo intuitivo instalado para dar lugar a un nuevo modelo mental vía el conflicto cognitivo. Este proceso sucede al surgir discrepancia entre la imagen mental formada anteriormente y la solicitada, los estudiantes utilizan sus habilidades y construyen conocimiento. El docente asesorará al estudiante para que amplíe y/o identifique nuevas situaciones a partir del contexto real del estudiante, asimismo, plantee y resuelva problemas. Suben 2, espera... Lleva, lleva, hace 6 minutos pasó el otro. Déjame 3 cuadras después del parque. Envía un mensaje, avisa que llegamos en 5 minutos.
  • 41. 41Dirección de Educación Básica Alternativa Tal como hemos visto anteriormente, el docente, además de identificar la situación problemática y plantear la experiencia de aprendizaje, debe tener claridad sobre cómo va a enseñar y cuál es la intención que persigue al desarrollar la experiencia de aprendizaje. Presentamos como propuesta la realización de la investigación en matemática, a través de un ejemplo, planificado para los estudiantes de Tercer Grado del Ciclo Intermedio de EBA. Analicemos información previa: El ciclo de la investigación se inicia motivando al estudiante a hacer preguntas sobre sí mismo, su entorno familiar, local u otro más amplio. Luego, elaboran un plan, recolectan datos por su propia cuenta o hacen uso de datos ya existentes en distintas fuentes. En grupo, los estudiantes, analizan los datos recolectados, construyen tablas, gráficos, buscan patrones, hacen inferencias, predicciones para sacar conclusiones; interpretan, comunican y generan nuevas preguntas. Fases de la Investigación La investigación2 Planteamiento del problema Desarrollo del plan Recolección y manejo de datos Fase de conclusiones Análisis de datos a. b. c. e. d.
  • 42. 42 Módulo de Orientaciones para el docente del Área de Matemática de EBA A continuación desarrollamos una experiencia de aprendizaje utilizandola investigación. Veamos la secuencia ejemplificada para cada fase: Experiencia de Aprendizaje para el Ciclo Intermedio de EBA Losestudiantesresolveránlasituaciónproblemática:identificar ycomunicarinformación estadística referida a la importancia de conocer el clima a través de la “Investigación”. Ciclo: Intermedio de EBA Grado: 3ro Área: Matemática, Área: CAS Componente del Área: Estadística y probabilidad Competencia, aprendizajes a lograr e indicadores: COMPETENCIA APRENDIZAJES A LOGRAR INDICADOR Recolecta y organiza datos, construye e in- terpreta gráficos esta- dísticos referentes a si- tuaciones y fenómenos de su entorno (natural, económico, social) va- lorando la importancia del lenguaje gráfico en la vida cotidiana. Elabora gráficos de barras con datos referidos a situaciones cotidianas y comunica el proceso que utiliza. Interpreta diagramas, esquemas, tablas, gráficos de barras y pictogramas. Emplea procedimientos de recolección de datos: preguntas orales y escritas, encuestas, registro de hechos. Plantea relaciones entre los datos (cualitativos y cuantitativos) en situaciones de contexto personal, expresándolos en tablas simples de conteo, barras simples o pictogramas (con escala dada) Responde a preguntas sobre información de tablas, pictogramas, gráficos de barras simples, con datos cuantitativos y cualitativos Expresa sus conclusiones respecto a la información obtenida. Situación problemática: emplear y comunicar información estadística referida al clima Los estudiantes de EBA necesitan comprender y utilizar la información, de carácter estadístico, que se presenta en distintos medios de difusión: periódicos, revistas, noticieros, encartes, etc. Éstas sonoportunidades para movilizar yponer en funcionamiento sus conocimientos, estrategias y re- cursos; por ende, desarrollar su pensamiento matemático. La información que se presenta en los medios es muy variada, por lo que es importante que el docente seleccione aquella que es relevante para el estudiante joven y adulto; por ejemplo, aquella que está asociada a sus demandas de carácter laboral, ciudadano, ambiental, social, económico, cuidado de la salud, etc.
  • 43. 43Dirección de Educación Básica Alternativa En este caso elegimos información referida al clima, dado que por lo general el estudiante de EBA está atento a conocer cómo evoluciona; tanto para vestirse apropiadamente como para cuidar su salud y la de su familia o anticipar las condiciones climáticas favorables a su trabajo o quehacer económico: venta de alimentos, cosecha, siembra, reproducción de animales, etc. La comprensión y utilización de información estadística y cuantitativa referida al clima, reta al estudiante en relación a utilizar apropiadamente la matemática: identificación y elaboración de gráficos estadísticos, gestión de datos, comunicación matemática, etc. A lo largo de este proceso desarrolla competencias matemáticas. Saberes previos El docente inicia la experiencia de aprendizaje, a través de la investigación, dinamizando un proceso de diálogo orientado a que los estudiantes exploren sus saberes previos: Las preguntas y comentarios deben centrarse en la expresión, por parte del estudiante, de información cuantitativa, asimismo, comunicarla a terceros: • ¿Cómo está el clima hoy? ¿Está más o menos caluroso que ayer? ¿Es más frio que el mes pasado? • ¿Es importante para ustedes conocer con anticipación cómo estará el clima? ¿Por qué? • ¿A qué personas les puede interesar conocer cómo evolucionará el clima? ¿En qué les beneficia ello? Secuencia de la Investigación a. Planteamiento del problema Veamos con ejemplo cada una de las fases de la investigación. El docente presenta una situación o problema a los estudiantes, ellas y ellos se organizan en grupos para expresar su comprensión b. Desarrollo del plan El objetivo de esta fase es que los estudiantes conozcan el tema de estudio que van a abordar, asimismo que planteen alternativas y opten por una. En este caso asumiremos que los estudiantes han consensuado en que una forma de verificar si es importante para los jóvenes y adultos conocer con anticipación las condiciones climáticas y tendencias de Consideramos que las personas jóvenes y adultas, debido a las ocupaciones laborales y familiares que tienen necesitan conocer con anticipación el estado del clima y cómo evoluciona, no solo de su localidad sino en otros lugares. Por ejemplo, para adquirir ropa de acuerdo a la estación y venderla en un lugar específico, para comercializar alimentos apropiados a la estación, sembrar, cosechar, criar animales, cuidar su salud y la de su familia o simplemente vestirse apropiadamente. Asumimos que es importante para los jóvenes y adultos conocer con anticipación las condiciones climáticas y las tendencias de su evolución ya que ello contribuye en su quehacer económico, laboral y familiar ¿Cómo verificamos esta hipótesis?
  • 44. 44 Módulo de Orientaciones para el docente del Área de Matemática de EBA su evolución es preguntándoles directamente. Para ello deciden entrevistarlos a un grupo de ellos en sus domicilios, un día domingo. Los estudiantes imaginan las entrevistas y analizan lo que puede suceder. Deciden que es conveniente realizar la entrevista a través de un cuestionario que elaborarán con anticipación y aplicarán a una muestra de jóvenes y adultos de 15 a más años. En esta fase es importante que los estudiantes decidan e identifiquen la muestra y posibles variables, también es parte de esta fase el diseño de un instrumento para el recojo de información, en este caso un cuestionario que aplicaran en el momento de realizar la entrevista a los jóvenes y adultos de 15 a más años. Los estudiantes: • Forman equipos de 4 a 5 estudiantes. • Seleccionan el trabajo a investigar y se documentan sobre el tema de estudio. • Diseñan una encuesta sencilla (4 a 6 preguntas) para recoger la información que necesitan. Dos datos útiles a considerar son la edad y el sexo. • Cada equipo recoge los datos a través de una encuesta. • En cada pregunta los estudiantes deben reconocer la variable que se está analizando y su tipo. • Contrastan las tabas elaboradas, que deben ser iguales para todos, y corrigen los errores. Instrucciones: estimado vecino, esta encuesta nos ayudará a conocer sobre la importancia que tiene para usted conocer con anticipación información sobre el clima. Señalar con ( X ) 1. Edad: De 15 a 24 ( ) De 25 a 34 ( ) De 35 a 44 ( ) De 45 a 54 ( ) De 55 a 64 ( ) De 65 a 74 ( ) De 75 a más ( ) 2. Sexo: Mujer ( ) Varón ( ) 3. ¿Le interesa conocer información sobre el clima? Sí ( ) No ( ) 4. ¿Por qué le interesa conocer información sobre el clima? a. Para utilizarla en mi trabajo. ( ) b. Para cuidar mi salud y la de mi familia. ( ) c. Para vestirme apropiadamente. ( ) d. Otras (indicar cuál) ……………………………………. 5. A través de qué medios te informas sobre el clima Diario ( ) Radio ( ) TV ( ) Otro (indicar cuál) ………………………………………………………
  • 45. 45Dirección de Educación Básica Alternativa c. Recolección y manejo de datos Los estudiantes se organizan antes de realizar la encuesta y toman decisiones respecto a acciones que deben realizar antes, durante y después de la encuesta; por ejemplo: • ¿Cómo nos vamos a organizar para realizar la encuesta? • ¿Cómo nos vamos a organizar para procesar la data? • ¿Quiénes integrarán los equipos de encuestadores? ¿tendrán alguna identificación y distintivo? • ¿Cuál será el ámbito de la encuesta? ¿a cargo de qué ámbito estará cada equipo? • ¿Cuál es exactamente la población a encuestar? En este caso son personas de 15 a más años ¿habrá una edad límite? • ¿Qué día realizaremos la encuesta? ¿De qué hora a qué hora se realizará? Durante este proceso los estudiantes deben reconocer con claridad la población, la muestra y las variables. d. Análisis de datos El docente debe monitorear y orientar esta fase asegurando el cumplimiento adecuado de las acciones a realizarse, por ejemplo: • La distribución equitativa de las encuestas en cada equipo. • El correcto llenado de las tablas en base a las encuestas asignadas, por parte de cada integrante de los equipos. • La unificación en una sola tabla de la información procesada por cada integrante del equipo. Esta acción puede estar a cargo del coordinador del equipo o un representante que designen. • La elaboración de las conclusiones en cada uno de los equipos. • La presentación y socialización de las conclusiones por parte de cada equipo. En este punto es necesario que los estudiantes analicen modelos de cómo se presenta la información. e. Fases de conclusiones En esta fase los estudiantes: • Desarrollan habilidades orientadas al desarrollo de competencias matemáticas. En este caso, habilidades de analizar datos, extraer conclusiones, interpretar un dato en su contexto, plantear afirmaciones, etc. • Argumentan su opinión en función a los datos obtenidos a lo largo del proceso vivenciado. Actividades de extensión Luego de realizar la investigación los estudiantes analizan otras situaciones problemática, en texto escrito, cuadros y gráficos estadísticos. Se recomienda que en un inicio estén referidos al clima y luego se amplíen hacia otras situaciones.
  • 46. 46 Módulo de Orientaciones para el docente del Área de Matemática de EBA Es importante recordar que la información cuantitativa que se presente debe estar resaltada y, cuando corresponda, los datos modificados para garantizar un tratamiento y solución adecuada al nivel de complejidad que retará a los estudiantes. El docente presenta la situación, lee el texto y lo coloca en un lugar visible del aula. Resalta la información cuantitativa y la repite para asegurarse que el estudiante la entendió. Veamos un ejemplo: Docente: Luego de que los estudiantes dialogan en base a las preguntas presentadas u otras similares, el docente presenta información gráfica sobre el clima. Veamos: Docente: En un diario de circulación nacional se ha presentado la siguiente información referida al clima: TEMPERATURA y RADIACIÓN SOLAR LIMA, CUSCO, ICA, HUANCAYO, TACNA, AREQUIPA, MOQUEGUA y PIURA Luego el docente dinamiza la realización de actividades en base a la información presentada. Veamos: En una escala de 1 a 10, considerando que 1 es frio y 10 es caluroso ¿En qué número ubicarías el clima de hoy? Al utilizar la escala de 1 a 10 ¿Todos tenemos la misma percepción respecto a la medición del clima? ¿Hay alguna convención o acuerdo establecido para medir el clima? ¿Dónde encontramos información sobre el clima? ¿Cómo comunicarías gráficamente el clima de diferentes regiones de nuestro país?
  • 47. 47Dirección de Educación Básica Alternativa Actividades 1. Analizar el gráfico y compartir apreciaciones Los estudiantes analizan el grafico y comparten sus interpretaciones. El docente debe dejar que este proceso fluya y surjan correcciones entre los estudiantes, cuidando el diálogo asertivo y el compartir afirmaciones con sustento. Finalmente, de no lograrse claridad, el docente cierra esta parte reforzando la interpretación correcta del cuadro de Temperatura y Radicación Solar. Veamos: “Para cada ciudad, los valores numéricos que aparecen debajo del cuadro de la izquierda indican, respectivamente, el valor mínimo y máximo de la temperatura. El cuadro de la derecha indica el nivel de radiación solar”. 2. Elaborar cuadros para presentar parte de la información del cuadro 3. Presentar la información del cuadro utilizando un Gráfico de Barras Temperaturas y radiación solar Ciudad Temperatura Radiación solar+ Baja + Alta Diferencia Lima 20° 28° 8° Muy alto Cusco 9° 18° 9° Alto 4 8 12 16 20 24 28 temperatura Variación de temperaturas Ciudades Lima Cusco
  • 48. 48 Módulo de Orientaciones para el docente del Área de Matemática de EBA Dadalaimportanciaypertinenciadelaresolucióndeproblemasenelaprendizajedelamatemática es fundamental que los docentes preparemos a los estudiantes para enfrentar sistemáticamente la solución de problemas y ser conscientes del proceso seguido. Presentamos como propuesta la Resolución de Problemas, a través de un ejemplo para ser aplicado en el área de matemática con los estudiantes de Primer Grado del Ciclo Avanzado de Educación Básica Alternativa. Analicemos información previa Veamos cada uno de los 4 pasos 1. FAMILIARIZACIÓN: comprensión del problema En esta fase el estudiante debe lograr comprender el problema. Se recomienda: • Asegurar la lectura atenta del problema por parte del estudiante. Dar espacio para que lo lea con tranquilidad. • Que el estudiante exprese el problema con sus propias palabras, éste discurso oral no necesariamente guardará el rigor de la formalidad exigida pero si evidenciará el entendimiento de los elementos involucrados en el problema y lo que se pretende resolver. • Que el estudiante explique a otro compañero de qué trata el problema, utilizando sus propias palabras. • Respetar el ritmo de aprendizaje del estudiante, sin presiones, ni apresuramientos, que juegue con la situación, que pierda el miedo inicial. Veamos algunas preguntas que el docente puede hacer a los estudiantes para facilitar la comprensión del problema: • ¿De qué trata el problema? • ¿Has visto alguna situación parecida? • ¿Qué es lo que piden? ¿Cuál es la incógnita? • ¿Cuáles son las condiciones? ¿La condición es suficiente para determinar la incógnita? ¿Es contradictoria? ¿Es redundante? 2. BÚSQUEDA DE ESTRATEGIAS: diseño o adaptación de una estrategia Durante esta fase el estudiante explora la situación. En ese momento necesita poner en valor una serie de estrategias heurísticas que le puedan ser útiles, además, saber elegir la más adecuada, dependiendo de la estructura del problema. Resolución de problemas3 George Polya (1887 – 1985). Matemático húngaro, fue uno de los primeros investigadores que se dedicó a trabajar sistemáticamente la resolución de problemas. En el año 1945 publicó el libro: Cómo plantear y resolver problemas (How to Solve It), en el libro presenta los llamados 4 pasos de Polya para resolver problemas.
  • 49. 49Dirección de Educación Básica Alternativa Esta fase es una de las más importantes en el proceso de solución de problemas pues depende mucho de la base de conocimientos así como de la calidad del pensamiento matemático. Algunas preguntas que el docente puede hacer en esta fase son: • ¿Te has encontrado con un problema semejante? ¿O has visto el mismo problema planteado de forma ligeramente diferente? • ¿Conoces un problema relacionado con este? • ¿Conoces alguna propiedad que te pueda ser útil? • ¿Has resuelto antes un problema similar? ¿Se puede aplicar el método que empleaste para solucionarlo al problema actual? ¿Puedes usar su resultado? • ¿Puedes enunciar el problema de otra manera? ¿Puedes plantearlo de forma distinta? En ese caso: ¿cambia la terminología?, ¿hay nuevas definiciones? Algunas sugerencias pueden ser: • Hazte un esquema, una figura o un diagrama para representar el problema. • Supón el problema resuelto: ¿cuál sería el resultado? ¿cómo crees que lo solucionaron? • Si no puedes resolver el problema propuesto, trata de resolver antes uno similar. • Date ejemplos de la situación. Experimenta. Particulariza, recuerda que empezar por lo fácil hace fácil lo difícil. • Imagínate un problema análogo pero más sencillo. • Resuelve una parte del problema. • Considera sólo una parte de la condición, descarta la otra parte. • Empieza al revés, usa el razonamiento regresivo. • ¿Has empleado todos los datos? ¿Has empleado toda la condición? 3. EJECUCIÓN DEL PLAN: Ejecución de la estrategia Una vez comprendido el problema, lo que se pretende lograr y decidido el camino a seguir para su solución, se procede a ejecutar la estrategia de solución. Durante este proceso entran a tallar los mecanismos de regulación mental y la habilidad para salir de bloqueos. Es recomendable que el estudiante: • Ejecute su plan de solución. • Compruebe cada uno de los pasos: el problema ha sido resuelto ¿estás seguro? ¿Cómo lo compruebas? • Actúe con flexibilidad, es decir, cambie de estrategia si las cosas se complican demasiado. • Aprenda a ser perseverante y variado, es decir, por una parte no se debe abandonar un aspecto examinado antes de que nos haya sugerido algo útil, por otro es necesario examinar tantos aspectos como sea posible, intenta ver siempre algo nuevo.
  • 50. 50 Módulo de Orientaciones para el docente del Área de Matemática de EBA 4. VISIÓN RETROSPECTIVA El estudiante debe aprender a mejorar sus habilidades para enfrentarse con problemas. Los psicólogos e investigadores señalan a esta fase como la principal para que la persona adquiera el conocimiento de sus procesos mentales así como sus preferencias y emociones a lo largo del proceso de solución. La solución de un problema involucra emociones diversas, estos sentimientos pueden impulsar o bloquear a la persona. Durante la fase de familiarización con el problema el estudiante suele experimentar una tensión natural ante la búsqueda de un plan de resolución, tensión que puede desembocar en interés o ansiedad. Cuando se produce la inspiración se tienen sentimientos positivos que cobran más o menos intensidad según las expectativas que se tengan sobre el éxito de dicho plan. El estudiante debe examinar a fondo el proceso seguido y preguntarse: • ¿En qué momento me quedé bloqueado? • ¿Cómo logré salir del bloqueo? • ¿Cómo llegué a la solución? • ¿Puedo verificar cada paso seguido? • ¿Por qué este camino me llevó a la solución? • ¿Qué pista me ayudó a decidir la estrategia a usar?: un dato, algún problema similar, algún modelo. Comprensión del problema Diseño o adaptación de una estrategia Ejecución de una estrategia Visión retrospectiva SÍ NO
  • 51. 51Dirección de Educación Básica Alternativa Es importante que el estudiante aprenda a: • Distinguir entre el problema en sí y la verificación de los procesos generales de su solución. • Reflexionar sobre sus emociones y estrategias de pensamiento. • Generar experiencia para el futuro: cada vez que resuelve un problema está desarrollando habilidades de solución y de trabajo con la matemática. Durante el proceso de resolución de problemas los docentes debemos observar a los estudiantes para: • Darnos cuenta de sus errores. • Identificar cómo convertir los errores en oportunidades para aprender. • Identificar las estrategias que emplean y cómo actúan los estudiantes al resolver problemas: ¿son impulsivos? ¿se bloquean? A continuación desarrollamos una experiencia de aprendizaje utilizando la Resolución de Problemas, según lo planteado por Polya. Para ello presentamos algunos problemas que se plantean a los estudiantes, previo a ello el docente debe haber identificado situaciones problemáticas relacionadas y explorado los saberes previos, tal como lo hemos hecho en los ejemplos para el ciclo inicial e intermedio (puntos 3.1 y 3.2) Experiencia de Aprendizaje para el Ciclo Avanzado de EBA Los estudiantes resolverán la situación problemática: a través de la “Resolución de Problemas”. Ciclo: Avanzado de EBA Grado: 1ro Área: matemática Componente del Área: Geometría y medida Competencia, aprendizajes a lograr e indicadores: COMPETENCIA APRENDIZAJES A LOGRAR INDICADOR Elabora estrategias y técnicas para medir o estimar el valor de una magnitud correspondiente a un objeto o fenómeno de su entorno inmediato, con unidades de longitud, superficie, volumen, masa, tiempo o unidades angulares, mostrando curiosidad, interés y seguridad al realizar su trabajo. Interpreta, identifi- ca y relaciona uni- dades de longitud, masa, superficie, tiempo y volumen en el contexto de la vida diaria. Interpreta datos y relaciones no explicitas respecto a la localización de lugares o desplazamientos de objetos, expresándolos en un croquis en el primer cuadrante del plano cartesiano. Emplea el plano cartesiano al resolver problemas de localización. Aplica las propiedades de las figuras bidimensionales (círculo, circunferencia) al plantear o resolver problemas.
  • 52. 52 Módulo de Orientaciones para el docente del Área de Matemática de EBA Situación problemática: Seguimos los 4 pasos de Polya: 1. FAMILIARIZACIÓN: comprensión del problema Los estudiantes observan el gráfico y se hacen preguntas conducentes a la comprensión plena del problema. Veamos algunos ejemplos de preguntas: • ¿De qué trata el problema? • ¿Has visto alguna situación parecida? • ¿Qué es lo que piden? • ¿Cuál es la incógnita? • ¿Hay suficiente información? • ¿Cuáles son los datos? • ¿Todos los datos son necesarios para resolver el problema? • ¿Qué datos son necesarios? • ¿Qué datos son innecesarios? Problema 1 ¿Cuánto tiempo demorará, aproximadamente, una persona en recorrer las 10 primeras cuadras de la Marcha Juvenil? Recorrido de una Marcha Juvenil
  • 53. 53Dirección de Educación Básica Alternativa Los estudiantes deben lograr comprender el problema e identificar la data relevante que lo caracteriza. 2. BÚSQUEDA DE ESTRATEGIAS: diseño o adaptación de una estrategia Los estudiantes exploran la situación. Analizan posibles estrategias y las ponen en valor, este proceso les permite aprender a elegir la más adecuada. Algunas preguntas que orienten este proceso podrían ser: • ¿Te has encontrado con un problema semejante? • ¿Conoces alguna propiedad que te pueda ser útil? ¿Quizá una forma de simplificar el problema? Los estudiantes deben lograr proponer o adaptar una estrategia, por ejemplo: Suponemos que la Marcha Juvenil avanza a ritmo constante y las cuadras tienen la misma longitud. Esto nos permitirá estimar el tiempo según el número de cuadras avanzadas. 3. EJECUCIÓN DEL PLAN: Ejecución de la estrategia Los estudiantes resuelven: Observan en el gráfico que de la cuadra 33 de la Avenida Brasil a la cuadra 6 la marcha se tarda 2 horas 50 minutos. Es decir: 27 cuadras 2 horas 50 minutos = 60’ + 60’ + 50’ = 170’ 1 cuadra 170’ / 27 = 6’ aproximadamente 10 cuadras 60’ = 1 hora aproximadamente 4. VISIÓN RETROSPECTIVA Los estudiantes se preguntan ¿Comprendí la solución? ¿Hay otras formas de resolverlo? Los estudiantes pueden llegar a conclusiones como las siguientes: Otras formas de resolver el problema es: • Mentalmente • Midiendo los espacios en el gráfico Problema 2 ¿Cómo representaría en un Diagrama Cartesiano la relación entre el espacio y el tiempo basándose en los datos de la Marcha Juvenil?
  • 54. 54 Módulo de Orientaciones para el docente del Área de Matemática de EBA Veamos, de modo abreviado, la resolución del problema siguiendo los 4 pasos de Polya: FAMILIARIZACIÓN: comprensión del problema ¿Puedes plantear el problema con tus propias palabras? BÚSQUEDA DE ESTRATEGIAS: diseño o adaptación de una estrategia Los estudiantes, orientados por el docente, averiguan cómo es un Diagrama Cartesiano y resuelven un problema similar. EJECUCIÓN DEL PLAN: Ejecución de la estrategia Los estudiantes, en base al ejemplo trabajado, hacen su propia representación, de acuerdo a lo solicitado. El Diagrama Cartesiano se utiliza para representar la relación de dos variables, por ejemplo, la cantidad de matracas compradas y el precio a pagar. Gráfico: Oferta de matracas 1 2 3 4 Precio S/. 1 2 3 4 b Matracas (unidades) a Lleve 3 y pague 2 10 1h 2h 3h 4h 20 30 40 Tiempo (horas) 41 Gráfico: Recorrido de la Marcha Juvenil (Espacio y tiempo) Espacio (cuadras Av. Brasil)
  • 55. 55Dirección de Educación Básica Alternativa VISIÓN RETROSPECTIVA Los estudiantes se preguntan y responden en base a reflexiones argumentadas: ¿Qué me ayudó a realizar correctamente la representación gráfica? ¿Puedo utilizar lo aprendido para solucionar otro problema? Veamos, de modo abreviado, la resolución del problema siguiendo los 4 pasos de Polya: FAMILIARIZACIÓN: comprensión del problema ¿Puedes plantear el problema de una manera más sencilla o directa? ¿Cuáles son los datos? ¿Todos los datos son necesarios para resolver el problema? BÚSQUEDA DE ESTRATEGIAS: diseño o adaptación de una estrategia Los estudiantes, al analizar cómo resolver el problema se dan cuenta que ya tienen una parte del total, el 28,6%, y que la diferencia es lo que les falta para completar el 100%, por tanto lo calcularán en base a una regla de tres simple. Además, necesitan recordar cómo se elabora un Gráfico Circular y cómo se utiliza el transportador. Veamos: Para construir un Gráfico Circular debemos recordar que el ángulo de 360° se obtiene de hacer girar una semirecta hasta colocarla en su posición inicial. Ejemplo: en el gráfico de la izquierda la semirecta OA gira 360° Problema 3 Lee la siguiente situación y representa, utilizando un diagrama circular, el porcentaje de familias que no tiene agua potable y el porcentaje de familias que sí tiene. Agua: Derecho de todos ¿Cuántos accedemos? En el mundo cerca de 100 millones de personas no tienen acceso al agua potable. Cada año más de3millonesymediomuerenporenfermedades transmitidas por agua contaminada. La diarrea es una de ellas; mata más niños menores de 5 años que el SIDA, la malaria y la viruela juntos. En Perú: 2 376 534 viviendas (28,6% del total) no tienen agua potable, significa que las familias que las habitan no acceden a este elemento fundamental para la dignidad humana. 360° 0°,360°180° 270° 90° O A
  • 56. 56 Módulo de Orientaciones para el docente del Área de Matemática de EBA EJECUCIÓN DEL PLAN: Ejecución de la estrategia Los estudiantes, en base a la estrategia planificada y lo recordado resuelven el problema: Observan que el resultado debe presentarse como un número natural ya que las personas son individualidades. Resultado: 8 309 559 = 71,4% Representamos en un Gráfico Circular el resultado obtenido y la información dada: 2 376 534 = 28,6% A C36° B Entonces, 28,6% corresponde en un gráfico circular a 103°. La diferencia será 71,4% que corresponde a 257° (360 – 103) ⇒ = 102,96 Redondeando = 103 28,6 x 360 100 Calculamos la región circular (ángulo) que corresponde al porcentaje dado (28,6%) El total → 360° = 100% Una parte → x = 28,6% 270° 90° 180° 360° 103° 28,6% 71,4% 257° 2 376 534 (28,6%) de viviendas no tienen agua potable 8 309 559 (71,4%) de viviendas si tienen agua potable. Recordemos también cómo se utiliza el transportador. ¿Cuántas viviendas de Perú sí tienen acceso al agua potable? Sabemos que 2 376 534 equivale al 28,6%. El total de viviendas equivale al 100%. Si restamos ambos porcentajes obtendremos el porcentaje de viviendas que sí tienen agua potable: 100 – 28,6 = 71,4 Para calcular la cantidad a la que equivale el 71,4% utilizamos la regla de tres simple, veamos: 2 376 534 x 71,4 28,6 = 8 309 559,4⇒ 2 376 534 → 28,6% χ → 71,4%